PolityPreviousYearQuestionBank · 1979. 8. 31.  · PolityPreviousYearQuestionBank PageNo.5 Company...

133
Polity Previous Year Question Bank www.laex.in https://elearn.laex.in Page No. 1

Transcript of PolityPreviousYearQuestionBank · 1979. 8. 31.  · PolityPreviousYearQuestionBank PageNo.5 Company...

Page 1: PolityPreviousYearQuestionBank · 1979. 8. 31.  · PolityPreviousYearQuestionBank PageNo.5 Company can give licence of trade to individuals and its own employees to trade in India:

Polity Previous Year Question Bank

www.laex.in https://elearn.laex.inPage No. 1

Page 2: PolityPreviousYearQuestionBank · 1979. 8. 31.  · PolityPreviousYearQuestionBank PageNo.5 Company can give licence of trade to individuals and its own employees to trade in India:

Polity Previous Year Question Bank

www.laex.in https://elearn.laex.inPage No. 2

Page 3: PolityPreviousYearQuestionBank · 1979. 8. 31.  · PolityPreviousYearQuestionBank PageNo.5 Company can give licence of trade to individuals and its own employees to trade in India:

Polity Previous Year Question Bank

www.laex.in https://elearn.laex.in

1. History and Evolution of Constitution…………………………….. 01

2. Preamble……………………………………………………………….. 11

3. Fundamental Rights……………………………………………..…… 13

4. Directive principle of state policy……………………………..…… 24

5. Fundamental duties……………………………………………...…… 31

6. Executive………………………………………………………..……… 35

7. Legislature……………………………………………………………… 46

8. Judiciary…………………………………………………………..……. 75

9. Constitutional and Non constitutional bodies…………….…..…. 93

10. Emergency Provisions…………………………………………..….. 104

11. Parliamentary System…………………………………………..…… 106

12. Federalism………………………………………………………….….. 108

13. Elections…………………………………………………………….….. 113

14. Amendment of Constitutions…………………………………….… 116

15. Local Government…………………………………………….……… 124

Page 4: PolityPreviousYearQuestionBank · 1979. 8. 31.  · PolityPreviousYearQuestionBank PageNo.5 Company can give licence of trade to individuals and its own employees to trade in India:

Polity Previous Year Question Bank

www.laex.in https://elearn.laex.inPage No. 1

Polity Previous Years Question Bank

1.1 Regulating Act of 1773

1. Consider the following statements:1. Warren Hastings was the first GovernorGeneral who established a regular policeforce in India on the British pattern2. A Supreme Court was established atCalcutta by the Regulating Act, 1773.3. The Indian Penal Code came into effectin the year 1860Which of the statements given above arecorrect?a) 1 and 2b) 2 and 3c) 1 and 3d) 1,2 and 3

1.2 Charter Act of 1793

2. By a regulation in 1793, the Districtcollector was deprived of his judicialpowers and made the collecting agentonly. What was the reason for suchregulation?a) Lord Cornwallis felt that the District

Collector’s efficiency of revenuecollection would enormously increasewithout the burdern of other work.

b) Lord Cornwallis felt that judicial powershould compulsarily be in the hands ofEuropeans while Indians can be giventhe job of revenue collection in thedistricts.

c) Lord Cornwallis was alarmed at theextent of power concentrated in theDistrict Collector and felt that suchabsolute power was undesirable in oneperson.

d) The judicial work demanded a deepknowledge of India and a good trainingin law and Lord Cornwallis felt that the

District Collector should only be arevenue collector.

1.3 Charter Act of 1813

3. Consider the following statementsabout 'the Charter Act of 1813

1. It ended the trade monopoly of theEast India Company in India except fortrade in tea and trade with China.

2. It asserted the sovereignty of theBritish Crown over the Indianterritories held by the Company.

3. The revenues of India were nowcontrolled by the British Parliament.Which of the statements given above

are correct?a) 1 and 2 onlyb) 2 and 3 onlyc) 1 and 3 onlyd) 1,2 and 3

4. Which of the following led to theintroduction of English educationin India?

1. Charter Act of 18132. General Committee of Public

Instruction, 18233. Orientalist and Anglicist controversySelect the correct answer using the codesgiven below:a) 1 and 2 onlyb) 2 onlyc) 1 and 3 onlyd) 1, 2 and 3

01. History and Evolution ofConstitution

Page 5: PolityPreviousYearQuestionBank · 1979. 8. 31.  · PolityPreviousYearQuestionBank PageNo.5 Company can give licence of trade to individuals and its own employees to trade in India:

Polity Previous Year Question Bank

www.laex.in https://elearn.laex.inPage No. 2

1.4 Charter Act of 18335. Which one of the following

provisions was NOT made in theCharter Act of 1833?

a) The trading activities of the East IndiaCompany wereto be abolishedb) The designation of the supremeauthority was tobe changed as theGovernor-General of India-in-Councilc) All law-making powers to be conferred onGovernor-General-in-Councild) An Indian was to be appointed as a LawMember inthe Governor-General's Council

1.5 Government of India Act of 18586. Consider the following statements:1. The Charter Act of 1853 abolished East

India Company’s monopoly of Indiantrade

2. Under the Government of India Act1858, the British Parliament abolishedthe East India Company altogether andundertook the responsibility of rulingIndia directly.

Which of the statements given below is/arecorrect?a) 1 onlyb) 2 onlyc) Both 1 and 2d) Neither 1 nor 2

1.6 Indian Council Act of 1861

7. Which one of the following Act ofBritish India strengthened theViceroy's authority over hisexecutive council by substituting"portfolio" or departmental systemfor corporate functioning?

a) Indian Council Act, 1861b) Government of India Act, 1858c) Indian Council Act, 1892d) Indian Council Act, 1909

1.7 Indian Council Act 19198. The Government of India act of

principle of election in India introducedclearly defined

a) The Separation of power between thejudiciary and the legislature

b) The jurisdiction of central andprovincial governments

c) The powers of Secretary of State forIndia and the Viceroy

d) None of the above

9. Which of the following is/are theprincipal feature(s) of the Governmentof India Act, 1919?1. Introduction of dyarchy in the

executive government in the provinces2. Introduction of separate communal

electorates for Muslims3. Devolution of legislative authority by

the centre to the provincesSelect the correct answer using the codesgiven below:a) 1 onlyb) 2 and 3 onlyc) 1 and 3 onlyd) 1, 2 and 3

1.8Government of India Act, 1935

10. In the federation established bythe Government of India Act, 1935,residuary powers were given to

a) Federal Legislatureb) Governor Generalc) Provincial Legislatured) Provincial Governors

Page 6: PolityPreviousYearQuestionBank · 1979. 8. 31.  · PolityPreviousYearQuestionBank PageNo.5 Company can give licence of trade to individuals and its own employees to trade in India:

Polity Previous Year Question Bank

www.laex.in https://elearn.laex.inPage No. 3

11. Consider the following statementsThe Government of India Act, 1935provided for

1. The provincial autonomy2. The establishment of Federal court3. All India Federation at the Centre

Which of the statements given above arecorrect?

a. 1 and 2b. 2 and 3c. 1 and 3d. 1, 2 and 3

12. Consider the following statements:Some of the main features of theGovernment of India Act, 1935 werethe1. Abolition of diarchy in the Governors'provinces2. Power of the Governors to veto legislativeaction and to legislate on their own3. Abolition of the principle of communalrepresentationWhich of the statements given above is/arecorrect?a) 1 onlyb) 1 and 2c) 2 and 3d) 1,2 and 3

13. The “Instrument of Instructions”contained in the Government ofIndia Act, 1935 have beenincorporated in the Constitutionof India in the year 1950 as?(2010).

(a) Fundamental Rights.(b) Directive Principles of State Policy.(c) Extent of executive power of State.(d) Conduct of business of the

Government of India.

14. Match List-I (Acts of ColonialGovernment of India) with List-III (Provisions) and select thecorrect answer using the codesgiven below the lists

List-I List-II

A. CharterAct, 1813

1. Set up a Board ofControl in Britainto fully regulatethe East India.

B. RegulatingAct

2. Company's trademonopoly in Indiawas ended.

C.Act of 1858

3. The power togovern wastransferred fromthe East IndiaCompany to theBritish Crown.

D.Pitt’s IndiaAct

4. The Company'sdirectors wereasked to present tothe Britishgovernment allcorrespondenceand documentspertaining to theadministration ofthe company.

Codes:(a) A –2; B–4; C– 3; D– 1(b) A –1; B–3; C– 4; D– 2(c) A –2; B–3; C– 4; D– 1(d) A –1; B–4; C– 3; D– 2

15 . Which one of the following is nota feature of the Government ofIndia Act of 1935?

(a) Diarchy at the Centre as well as inthe provinces

(b) A bicameral legislature(c) Provincial autonomy(d) An All-India federation

Page 7: PolityPreviousYearQuestionBank · 1979. 8. 31.  · PolityPreviousYearQuestionBank PageNo.5 Company can give licence of trade to individuals and its own employees to trade in India:

Polity Previous Year Question Bank

www.laex.in https://elearn.laex.inPage No. 4

1.9 Miscellaneous

16. The members of the ConstituentAssembly which drafted theConstitution of India were: (2002)

(a) Nominated by the British Parliament.(b) Nominated by the Governor General.(c) Elected by the Legislative Assemblies

of various province.Elected by the Indian National Congressand Muslim League

17. Consider the followingstatements:

1. The Constitution of India has 40parts.

2. There are 390 Articles in theConstitution of India in all.

3. Ninth, Tenth, Eleventh and TwelfthSchedules were added to theConstitution of India by theConstitution (Amendment) Acts.

Which of the statements given aboveis/are correct?

(a) 1 and 2 only(b) 2 only(c) 3 only(d) 1, 2 and 3

18. The basic structure theory of theConstitution of India implies that?(2018).

(a) Certain features of the Constitutionare so essential to it that they cannotbe abrogated.

(b) Fundamental rights cannot beabridged or taken away.

(c) The Constitution cannot be amendedexcept in accordance with theprocedure prescribed in Article 368.

(d) The Preamble of the Constitutioncannot be amended for it is not a partof the Constitution.

Key & Explanation1. Answer: BExplanation:Statement 1 is incorrect as Lord Curzonwas the first Governor General whoestablished a regular police force in Indiaon the British pattern. A Supreme Courtwas established at Fort Williams by theRegulating Act, 1773 with jurisdiction overBengal, Bihar and Orissa.The code was drafted in 1860 on therecommendations of first law commission ofIndia established in 1834 under theCharter Act of 1833 under theChairmanship of Lord Thomas BabingtonMacaulay. It came into force inBritish India during the early British Rajperiod in 1862Hence option B is right answer.Educational objective: Important acts ofBritish administration. and contribution tothe present ConstitutionSource: Polity-Laxmikanth

2. Answer: CExplanation:Charter Act of 1793 Features It extended the 1st demand of

Cornwallis (above) to all future GGsand Governors of presidencies.

Royal approval mandated forappointment of GG, governors andcommander-in-chief.

Revenue administration was separatedfrom judiciary functionsDisappearance of MaalAdalats

Senior official of company debarredfrom leaving India without permission seen as resignation

Gave GG more control overPresidencies – Bombay and Madras

Trade monopoly of Company extendedfor 20 years

Commander-in-chief not to be a memberof GG-in-Council unless appointed.

Members of Board of Control: Paid outof Indian revenue

Page 8: PolityPreviousYearQuestionBank · 1979. 8. 31.  · PolityPreviousYearQuestionBank PageNo.5 Company can give licence of trade to individuals and its own employees to trade in India:

Polity Previous Year Question Bank

www.laex.in https://elearn.laex.inPage No. 5

Company can give licence of trade toindividuals and its own employees totrade in India: Paved the way forshipment of opium to china

Educational objective: the To know featuresof Charter Act of 1793 and its impact onjudicial systemSource: Polity-Laxmikanth

3 Answer: AExplanation:Monopoly of East India Company to tradewith India was brought to an end but thecompany retained the trade with China andthe trade in tea. Hence statement 1 iscorrect.The company’s shareholders were given a10.5 percent dividend on the revenue inIndia.The company was allowed to continuedterritorial possession for period of 20 yearsi.e. from 1833 to 1853, without prejudice tothe sovereignty of the Crown. The companywas held in trust to the Crown. Hencestatement 2 is correct.The Charter Act of 1813 required theCompany to maintain its territorial andcommercial accounts separately. By thisAct the Company was put under greatercontrol of the British Crown, but couldretain its hold on the details ofadministration and Indian revenues. From1853, the revenues of India were controlledby the British Parliament. Hence,Statement 3 is not correct.Educational objective: To know aboutCharter acts and their provision.Source: Polity-Laxmikanth

4. Answer: DExplanation:In 1823, the Governor-General-in Councilappointed a “General Committee of PublicInstruction”, which had the responsibilityto grant the one lakh of rupees foreducation. That committee consisted of10(ten) European members of which LordMacaulay were the president. Thecommittee decided to spend major portions

from the grant for the improvement oforiental literature. The British scholarswere divided into two groups on the issue ofdevelopment of education in India. Thesewere orientalists and Anglicits.Educational objective: To know aboutBritish reforms and initiatives towardsimprovisation of education system of Indiaduring British RuleSource: Polity-Laxmikanth

5.Answer: DExplanation: The member was lordMacaulay and we could not even think ofbeing part of that bodyEducational objective: important acts ofBritish administration. and contribution tothe present ConstitutionSource: Polity-Laxmikanth

6. Answer: BExplanation:Charter Acts of 1813 was an Act of theParliament of the United Kingdom whichrenewed the charter issued to the BritishEast India Company, and continued theCompany’s rule in India.However, the Company’s commercialmonopoly was ended, except for the teatrade and the trade with China.In August 1858, the British parliamentpassed an act that set an end to the rule ofthe company. The control of the Britishgovernment in India was transferred to theBritish crown. As the British governmentwas responsible to parliament, the supremebody for India also was the Britishparliament.Educational objective: important acts ofBritish administration. and contribution tothe present ConstitutionSource: Polity-Laxmikant

7. Answer: AExplanation:The legislative council had limited role. Itwas chiefly advisory. No discussion onfinance was permitted.

Page 9: PolityPreviousYearQuestionBank · 1979. 8. 31.  · PolityPreviousYearQuestionBank PageNo.5 Company can give licence of trade to individuals and its own employees to trade in India:

Polity Previous Year Question Bank

www.laex.in https://elearn.laex.inPage No. 6

Even though Indians were nominated,there was no statutory provision for theinclusion of Indians in it.It allowed for the decentralisation ofadministration with the vesting oflegislative power to the presidencies ofBombay and Madras.The power of ordinance given to thegovernor-general gave him absolute powers.Educational objective: important acts ofBritish administration. and contribution tothe present ConstitutionSource: Polity-Laxmikanth

8. Answer: BExplanation:GOI Act,1919, relaxed the central controlover the provinces by demarcating andseparating the central and provincialsubjects. The central and provinciallegislatures were authorised to make lawson their respective list of subjects.Educational objective: important acts ofBritish administration. and contribution tothe present ConstitutionSource: Polity-Laxmikanth

9. Answer: BExplanation:The Act provided a dual form ofgovernment (dyarchy) for the majorprovinces.In each such province, control of some areasof government, (transferred list), weregiven to a Government of ministersanswerable to the Provincial Council. Thetransferred list included Agriculture,supervision of local government, Healthand Education.All other areas of government (reserved list)remained under the control of the Viceroy.The reserved list included Defence, ForeignAffairs, and Communications.Introduction of separate communalelectorates for muslims was under 1909 actHence option B is the answer.Government of India Act of 1919

British for the first time in 1917declared: Objective is to graduallyintroduce responsible government in India Also called as Montagu-Chelmsford

Reforms First time introduced bicameralism and

direct election. These reforms were in response to the

Home Rule Movement Women for first time got right to voteFeatures It separated central and provincial

subjects for lawmaking. Provincial Subjects were divided: 1)

Transferred: administered by Governorwith the aid of ministers who wereresponsible to legislative assembly 2)Reserved: administered by Governorand his executive council (notresponsible to legislature -------Thissystem came to be known as Dyarchy

Indian legislative council now have 2houses: 1) Upper House: Council ofState 2) Legislative Assembly: LowerHouse ------ Majority of members in bothhouses were directly elected

Council of state: Tenure of 5 years andhad only male members

Central legislative Assembly: Tenure of3 years

3/6 members of Viceroy’s executivecouncil (other than commander-in-chief)were to be Indian.

Extended Communal representation:now to Sikhs, Christian etc.

Created new office of HighCommissioner for India in London

Provided for public service commission:Central PSC in 1926

Separated Central and Provincialbudgets

Pay of secretary of state: were beingpaid out of Indian revenue till now, butunder the Act, they were to be paid byBritish exchequer from now on.(undoing Act of 1793)

Page 10: PolityPreviousYearQuestionBank · 1979. 8. 31.  · PolityPreviousYearQuestionBank PageNo.5 Company can give licence of trade to individuals and its own employees to trade in India:

Polity Previous Year Question Bank

www.laex.in https://elearn.laex.inPage No. 7

Educational objective: important acts ofBritish administration. and contribution tothe present ConstitutionSource: Polity-Laxmikanth

10. Answer: BExplnation:The provincial legislatures were givenexclusive power to legislate with respect tomatters in the Provincial list. The federallegislature had the exclusive power to makelaw on matters in the Federal List. Thefederal and the provincial legislatures hadconcurrent jurisdiction with respect tomatters in the Concurrent List.In case of conflict between a provincial lawand a federal law on a matter enumeratedin the Concurrent List, the latter was toprevail, and the former would, to the extentof the repugnancy be void. Residuarypowers were vested in the Governor -General, who could, in his discretion,assign any such power by a publicnotification to the federal legislature or theprovincial legislature .

Government of India Act of 1935Features All India Federation (never came into

existence): Provinces + Princely States Three list system: Central, Provincial

and Concurrent Residuary power to Viceroy Dyarchy abolished at provincial levelResponsible government in provinces(All members were directly elected)

Dyarchy at the Central level (Reservedand Transferred subject system) [neverbecame operational in practice]

Central Legislature:Elections to Councilof state (Permanent body with 1/3rdmember retiring every 3 years) wasdirect and that to Federal Assembly (5year duration) was indirect

Introduced bicameralism at provinciallevel

Extended communal representation:Further to depressed classes, womenand workers

Provided for RBI Provided for Federal PSC, Provincial

PSC and Joint PSC and Federal Court Indian council of Secretary of state was

abolished.Educational objective: important acts ofBritish administration.Source: Polity-Laxmikanth

11. Answer: DExplanation:The Government of India Act 1935 providedfor the provincial autonomy, theestablishment of federal court and all IndiaFederation at the centre.Government of India Act of 1935Features All India Federation (never came into

existence): Provinces + Princely States Three list system: Central, Provincial

and Concurrent Residuary power to Viceroy Dyarchy abolished at provincial level Responsible government inprovinces (All members were directlyelected)

Dyarchy at the Central level (Reservedand Transferred subject system) [neverbecame operational in practice]

Central Legislature:Elections toCouncil of state (Permanent body with1/3rd member retiring every 3 years)was direct and that to Federal Assembly(5 year duration) was indirect

Introduced bicameralism at provinciallevel

Extended communal representation:Further to depressed classes, womenand workers

Provided for RBI Provided for Federal PSC, Provincial

PSC and Joint PSC and Federal Court

Page 11: PolityPreviousYearQuestionBank · 1979. 8. 31.  · PolityPreviousYearQuestionBank PageNo.5 Company can give licence of trade to individuals and its own employees to trade in India:

Polity Previous Year Question Bank

www.laex.in https://elearn.laex.inPage No. 8

Indian council of Secretary of state wasabolished.

Educational objective: important acts ofBritish administration. and contribution tothe present ConstitutionSource: Polity-Laxmikanth

12.Answer: BExplanation:Extended communal representation:Further to depressed classes, women andworkers. Hence third statement is wrong.Government of India Act of 1935Features All India Federation (never came into

existence): Provinces + Princely States Three list system: Central, Provincial

and Concurrent Residuary power to Viceroy Dyarchy abolished at provincial level Responsible government inprovinces (All members were directlyelected)

Dyarchy at the Central level (Reservedand Transferred subject system) [neverbecame operational in practice]

Central Legislature:Elections toCouncil of state (Permanent body with1/3rd member retiring every 3 years)was direct and that to Federal Assembly(5 year duration) was indirect

Introduced bicameralism at provinciallevel

Extended communal representation:Further to depressed classes, womenand workers

Provided for RBI Provided for Federal PSC, Provincial

PSC and Joint PSC and Federal Court Indian council of Secretary of state was

abolishedEducational objective: The importantfeatures of GOI Act of 1935 –all IndiaFederation, dyarchy was abolished in theprovinces, dyarchy at the Centre,bicameralism in provinces, communalrepresentation for SCs, women and workers,

and establishment of Federal andProvincial Public Service Commissions,Reserve Bank and a Federal court.Source: Polity-Laxmikanth

13. Answer: BExplanation: Instrument ofInstructions under the Government ofIndia Act of 1858 were envisioned to act asguiding principles to the Governor-Generalof India and the Governors of Provinces indischarge of their official duties. Accordingto Dr. B R Ambedkar, the DirectivePrinciples of State Policy resemble theInstrument of Instructions under theGovernment of India Act of 1935.Educational Objective: The DirectivePrinciples of State Policy resemble theInstrument of Instructions under theGovernment of India Act of 1935.Source: Indian Polity by Laxmikant (4thEdition) – page no: 8.1

14. Answer: AExplanation: Regulating Act of 1773 -first step taken by the British Governmentto control and regulate the affairs of theEast India Company in India. The politicaland the administrative functions of thecompany were for the first time recognized.This act laid the foundations of centraladministration in India. The features ofthis act are –(i) Governor of Bengal was designated as

the Governor-General of Bengal. AnExecutive Council of 4 members wascreated to assist him.

(ii) Governors of Bombay and Madras weremade subordinate to the Governor-General of Bengal.

(iii) Provided for the establishment of aSupreme court at Calcutta comprisingof one Chief Justice and 3 other Judges.

(iv) Prohibited the servants of the companyfrom engaging in any private trade.

(v) Control of the British Government overthe company was increased. Court ofDirectors had to report on itsrevenue, civil and military affairsin India.

Page 12: PolityPreviousYearQuestionBank · 1979. 8. 31.  · PolityPreviousYearQuestionBank PageNo.5 Company can give licence of trade to individuals and its own employees to trade in India:

Polity Previous Year Question Bank

www.laex.in https://elearn.laex.inPage No. 9

Pitt’s India Act of 1784 – this act wassignificant because - a) for the first-timecompany’s territories in India were calledBritish possessions in India and b) theBritish Government was given supremecontrol over company’s affairs andadministration in India. The features ofthis act are –(i) Distinguished between the political and

the commercial functions of thecompany.

(ii) Court of Directors were allowed tomanage the commercial affairs butthe political affairs were to bemanaged by a new body, Board ofControl. This established a system ofdouble government.

(iii) Board of Control was empowered tosupervise and direct all operations ofthe civil and military government orrevenues of the British possessions inIndia.Charter Act of 1813 – this act for thefirst time explicitly defined theconstitutional position of the Britishterritories in India. The features of thisare -

(i) Ended the Company's commercialmonopoly, except for the tea andopium trade and the trade with China.

(ii) Permitted Christian missionaries topropagate and preach their religion.

(iii) Company should invest Rs. 1 Lakhevery year on the education of Indians.

(iv) Empowered the local governments toimpose taxes on the persons subject tothe jurisdiction of the Supreme Court.The Government of India Act of1858 – this act was also known as theAct for the Good Government of India.It ended the Company’s Rule inIndia and India was brought underthe rule of the British Crown. Thisact was largely confined toimprovement of administrativemachinery by which the IndianGovernment was to be supervised andcontrolled in England. It did not alter inany substantial way the system ofgovernment that prevailed in India. Thefeatures of this act are –

(i) India will be governed in the nameof, Her Majesty.

(ii) Governor-General of India wasdesignated as the Viceroy of India,who is the direct representative ofthe crown.

(iii) System of double government wasended.

(iv) A new office of Secretary of State forIndia was created and a 15-member‘Council of India’ was established tohim.

Educational Objective: Regulating Act of1773 - first step taken by the BritishGovernment to control and regulate theaffairs of the East India Company in India.Pitt’s India Act of 1784 was significantbecause the company’s territories in Indiawere called British possessions in India forthe first-time.Charter Act of 1813 ended the Company'scommercial monopoly, except for the teaand opium trade and the trade with China.The Government of India Act of 1858 wasalso known as the ‘Act for the GoodGovernment of India’. Company’s Rule inIndia was ended and India was broughtunder the rule of the British Crown.Source: Indian Polity by Laxmikant (4thEdition) – page no: 1.1-515. Answer: AExplanation: Features of theGovernment of India (GOI) Act of 1935are -1. It provided for the establishment of

an All-India Federation consisting ofprovinces and princely states as units.The Act divided the powers between theCentre and units in terms of threelists—Federal List, Provincial List andthe Concurrent List. Residuary powerswere given to the Viceroy. However, thefederation never came into being as theprincely states did not join it.

2. It abolished dyarchy in theprovinces and introduced‘provincial autonomy’ in its place.The provinces were allowed to actas autonomous units ofadministration in their defined

Page 13: PolityPreviousYearQuestionBank · 1979. 8. 31.  · PolityPreviousYearQuestionBank PageNo.5 Company can give licence of trade to individuals and its own employees to trade in India:

Polity Previous Year Question Bank

www.laex.in https://elearn.laex.inPage No. 10

spheres. Moreover, the Act introducedresponsible governments in provinces,that is, the governor was required to actwith the advice of ministers responsibleto the provincial legislature. This cameinto effect in 1937 and was discontinuedin 1939.

3. It provided for the adoption ofdyarchy at the Centre. Consequently,the federal subjects were divided intoreserved subjects and transferredsubjects. However, this provision of theAct did not come into operation at all.

4. It introduced bicameralism in sixout of eleven provinces. Thus, thelegislatures of Bengal, Bombay, Madras,Bihar, Assam and the United Provinceswere made bicameral consisting of alegislative council (upper house) and alegislative assembly (lower house).Bicameral legislature at the Centre wasintroduced by GOI Act, 1919.

5. It further extended the principle ofcommunal representation by providingseparate electorates for depressedclasses (scheduled castes), women andlabour (workers).

6. It abolished the Council of India,established by the Government of IndiaAct of 1858 and the Secretary of Statefor India was provided with a team ofadvisors.

7. It extended franchise (about 10 per centof the total population got the votingright).

8. It provided for the establishment of aReserve Bank of India to control thecurrency and credit of the country.

9. It provided for the establishment of notonly a Federal Public ServiceCommission but also a Provincial PublicService Commission and Joint PublicService Commission for two or moreprovinces.

10. It provided for the establishment of aFederal Court, which was set up in1937.

Educational Objective: The importantfeatures of GOI Act of 1935 –all IndiaFederation, dyarchy was abolished in theprovinces, dyarchy at the Centre,

bicameralism in provinces, communalrepresentation for SCs, women and workers,and establishment of Federal andProvincial Public Service Commissions,Reserve Bank and a Federal court.Source: Indian Polity by Laxmikant (4thEdition) – page no: 1.7-8

16.Answer: CExplanation: The Constituent Assemblywas constituted in November 1946 underthe scheme formulated by the CabinetMission Plan. The features of the Schemeare –(i) The total strength of the Constituent

Assembly was to be 389 (296 seats toBritish India and 93 seats to thePrincely States).

(ii) Allocation of seats to a Province or aPrincely State was in proportion totheir population.

(iii) Seats were reserved to Muslims andSikhs in proportion to their populationin British India.

(iv) In British India, representatives wereelected by members of thelegislative assemblies of provincesbased on communal representation.Voting was to be by the method ofproportional representation by means ofsingle transferable vote.

(v) The representatives of the PrincelyStates were to be nominated by theirrespective Heads.

Educational Objective: The members ofthe Constituent Assembly were elected bymembers of the legislative assemblies ofprovinces in British India. However, themembers from the Princely States werenominated by the respective Heads of thePrincely States.Source: Indian Polity by Laxmikant (4thEdition) – page no: 2.1

17. Answer: CExplanation: Originally, the Constitutionof India (1949) contained a Preamble, 395Articles divided into 22 Parts and 8Schedules. Over the years many Articleswere added and repealed and 4 new

Page 14: PolityPreviousYearQuestionBank · 1979. 8. 31.  · PolityPreviousYearQuestionBank PageNo.5 Company can give licence of trade to individuals and its own employees to trade in India:

Polity Previous Year Question Bank

www.laex.in https://elearn.laex.inPage No. 11

Schedules were added through variousConstitutional Amendment Acts. Atpresent, the Constitution of India containsa Preamble, 448 Articles divided into 25Parts and 12 Schedules.Educational Objective: At present, theConstitution of India contains a Preamble,448 Articles divided into 25 Parts and 12Schedules.Source: Indian Polity by Laxmikant (4thEdition) – page no: not available

18. Answer: DExplanation: Article 13 of theConstitution of India says that laws thatare inconsistent with the FundamentalRights are null and void. The question ofthe extent of the power with the Parliamentto amend the Constitution (especiallyFundamental Rights) under Article 368 hascome up before the Supreme court manytimes – Shankari Prasad case (1951), GolakNath case (1967) and Kesavananda Bharaticase (1973). In the Kesavananda Bharaticase, the Supreme court said that theParliament has the power to abridge ortake away any of the Fundamental Rights.However, it also said that the Parliamentunder Article 368 cannot alter the BasicStructure of the Constitution. This meansthat the Parliament cannot amend orabrogate those features of theConstitution which are a part of itsBasic Structure. However, the Supremecourt has not defined the features of theConstitution that are part of its basicStructure. But over the course ofJudgements, the Supreme Court hasincluded the following features of theConstitution as its basic structure –

Supremacy of theConstitution,

Sovereign,democratic andrepublicannature of theIndian polity,

Secularcharacter of theConstitution,

Separation of

Parliamentarysystem,

Rule of law, Harmony

betweenFundamentalRights andDPSP,

Principle ofequality,

powers, Federal

character of theIndian polity,

Unity andintegrity of thenation,

Welfare state, Judicial review,Freedom and dignityof the individual,

Free and fairelections,

Independenceof judiciary,

Limited powerof theParliament toamend theConstitution,

Effective accessto justice,

Principle ofreasonableness,and

Power of theSupreme courtunder Articles32, 136, 141and 142.

Educational Objective: Basic StructureDoctrine was evolved for the first time bythe Supreme court in the KesavanandaBharati case (24th April, 1973).The Parliament does not have the power toamend or abrogate those features of theConstitution which are a part of its BasicStructure.Source: Indian Polity by Laxmikant (4thEdition) – page no: 11.1-2

02. Preamble

1. Which one of the followingobjectives is not embodied in thePreamble to the Constitution ofIndia? (2017)

a) Liberty of thoughtb) Economic libertyc) Liberty of expressiond) Liberty of belief

2. Which one of the following is partof Preamble of the Constitutionof India? (2018).

a) We, the people of India, havingsolemnly resolved to constitute Indiainto a Sovereign Socialist SecularDemocratic Republic .... do herebyAdopt, Enact and Give to ourselvesthis Constitution.

Page 15: PolityPreviousYearQuestionBank · 1979. 8. 31.  · PolityPreviousYearQuestionBank PageNo.5 Company can give licence of trade to individuals and its own employees to trade in India:

Polity Previous Year Question Bank

www.laex.in https://elearn.laex.inPage No. 12

b) We, the members of the Parliament,do hereby Adopt, Enact and Give toourselves this Constitution.

c) We, the people of ConstituentAssembly, do hereby Adopt, Enactand Give to ourselves thisConstitution.

d) None of the above

3. The mind of the makers of theConstitution of India is reflectedin which of the following? (2017).

a) The Preambleb) The Fundamental Rightsc) The Directive Principles of State

Policyd) The Fundamental Duties

4. The word “secular” denotes (2018)a) Keeping away from all religionsb) Belief in one Godc) Freedom of religion and worship to

all citizensd) Practicing different religions

Key & Explanation1. Answer: BExplanation: The Preamble to theConstitution of India is as follows –“We, THE PEOPLE OF INDIA, havingsolemnly resolved to constitute India into aSOVEREIGN SOCIALIST SECULARDEMOCRATIC REPUBLIC and to secureto all its citizens:JUSTICE, Social, Economic and Political;LIBERTY of thought, expression, belief,faith and worship;EQUALITY of status and of opportunity;and to promote among them all;FRATERNITY assuring the dignity of theindividual and the unity and integrity ofthe Nation;IN OUR CONSTITUENT ASSEMBLY thistwenty-sixth day of November, 1949, doHEREBY ADOPT, ENACT AND GIVE TOOURSELVES THIS CONSTITUTION”.

Educational Objective: Preamble assuresus with Liberty of thought, expression,belief, faith and worship only.Source: Indian Polity by Laxmikant (4thEdition) – page no: 4.1

2. Answer: AExplanation: The Preamble to theConstitution of India is as follows –“We, THE PEOPLE OF INDIA, havingsolemnly resolved to constitute India into aSOVEREIGN SOCIALIST SECULARDEMOCRATIC REPUBLIC and to secureto all its citizens:JUSTICE, Social, Economic and Political;LIBERTY of thought, expression, belief,faith and worship;EQUALITY of status and of opportunity;and to promote among them all;FRATERNITY assuring the dignity of theindividual and the unity and integrity ofthe Nation;IN OUR CONSTITUENT ASSEMBLY thistwenty-sixth day of November, 1949, doHEREBY ADOPT, ENACT AND GIVE TOOURSELVES THIS CONSTITUTION”.Educational Objective: To remember thePreamble in its entirety.Source: Indian Polity by Laxmikant (4thEdition) – page no: 4.13. Answer: AExplanation: The Preamble embodies thebasic philosophy and fundamental valueson which the Constitution is based. Itcontains the grand noble vision of theConstituent Assembly and reflects thedreams and aspirations of the foundingfathers of the Constitution.Educational Objective: The Preambleembodies the basic philosophy andfundamental values on which theConstitution is based.Source: Indian Polity by Laxmikant (4thEdition) – page no: 4.44. Answer: CExplanation: The word ‘Secular’ wasadded to the Preamble by the 42ndConstitution Amendment Act of 1976. The

Page 16: PolityPreviousYearQuestionBank · 1979. 8. 31.  · PolityPreviousYearQuestionBank PageNo.5 Company can give licence of trade to individuals and its own employees to trade in India:

Polity Previous Year Question Bank

www.laex.in https://elearn.laex.inPage No. 13

western concept of secularism connotes acomplete separation between the religion(the church) and the State. This negativeconcept of secularism is inapplicable in theIndian situation where the society ismultireligious. Hence, the IndianConstitution embodies the positiveconcept of secularism i.e. giving equalrespect to all religions or protectingall religions equally. The Preamble alsosecures us with the Liberty of thought,expression, belief, faith and worship.Educational Objective: IndianConstitution embodies the positive conceptof secularism i.e. giving equal respect to allreligions or protecting all religions equally.42nd Constitution Amendment Act of 1976 –added 3 new words to the Preamble –Socialist, Secular and Integrity.Source: Indian Polity by Laxmikant (4thEdition) – page no: 3.5

03. Fundamental Rights

3.1 Nature and characteristicfeatures of FR

1. Which one of the followingstatements is correct?

(a) Rights are claims of the State againstthe citizens.

(b) Rights are privileges which areincorporated in the Constitution of aState

(c) Rights are claims of the citizens againstthe State.

(d) Rights are privileges of a few citizensagainst the many

3.2 Right to Equality: Article 14-18

2. In the Indian Constitution, theRight to Equality is granted byfive Articles. They are

(a) Article 16 to Article 20(b) Article 15 to Article 19(c) Article 14 to Article 18(d) Article 13 to Article 17

3. One of the implications ofequality in society is the absenceof

(a) Privileges(b) Restraints(c) Competition(d) Ideology

4. Match List I (Articles of theConstitution of India) with List II(Provision) and select the correctanswer using the codes givenbelow in the lists:

List-I List-II

A. Article14

1. The State shall notdiscriminate againstany citizen on groundsonly of religion, race,caste, sex place of birthor any of term.

B. Article15

2. The State shall notdeny to any personequality before the lawor the equal protectionof laws within theterritory of India.

C. Article16

3. Untouchability isabolished anditspractice in any form isforbidden.

D. Article17

4. There shall be equalityof opportunity for allcitizens in mattersrelating to employmentor appointment to anyoffice under the State

Codes:a) A-2, B-4, C-1, D-3b) A-3, B-1, C-4, D-2c) A-2, B-1, C-4, D-3d) A-3, B-4, C-1, D-2

Page 17: PolityPreviousYearQuestionBank · 1979. 8. 31.  · PolityPreviousYearQuestionBank PageNo.5 Company can give licence of trade to individuals and its own employees to trade in India:

Polity Previous Year Question Bank

www.laex.in https://elearn.laex.inPage No. 14

3.3 Inferred Rights – Article 215. Which Article of the Constitution

of India safeguards one’s right tomarry the person of one’s choice?

(a) Article 19(b) Article 21(c) Article 25(d) Article 29

6. Which one of the followingreflects the nicest, appropriaterelationship between law andliberty?

a) If there are more laws, there is lessliberty.

b) If there are no laws, there is noliberty.

c) If there is liberty, laws have to bemade by the people.

d) If laws are changed too often, libertyis in danger.

7. Right to Privacy is protected asan intrinsic part of Right to Lifeand Personal Liberty. Which ofthe following in the Constitutionof India correctly andappropriately imply the abovestatement?

(a) Article 14 and the provisions underthe 42nd Amendment to theConstitution.

(b) Article 17 and the DirectivePrinciples of State Policy in Part IV.

(c) Article 21 and the freedomsguaranteed in PartIII.

(d) Article 24 and the provisions underthe44th Amendment to theConstitution.

8. In the context of polity, whichone of the following would you acceptas the most appropriate definition ofliberty? (2019)(a) Protection against the tyranny of

political rulers.(b) Absence of restraint.

(c) Opportunity to do whatever one likes.(d) Opportunity to develop oneself fully

9. Consider the followingstatements:

1. Free and compulsory education to thechildren of 6-14 years age-group bythe State by the seventy- sixthAmendment to the Constitution ofIndia.

2. Sarva Shiksha Abhiyan seeks toprovide computer education even inrural areas.

3. Education was included in theConcurrent List by the Forty-secondAmendment, 1976 to the Constitutionof India.

Which of the statements given aboveare correct?

(a) 1, 2 and 3(b) 1 and 2(c) 2 and 3(d) 1 and 3

3.4 Right against Exploitation – Art23, 24

10. Which of the following areenvisaged by the Right againstExploitation in the Constitutionof India?

1. Prohibition of traffic in human beingsand forced labour.

2. Abolition of Untouchability.3. Protection of the interests of

minorities.4. Prohibition of employment of children

in factories and minesSelect the correct answer using the

code given below:(a) 1,2 and 4 only(b) 2, 3 and 4 only(c) 1 and 4 only(d) 1,2, 3 and 4

Page 18: PolityPreviousYearQuestionBank · 1979. 8. 31.  · PolityPreviousYearQuestionBank PageNo.5 Company can give licence of trade to individuals and its own employees to trade in India:

Polity Previous Year Question Bank

www.laex.in https://elearn.laex.inPage No. 15

11. Which Article of the Constitutionof India says, no child below theage of fourteen years shall theemployed to work in any factoryor mine or engaged in any otherhazardous employment?

(a) Article 24(b) Article 45(c) Article 330(d) Article 368

3.5 Cultural and Educational Rights –Art 29 and 30

12. In India, if a religioussect/community is given thestatus of a national minority,what special advantages it isentitled to?

1. It can establish and administerexclusive educational institutions.

2. The President of India automaticallynominates a representative of thecommunity to Lok Sabha.

3. It can derive benefits from the PrimeMinister's 15-Point Programme.

Which of the statements given aboveis/are correct?(a) 1 only(b) 2 and 3 only(c) 1 and 3 only(d) 1, 2 and 3

13. Match List-I (Article of IndianConstitution) with List -II(Provisions) and select thecorrect answer using the codesgiven below in the lists:

List-I List-II

A. Article 16

1. No person shall bedeprived of hisproperty save bythe authority oflaw.

B. Article 29 (2)

2. No person can bediscriminatedagainst in thematter of publicappointment on theground of race,religious or caste.

C. Article 30 (I)

3. All minoritieswhether based onreligion orlanguage shallhave to establishand administereducationalinstitutions of theirchoice.

D. Article 31 (I)

4. No citizen shall bedenied admissioninto anyeducationalinstitutionmaintained by theState, or receivingState aid, ongrounds of religion,race, caste,language or any ofthem.

Codes:(a) A-2, B-4, C-3, D-1(b) A-3, B-1, C-2, D-4(c) A-2, B-1, C-3, D-4(d) A-3, B-4, C-2, D-1

3.6 Right to Constitutional Remedies:-Article 32

14. Which one of the following rightswas described by Dr. B.R.Ambedkar as the heart and soulof the Constitution?

(a) Right to Freedom of Religion(b) Right to Property(c) Right to Equality(d) Right to Constitutional Remedies

Page 19: PolityPreviousYearQuestionBank · 1979. 8. 31.  · PolityPreviousYearQuestionBank PageNo.5 Company can give licence of trade to individuals and its own employees to trade in India:

Polity Previous Year Question Bank

www.laex.in https://elearn.laex.inPage No. 16

3.7 Rights outside Fundamental Rights15. Right to vote and to be elected in

India is a(a) Fundamental Right(b) Natural Right(c) Constitutional Right(d) Legal Right16. British citizen staying in India

cannot claim the right toa) Freedom of trade and professionb) Equality before the Lawc) Protection of life and personal libertyd) Freedom of religion

17. Consider the followingstatements:

1. Article 301 pertains to the Right toProperty.

2. Right to Property is a legal right butnot a Fundamental Right.

3. Article 300A was inserted in theConstitutional Amendment.

Which of the statement given aboveis/are correct?

a) 2 onlyb) 2 and 3c) 1 and 3d) 1, 2 and 3

Key & Explanation1. Answer: DExplanation: A mere declaration offundamental rights in the Constitution ismeaningless without providing an effectivemachinery for their enforcement, if andwhen they are violated. Hence, Article 32confers the Right to remedies for theenforcement of the fundamental rights ofan aggrieved citizen. In other words, theright to get the Fundamental Rightsprotected is in itself a fundamental right.That is why Dr B R Ambedkar calledArticle 32 as the most importantarticle of the Constitution – an articlewithout which the Constitution would be anullity. The Supreme court acts as the

guarantor and protector of theFundamental Rights. The Supreme courthas said Article 32 is a basic feature of theConstitution.Article 32 – Remedies for enforcement ofrights conferred are -

1. The right to move the Supremecourt for the enforcement of theFundamental Rights.

2. The Supreme court shall have thepower to issue directions or ordersor writs, including writs of habeauscorpus, mandamus, prohibition, quowarranto and certiorari for theenforcement of the FundamentalRights.

3. The Parliament may by lawempower any other court to exercisethe same powers of the Supremecourt in the enforcement of theFundamental Rights.

4. The rights guaranteed by thisarticle shall not be suspendedexcept as otherwise provided by theConstitution.

Educational Objective: Article 32 ofthe Constitution of India –Fundamental Right of Right toremedies.The right to move the Supreme court inthe first instance itself for enforcementof Fundamental Rights.Source: Indian Polity by Laxmikant(4th Edition) – page no: 7.17Right to Equality: Article 14-18

2. Answer: CExplanation: The provisions relatingto the Right to Equality are presentin the following articles of theConstitution of India –

(i) Article 14 – the state shall not denyto any person equality before law or theequal protection of laws within theterritory of India. This provision confersrights on all persons whether citizens orforeigners.

I. Article 15 –

Page 20: PolityPreviousYearQuestionBank · 1979. 8. 31.  · PolityPreviousYearQuestionBank PageNo.5 Company can give licence of trade to individuals and its own employees to trade in India:

Polity Previous Year Question Bank

www.laex.in https://elearn.laex.inPage No. 17

a) the state shall not discriminateagainst any citizen on grounds onlyof religion, race, caste, sex or placeof birth.

b) no citizen shall be subjected to anydisability, liability, restriction orcondition on grounds only of religion,race, caste, sex or place of birth withregard to –

(i) Access to shops, public restaurants,hotels and places of publicentertainment; or

(ii) The use of wells, tanks, bathingghats, road and places of publicresort maintained wholly or partlyby state funds or dedicated togeneral public use.This provision (b) prohibitsdiscrimination both by the Stateand private individuals, while theformer provision (a) prohibitsdiscrimination only by the state.

(III) Article 16 – provides for equalityof opportunity for all citizens inmatters of employment orappointment to any office under thestate. No citizen can bediscriminated against or beineligible for any employment oroffice under the state on groundsonly of religion, race, caste, sex,decent, place of birth or residence.

(IV) Article 17 – abolishesuntouchability and forbids itspractice in any form. Theenforcement of any disabilityarising out of untouchability shallbe an offence punishable inaccordance with law.

(IV) Article 18 – abolition of titles.Educational Objective: Right toEquality is covered by 5 Articles of theConstitution – Articles 14,15,16,17,18.Source: Indian Polity by Laxmikant(4th Edition) – page no: 7.8

3. Answer: AExplanation: Article 14 of theConstitution of India states that the stateshall not deny to any person equality before

law or the equal protection of laws withinthe territory of India.The concept of ‘Equality before law’ is ofBritish origin and it means –(i) The absence of any special privilegesin favour of any person,(ii) The equal subjection of all persons to

the ordinary law of the landadministered by ordinary courts, and

(iii) No person is above the law.The concept of ‘Equal protection of laws’

means –(i) The equality of treatment under

equal circumstances, both inprivileges conferred and liabilitiesimposed by the laws,

(ii) The similar application of the same lawsto all persons who are similarly placed,and

(iii) The like should be treated alikewithout any discrimination.

There can be Equality in the society onlywhen there is absence of privileges to aperson or a section of society that are notenjoyed by the society as a whole.Educational Objective: Equality insociety means, absence of privileges to aperson or a section of the society that arenot enjoyed by the society as a whole.

Source: Indian Polity by Laxmikant(4th Edition) – page no: 7.4

4. Answer: CExplanation: Article 14 – the state shallnot deny to any person equality before lawor the equal protection of laws within theterritory of India. This provision confersrights on all persons whether citizens orforeigners.Article 15 –

a) the state shall not discriminateagainst any citizen on grounds onlyof religion, race, caste, sex or placeof birth.

b) no citizen shall be subjected to anydisability, liability, restriction orcondition on grounds only of religion,

Page 21: PolityPreviousYearQuestionBank · 1979. 8. 31.  · PolityPreviousYearQuestionBank PageNo.5 Company can give licence of trade to individuals and its own employees to trade in India:

Polity Previous Year Question Bank

www.laex.in https://elearn.laex.inPage No. 18

race, caste, sex or place of birth withregard to –

i. access to shops, public restaurants,hotels and places of publicentertainment; or

ii. the use of wells, tanks, bathingghats, road and places of publicresort maintained wholly or partlyby state funds or dedicated togeneral public use.

This provision (b) prohibits discriminationboth by the State and private individuals,while the former provision (a) prohibitsdiscrimination only by the state.Article 16 – provides for equality ofopportunity for all citizens in matters ofemployment or appointment to any officeunder the state. No citizen can bediscriminated against or be ineligible forany employment or office under the stateon grounds only of religion, race, caste, sex,decent, place of birth or residence.Article 17 – abolishes untouchability andforbids its practice in any form. Theenforcement of any disability arising out ofuntouchability shall be an offencepunishable in accordance with law.Educational Objective: Article 14 –Equality before law and Equal Protection oflaws.Article 15 – Prohibition on discriminationbased only on religion, race, caste, sex orplace of birth.Article 16 – Equality of opportunity for allcitizens in matters of public employment.Article 17 – Abolition of untouchability.Source: Indian Polity by Laxmikant (4th

Edition) – page no: 7.4-8Inferred Rights – Article 21

5. Answer: BExplanation: Article 21 of theConstitution of India declares that noperson shall be deprived of his life orpersonal liberty except according toprocedure established by law. This right isavailable to both citizens and non-citizens.The Supreme court in Menaka Gandhi case,1978 took a wider interpretation of the

Article 21 and ruled that the protectionunder Article 21 is available not onlyagainst arbitrary executive action but alsoagainst arbitrary legislation. The Supremecourt has said that the Right to life asembodied in the Article 21 is not merelyconfined to animal existence or survival butit includes within its ambit the Right to livewith human dignity and all those aspects oflife which makes living meaningful,complete and worth living.The Supreme court has expanded the scopeof the Rights conferred under Article 21through its judgements over time. Theyinclude– Right to live with dignity, Right to decent environment, Right to livelihood, Right to privacy, Right to shelter, Right to health, Right to education upto 14 years of age, Right to information, Right to a fair trial, Right to free legal aid, Right of women to be treated with

decency and dignity, etc.Recently, the Supreme court in the Hadiyacase has said that the Right to marry aperson of one’s choice is integral toArticle 21 (right to life and personalliberty) of the Constitution of India.Educational Objective: Right to marry aperson of one’s choice is integral to Article21(right to life and personal liberty).Source: Indian Polity by Laxmikant (4thEdition)

6. Answer: BExplanation: We are of the opinion thatliberty is the ability to do whatever wewant. We also think that laws restrict one’sexercise of liberty. But in reality, the twoare not at odds with each other rather, theycomplement one another. True liberty onlycomes as a result of established law. It isthe law that guarantees freedom andliberty to individuals.Where there are no

Page 22: PolityPreviousYearQuestionBank · 1979. 8. 31.  · PolityPreviousYearQuestionBank PageNo.5 Company can give licence of trade to individuals and its own employees to trade in India:

Polity Previous Year Question Bank

www.laex.in https://elearn.laex.inPage No. 19

laws to protect or provide freedom andliberty, violations of the same take place.The Indian Constitution provides to itscitizen Liberty through Article 21, andArticle 19 guarantees varied types offreedoms including speech and expression,movement, assembly, profession etc. Inmost of the constitutional democracies andmonarchies, freedoms and liberties areguaranteed by the constitution.The Right to liberty given by theConstitution of India under Article 21includes development of individuals to theirfull potential and their protection from theinterference of external agents.Educational Objective: True liberty onlycomes as a result of established law. It isthe law that guarantees freedom andliberty to individuals.Source: Indian Polity by Laxmikant (4thEdition)

7. Answer: CExplanation: Privacy as a fundamentalhuman right is recognized by the UNDeclaration of Human Rights. Privacyunderpins human dignity and other keyvalues such as freedom of association andfreedom of speech. The right to privacy inIndia has developed through a series ofdecisions of the Supreme court. Recently,the Supreme Court in K S Puttaswamycase has reiterated that the Right toPrivacy is protected as an intrinsic part ofthe Right to life and personal liberty underArticle 21 and as a part of the freedomsguaranteed by Part III of the Constitution.Educational Objective: Right to privacyis a Fundamental Right under Article 21(Right to life and personal liberty) of theConstitution of India.Source: Indian Polity by Laxmikant (4thEdition) – page no: 7.11

8. Answer: DExplanation: Liberty comes from theLatin root word “liber” meaningfreedom. It is the absence of constraintsand not merely the absence of restraints.A restraint is the act of holding back by

an external agent whereas Constraintsare developed by one’s own internalstandards and values. Liberty isgenerally perceived as the freedom to doanything he/she pleases however, it canbe classified as positive and negative.

Positive Liberty is defined as thepossession of the power and resources tofulfill one’s own potential. It is the freedomfrom internal constraints.Negative Liberty is defined as freedomfrom interference by other people. It is thefreedom from external restraints.The Right to Liberty provided by theConstitution under Article 21 provides forpositive conception of liberty i.e., to createan enabling atmosphere where anindividual can develop to their fullpotential.Educational Objective: Right to libertyunder Article 21 means the right to developoneself to their full potential.Source: Indian Polity by Laxmikant (4thEdition) – page no: 4.3

9. Answer: CExplanation: The 86th ConstitutionAmendment Act of 2002 has madethe following changes to theConstitution of India -i. Article 21Ais added to the

Constitution of India which confersthe Right to free and compulsoryeducation to the children in the agegroup of 6-14 years.

ii. Amended the DirectivePrinciple under Article 45.According to it, the state shall striveto provide early childhood care andeducation to children below the ageof 6 years.

iii. Added a new FundamentalDuty under Article 51A(k).According to it, it shall be the dutyof a parent or guardian to provideopportunities for education to theirchild between the age of 6 and 14years.

The 42nd Constitution Amendment Actof 1976 has transferred 5 subjects from

Page 23: PolityPreviousYearQuestionBank · 1979. 8. 31.  · PolityPreviousYearQuestionBank PageNo.5 Company can give licence of trade to individuals and its own employees to trade in India:

Polity Previous Year Question Bank

www.laex.in https://elearn.laex.inPage No. 20

state list to concurrent list. They areEducation, Forests, Weights &Measures, Protection of Wild Animalsand Birds, and Administration ofJusticeSarva Shiksha Abhiyan (SSA) isGovernment of India's flagshipprogramme for achievement ofUniversalization of ElementaryEducation (UEE) in a time boundmanner, as mandated by 86thamendment to the Constitution ofIndia.SSA is being implemented inpartnership with State Governments tocover the entire country. This schemealso seeks to attain 100% retention ratein elementary schools. The programme seeks to open new

schools in those habitations whichdo not have schooling facilities andstrengthen existing schoolinfrastructure through provision ofadditional class rooms, toilets,drinking water, etc.

Existing schools with inadequateteacher strength are provided withadditional teachers, while thecapacity of existing teachers is alsobeing strengthened.

SSA seeks to provide life skillsalong with elementary education.

SSA has a special focus on girl'seducation and children with specialneeds.

SSA also seeks to provide computereducation to bridge the digitaldivide.

Educational Objective: The 86thConstitution Amendment Act of 2002 –i. Article 21A confers the Right

to free and compulsory education tothe children in the age group of 6-14years.

ii. Article 51A(k) - it shall be theduty of a parent or guardian toprovide opportunities for educationto their child between the age of 6and 14 years.

iii. Article 45 - the state shallstrive to provide early childhood

care and education to childrenbelow the age of 6 years.

Source: Indian Polity by Laxmikant(4th Edition) – page no: 7.12Right against Exploitation – Art 23, 24

10. Answer: CExplanation: The Right againstExploitation is covered under Articles 23and 24 of the constitution of India.Article 23 – Prohibition of traffic in humanbeings, beggar, and other similar forms offorced labour. Any contravention of thisprovision shall be punishable in accordancewith law. In order to give teeth to theseprovisions, the Parliament has enacted thefollowing laws

Bonded Labour System (Abolition)Act, 1976,

the Minimum Wages Act, 1948, the Contract Labour Act,1970, and the Equal Remuneration Act, 1976.However, the Constitution provides anexception to these provisions. It permitsthe State to impose compulsory servicefor public purposes, for which it is notbound to pay. In imposing such aservice, the State shall not make anydiscrimination on grounds only ofreligion, race, caste or class.Article 24 – Prohibition of theemployment of children below the age of14 years in any factory, mine or otherhazardous activities. But it does notprohibit their employment in anyharmless or innocent work. TheParliament has enacted the followinglaws to give teeth to these provisions – The Child Labour (Prohibition and

Regulation) Act, 1986, The Factories Act, 1948, The Mines Act, 1952, The Plantation Labour Act, 1951,

etc.The Child Labour (Prohibition andRegulation) Amendment Act, 2016completely prohibits the employment ofchildren below 14 years. It also

Page 24: PolityPreviousYearQuestionBank · 1979. 8. 31.  · PolityPreviousYearQuestionBank PageNo.5 Company can give licence of trade to individuals and its own employees to trade in India:

Polity Previous Year Question Bank

www.laex.in https://elearn.laex.inPage No. 21

prohibits the employment of adolescentsin the age group of 14-18 years inhazardous occupations. The act hasmade the violation of the aboveprovisions as a cognizable offence.

Educational Objective: Article 23 –Prohibition of traffic in human beings,beggar, and other similar forms of forcedlabour.Article 24 - Prohibition of the employmentof children below the age of 14 years in anyfactory, mine or other hazardous activities.Source: Indian Polity by Laxmikant (4thEdition) – page no: 7.14

11. Answer: AExplanation: Article 24 – Prohibitionof the employment of children below theage of 14 years in any factory, mine orother hazardous activities. But it doesnot prohibit their employment in anyharmless or innocent work. TheParliament has enacted the followinglaws to give teeth to these provisions – The Child Labour (Prohibition and

Regulation) Act, 1986, The Factories Act, 1948, The Mines Act, 1952, The Plantation Labour Act, 1951,

etc.The Commissions for Protection ofChild Rights Act, 2005 was enacted toprovide for the establishment of aNational Commission and StateCommissions for protection of childrights. It also provides forestablishment of Children’s courts forspeedy trial of offences against childrenor for violation of child rights.The Child Labour (Prohibition andRegulation) Amendment Act, 2016completely prohibits the employment ofchildren below 14 years. It alsoprohibits the employment of adolescentsin the age group of 14-18 years inhazardous occupations. The act hasmade the violation of the aboveprovisions as a cognizable offence.Educational Objective: Article 24 -Prohibition of the employment of

children below the age of 14 years inany factory, mine or other hazardousactivities.Source: Indian Polity by Laxmikant(4th Edition) – page no: 7.14Cultural and Educational Rights – Art29 and 30

12. Answer: CExplanation: The Cultural andEducational Rights are protectedunder Article 29 and Article 30 of theConstitution of India.Article 29 – deals with the protectionof interests of minorities. It says that –(1) Any section of the citizens residing

in the territory of India or any partthereof having a distinct language,script or culture of its own shallhave the right to conserve the same.

(2) No citizen shall be denied admissioninto any educational institutionmaintained by the state or receivingaid out of State funds on groundsonly of religion, race, caste,language.Article 30 – deals with Right ofminorities to establish andadminister educational institutions.It says that –All minorities, whether based onreligion or language, shall have theright to establish and administereducational institutions of theirchoice.

(1) The State shall not, in granting aidto educational institutions,discriminate against anyeducational institution on theground that it is under themanagement of a minority, whetherbased on religion or language.

The Prime Minister's15-pointProgramme for minorities is aprogramme launched by Indiangovernment in 2006 for welfare of religiousminorities.Reports by committees such asthe Sachar Committee have highlightedthat minorities, especially Muslims, in thecountry were often in a worse socio-

Page 25: PolityPreviousYearQuestionBank · 1979. 8. 31.  · PolityPreviousYearQuestionBank PageNo.5 Company can give licence of trade to individuals and its own employees to trade in India:

Polity Previous Year Question Bank

www.laex.in https://elearn.laex.inPage No. 22

economic and political condition than theScheduled Castes and Scheduled tribes.The 15-point program lays down theguidelines to specifically target minoritiesin schemes that are already in place and indesigning and executing new schemesaimed at the empowerment of theminorities. Therefore, if a religioussect/community is given the status of anational minority, it can derive benefitsunder the Prime Minister's 15-PointProgramme.The President can nominate two membersbelonging to Anglo–Indian community onlyto the Lok Sabha if he is of the opinion thatthey are not adequately represented. Thereis no provision in the Constitution for thePresident to nominate a member belongingto a minority community to the Lok Sabha.Educational Objective: Article 29 -Protection of language, script and culture ofminorities.Article 30 - Right of minorities to establishand administer educational institutions.Source: Indian Polity by Laxmikant (4thEdition) – page no: 7.16 and 17.8

13. Answer: AExplanation: Article 16 – provides forequality of opportunity for all citizens inmatters of employment or appointment toany office under the state. No citizen can bediscriminated against or be ineligible forany employment or office under the stateon grounds only of religion, race, caste, sex,decent, place of birth or residence.Article 29 – deals with the protection ofinterests of minorities. It says that –

(1) Any section of the citizens residingin the territory of India or any partthereof having a distinct language,script or culture of its own shallhave the right to conserve the same.

(2) No citizen shall be denied admissioninto any educational institutionmaintained by the state or receivingaid out of State funds on groundsonly of religion, race, caste,language.Article 30 – deals with Right ofminorities to establish and

administer educational institutions.It says that –

(1) All minorities, whether based onreligion or language, shall have theright to establish and administereducational institutions of theirchoice.

(2) The State shall not, in granting aidto educational institutions,discriminate against anyeducational institution on theground that it is under themanagement of a minority, whetherbased on religion or language.

Article 31 – deals with the compulsoryacquisition of property by the state. Itguaranteed to every person, whethercitizen or alien the Right againstdeprivation of his property. It statesthat no person shall be deprived of hisproperty except by authority of law. Italso places certain restrictions on theState in acquisition of property –i. It should be for public

purpose, andii. It should provide

compensation to the owner.This article was repealed by the 44thConstitution Amendment of 1978.

Educational Objective: Article 16 –Equality of opportunity for all citizens inmatters of public employment.Article 29 - Protection of language, scriptand culture of minorities.Article 30 - Right of minorities to establishand administer educational institutions.Article 31 (repealed) – Compulsoryacquisition of property by the State.Source: Indian Polity by Laxmikant (4thEdition) – page no: 7.6, 7.16, and 7.2

14. Answer: DExplanation: A mere declaration offundamental rights in the Constitution ismeaningless without providing an effectivemachinery for their enforcement, if andwhen they are violated. Hence, Article 32confers the Right to remedies for theenforcement of the fundamental rights ofan aggrieved citizen. In other words, the

Page 26: PolityPreviousYearQuestionBank · 1979. 8. 31.  · PolityPreviousYearQuestionBank PageNo.5 Company can give licence of trade to individuals and its own employees to trade in India:

Polity Previous Year Question Bank

www.laex.in https://elearn.laex.inPage No. 23

right to get the Fundamental Rightsprotected is in itself a fundamental right.That is why Dr B R Ambedkar calledArticle 32 as the most importantarticle of the Constitution – an articlewithout which the Constitution would be anullity. The Supreme court acts as theguarantor and protector of theFundamental Rights. The Supreme courthas said Article 32 is a basic feature of theConstitution.Article 32 – Remedies for enforcement ofrights conferred are -1. The right to move the Supreme court for

the enforcement of the FundamentalRights.

2. The Supreme court shall have thepower to issue directions or orders orwrits, including writs of habeaus corpus,mandamus, prohibition, quo warrantoand certiorari for the enforcement of theFundamental Rights.

3. The Parliament may by law empowerany other court to exercise the samepowers of the Supreme court in theenforcement of the Fundamental Rights.

4. The rights guaranteed by this articleshall not be suspended except asotherwise provided by the Constitution.

Educational Objective: Article 32 of theConstitution of India – Fundamental Rightof Right to remedies.The right to move the Supreme court in thefirst instance itself for enforcement ofFundamental Rights.Source: Indian Polity by Laxmikant (4thEdition) – page no: 7.17Right to Equality: Article 14-18

15. Answer: CExplanation: Article 325 of theConstitution of India says that ‘No personshall be ineligible for inclusion in electoralroll on grounds only of religion, race, casteor sex’.Article 326 of the Constitution of Indiasays that elections to the House of thepeople and to the Legislative Assemblies ofstates to be on the basis of Adult Suffrage.A person shall be eligible to be a voter if he

is not otherwise disqualified under thisConstitution or any law made by theParliament. A person is disqualifiedfrombeing a voter only on the grounds of non-residence, unsoundness of mind, crime orcorrupt or illegal practice.The voting agehas been reduced from 21 years to 18 yearsby the 61st Constitutional Amendment Actof 1988.The Right to Vote originates from theConstitution itself (Article 326) and isregulated by the Representation of PeopleAct, 1951. The Right to Vote is notaFundamental Right but a ConstitutionalRight because it is not covered under PartIII of the Constitution.The Constitution of India has only placedcertain minimum qualifications in order tobe eligible to contest the elections to theposts of the President, Vice-President,Member of Parliament and Member ofState Legislature. Any person having thesequalifications and not disqualified underthe Representation of People Act, 1951 hasthe Right to contest an election.Therefore, the Right to Contest and to beelected is a Constitutional Right.Educational Objective: The Right to Voteand the Right to Contest and to be electedare onstitutional Rights.Article 326 – elections to the Lok Sabhaand Legislative Assemblies based on AdultSuffrage.Source: Indian Polity by Laxmikant (4thEdition) – page no: not available

16. Explanation: The following are theFundamental Rights that are availableonly to the citizens and not available toforeigners – Article 15 - Prohibition of

discrimination on grounds ofreligion, race, caste, sex or place ofbirth.

Article 16 - Equality of opportunityin matters of public employment.

Article 19 - Six basic freedomsregarding freedom of speech,assembly, formation of associationsor unions, movement, residence andoccupation.

Page 27: PolityPreviousYearQuestionBank · 1979. 8. 31.  · PolityPreviousYearQuestionBank PageNo.5 Company can give licence of trade to individuals and its own employees to trade in India:

Polity Previous Year Question Bank

www.laex.in https://elearn.laex.inPage No. 24

Article 29 - Protection of language,script and culture of minorities.

Article30 - Right of minorities toestablish and administereducational institutions.

The rest of the Fundamental Rightscovered under Articles 14, 20, 21, 21A,22, 23, 24, 25, 26, 27 and 28 areavailable to both citizens as well asforeigners.Educational Objective: TheFundamental Rights covered by Articles15, 16, 19, 29 and 30 are available onlyto the citizens and not to the foreigners.Source: Indian Polity by Laxmikant(4th Edition) – page no: 7.4, 7.6, 7.9 and7.16.

17. Answer: AExplanation: Originally, the Right toProperty was one of the FundamentalRights under part – III of theConstitution. Article 19(1)(f) and Article31 dealt with the Right to Property.

Article 19(1)(f) – guaranteed to everycitizen the right to acquire, hold anddispose of property.Article 31 – guaranteed to every person,whether citizen or alien the Right againstdeprivation of his property. It states that noperson shall be deprived of his propertyexcept by authority of law. It also placescertain restrictions on the State inacquisition of property–i. it should be for public

purpose, andii. it should provide

compensation to the owner.Since the commencement of theConstitution, the Fundamental Right toProperty has been the most controversial.It has caused many confrontations betweenthe Supreme court and the Parliament. Ithas led to a number of ConstitutionalAmendments – 1st, 4th, 7th, 25th, 39th, 40thand 42nd amendments. Through theseamendments, Articles 31A, 31B and 31Cwere added and modified from time to timeto nullify the effect of the Supreme courtjudgements and to protect laws from being

challenged on the grounds of contraventionof Fundamental Rights.Therefore, the 44th ConstitutionAmendment Act of 1978 abolished theRight to Property as a Fundamental Rightby repealing Article 19(1)(f) and Article 31from Part III of the Constitution. Instead,Right to Property is covered by a newArticle 300A under Part XII of theConstitution. Thus, the Right to Propertyremains as a Legal Right but not as aFundamental Right.Educational Objective: 44thConstitutional Amendment Act of 1978 hasremoved the Right to Property fromFundamental Rights. At present Right toProperty is covered under Article 300A ofthe Constitution of India and it is a LegalRight.Source: Indian Polity by Laxmikant (4thEdition) – page no: 7.21.

04. Directive Principles of State Policy

4.1 Common features of DPSP

1. Consider the followingstatements:With reference to theConstitution of India, theDirective Principles of StatePolicy constitute limitationsupon

1. Legislative function2. Executive function

Which of the above statements is/arecorrect?

a) 1 onlyb) 2 onlyc) Both 1 and 2d) Neither 1 Nor 2

2. The ideal of Welfare State' in theIndian Constitution is enshrinedin its

a) Preambleb) Directive Principles of State Policy

Page 28: PolityPreviousYearQuestionBank · 1979. 8. 31.  · PolityPreviousYearQuestionBank PageNo.5 Company can give licence of trade to individuals and its own employees to trade in India:

Polity Previous Year Question Bank

www.laex.in https://elearn.laex.inPage No. 25

c) Fundamental Rightsd) Seventh Schedule

3. Consider the followingstatements regarding theDirective Principles of StatePolicy:

1. The Principles spell out the socio-economic democracy in the country.

2. The provisions contained in thesePrinciples are not enforceable by anycourt.Which of the statements givenabove is / are correct?

(a) 1 only(b) 2 only(c) Both 1 and 2(d) Neither 1 Nor 2

4. ‘Economic Justice’ as one of theobjectives of the IndianConstitutional has been providedin

a) The Preamble and the FundamentalRights.

b) The Preamble and the DirectivePrinciples of State Policy.

c) The Fundamental Rights and theDirective Rights and the DirectivePrinciples of State Policy.

d) None of the above.

5. According to the constitution onIndia, which of the following arefundamental for the governanceof the country?

a) Fundamental rightsb) Fundamental dutiesc) Directive principles of state policyd) Fundamental Rights and

Fundamental Duties

6. The purpose of the inclusion ofDirective Principles of the StatePolicy in the Indian Constitutionis to establish:

a) political democracyb) social democracyc) Gandhian democracyd) social and economic democracy

4.2 DPSP Classification

7. Which of the following is/areincluded in the DirectivePrinciples of the State Policy?

1. Prohibition of traffic in human beingsand forced labour.

2. Prohibition of consumption except formedicinal purposes of intoxicatingdrinks and of other drugs which areinjurious to health.

Select the correct answer using thecode given below:

a) 1 onlyb) 2 onlyc) Both 1 and 2d) Neither 1 Nor 2

8. Which principle among thefollowing was added to theDirective Principles of StatePolicy by the 42nd Amendment tothe Constitution?

a) Equal pay for equal work for bothmen and women

b) Participation of workers in themanagement of industries

c) Right to work, education and publicassistance

d) Securing living wage and humanconditions of work to workers

9. In the Constitution of India,promotion of international peaceand security is included in the

a) Preamble to the Constitutionb) Directive Principles of State Policyc) Fundamental Dutiesd) Ninth Schedule

Page 29: PolityPreviousYearQuestionBank · 1979. 8. 31.  · PolityPreviousYearQuestionBank PageNo.5 Company can give licence of trade to individuals and its own employees to trade in India:

Polity Previous Year Question Bank

www.laex.in https://elearn.laex.inPage No. 26

10. Consider the followingprovisions under the DirectivePrinciples of State Policy asenshrined in the Constitution ofIndia:

1. Securing for citizens of India auniform civil code

2. Organizing village Panchayats3. Promoting cottage industries in rural

areas4. Securing for all the workers

reasonable leisure and culturalopportunities

Which of the above are the GandhianPrinciples that are reflected in theDirective Principles of State Policy?a) 1, 2 and 4 onlyb) 2 and 3 onlyc) 1, 3 and 4 onlyd) 1, 2, 3 and 4

11. Consider the followingstatements:

1. There is no provision in theConstitution of India to encourageequal pay for equal work for bothmen and women.

2. The Constitution of India does notdefine backward classes.

Which of the statement(s) given aboveis/are correct?a) 1 onlyb) 2 onlyc) Both 1 and 2d) Neither 1 Nor 2

12. Which one of the followingarticles of the DirectivePrinciples of State Policy dealswith the promotion ofinternational peace and security?

a) Article 51b) Article 48Ac) Article 43A

d) Article 41

4.3 DPSP – Miscellaneous13. With reference to constitution of

India, consider the following:1. Fundamental Rights2. Fundamental duties3. Directive Principles of State Policy

Which of the above provisions of theconstitution of India is/are fulfilled byNational Social Assistance programmelaunched by the Government of India?a) 1 onlyb) 3 onlyc) 1 and 3 onlyd) 1, 2 and 3

Key & Explanation1. Answer: DExplanation: The Directive Principlesof State Policy (DPSP) are enumeratedin part IV of the Constitution fromArticles 36 to 51. They are theconstitutional instructions to the State inlegislative, executive and administrativematters. The DPSP strive to establish aneconomic and social democracy in thecountry. The DPSP along with theFundamental Rights form the soul of theConstitution.Article 37 of the Constitution of India saysthat the DPSP are non-justiciable in nature.However, it also makes it clear that theDPSP are fundamental to the governance ofthe country and it shall be the duty of thestate to apply these principles in makinglaws. They impose a moral obligation onthe state for their application, but the realforce behind is political i.e. public opinion.The makers of the Constitution believedmore in an awakened public opinion ratherthan in courts as the ultimate sanction forthe fulfilment of these principles.Therefore, the DPSP doesn’t impose anylimitation directly on the state but agovernment which rests on popular votecan hardly ignore them while shaping its

Page 30: PolityPreviousYearQuestionBank · 1979. 8. 31.  · PolityPreviousYearQuestionBank PageNo.5 Company can give licence of trade to individuals and its own employees to trade in India:

Polity Previous Year Question Bank

www.laex.in https://elearn.laex.inPage No. 27

policy. The DPSP also help the courts inexamining and determining theconstitutional validity of a law.Educational Objective: The DPSPdoesn’t impose any limitation directly onthe state but a government which rests onpopular vote can hardly ignore them whileshaping its policy.Source: Indian Polity by Laxmikant (4thEdition) – page no: 8.1-3

2. Answer: BExplanation: The welfare state is aform of government in which the stateprotects and promotes the economic andsocial well-being of the citizens, basedupon the principles of equal opportunity,equitable distribution of wealth, andpublic responsibility for citizens unableto avail themselves of the minimalprovisions for a good life.The DPSP constitutes a verycomprehensive economic, social andpolitical programme for a moderndemocratic State which is aimed atrealising the high ideals of justice,liberty, equality and fraternity asoutlined in the Preamble to theConstitution. They embody the conceptof a ‘welfare state’. The followingprovisions in the Constitution of Indiastrives to establish a welfare state.

Article 38 – to promote the welfare ofthe people by securing a social orderpermeated by justice (social, economicand political) and to minimizeinequalities in income, status, facilitiesand opportunities.

Article 39 – to securei. The right adequate means of

livelihood for all citizens.ii. Equitable distribution of

material resources of thecommunity for common good.

iii. Prevention of concentrationof wealth and means of production.

iv. Equal pay for equal work formen and women.

v. Preservation of the healthand strength of workers andchildren against forcible abuse.

vi. Opportunities for healthydevelopment of children.

Article 39A – to promote equaljustice and to provide free legal aidto the poor.

Article 41 – to secure the right towork, to education and to publicassistance in cases ofunemployment, old age, sicknessand disablement.

Article 42 – to make provision forjust and humane conditions forwork and maternity relief.

Article 43 – to secure a living wage,a decent standard of life andcultural opportunities for allworkers.

Article 43A – to secure theparticipation of workers in themanagement of industries.

Article 45 – to provide earlychildhood care and education for allchildren until they complete the ageof 6 years.

Article 46 – to promote theeducational and economic interestsof SCs, STs and other weakersections of the society and to protectthem from social injustice andexploitation.

Article 47 – to raise the level ofnutrition and the standard of livingof people and to improve publichealth.

Educational Objective: The DPSPcontain the provisions for theestablishment of a welfare state inIndia.Source: Indian Polity by Laxmikant(4th Edition) – page no: 8.2

3. Answer: CExplanation: The DirectivePrinciples of State Policy (DPSP)are enumerated in part IV of theConstitution from Articles 36 to 51.

Page 31: PolityPreviousYearQuestionBank · 1979. 8. 31.  · PolityPreviousYearQuestionBank PageNo.5 Company can give licence of trade to individuals and its own employees to trade in India:

Polity Previous Year Question Bank

www.laex.in https://elearn.laex.inPage No. 28

They are the constitutional instructionsto the State in legislative, executive andadministrative matters. The DPSPstrive to establish an economic andsocial democracy in the countrywhereas, the political democracy isestablished by the Fundamental Rights.The DPSP along with the FundamentalRights form the soul of the Constitution.Article 37 of the Constitution of Indiasays that the DPSP are non-justiciable in nature. However, it alsomakes it clear that the DPSP arefundamental to the governance of thecountry and it shall be the duty of thestate to apply these principles inmaking laws. They impose a moralobligation on the state authorities fortheir application, but the real forcebehind is political i.e. public opinion.The makers of the Constitution believedmore in an awakened public opinionrather than in courts as the ultimatesanction for the fulfilment of theseprinciples.Educational Objective: Article 37 ofthe Constitution of India says that theDPSP are non-justiciable in nature.Source: Indian Polity by Laxmikant(4th Edition) – page no: 8.1

4. Answer: BExplanation: The Preamble states that‘Justice’ shall include social, economic andpolitical justice.

Social justice denotes equaltreatment of all citizens without anydiscrimination. It means absence ofprivileges being extended to anyparticular section of the society andimprovement in the conditions ofbackward classes and women.

Economic justice denotes non-discrimination of people on the basisof economic factors. It involveselimination of glaring inequalitiesin wealth, income and property.

Political justice denotes that allcitizens should have equal politicalrights, equal access to all political

offices and equal voice in thegovernment.

The Directive Principles of State Policystrive to achieve the ideals of social andeconomic justice and the ideal of politicaljustice is established by the FundamentalRights.The following directive principles in theConstitution of India strive to achieve theideal of economic justice -

Article 38 – to promote the welfareof the people by securing a socialorder permeated by justice (social,economic and political) and tominimize inequalities in income,status, facilities and opportunities.

Article 39 – to securei. the right to adequate means of

livelihood for all citizens.ii. equitable distribution of material

resources of the community forcommon good.

iii. prevention of concentration ofwealth and means of production.

iv. equal pay for equal work for menand women.

v. preservation of the health andstrength of workers and childrenagainst forcible abuse.

vi. opportunities for healthydevelopment of children.

Article 41 – to secure the right towork, to education and to publicassistance in cases ofunemployment, old age, sicknessand disablement.

Article 42 – to make provision forjust and humane conditions forwork and maternity relief.

Article 43 – to secure a living wage,a decent standard of life andcultural opportunities for allworkers.

Article 46 – to promote theeducational and economic interestsof SCs, STs and other weakersections of the society and to protectthem from social injustice andexploitation.

Page 32: PolityPreviousYearQuestionBank · 1979. 8. 31.  · PolityPreviousYearQuestionBank PageNo.5 Company can give licence of trade to individuals and its own employees to trade in India:

Polity Previous Year Question Bank

www.laex.in https://elearn.laex.inPage No. 29

Educational Objective: ‘Economic andSocial Justice’ as the objectives of theIndian Constitution have been providedinthe Preamble and the DirectivePrinciples of State Policy.Source: Indian Polity by Laxmikant (4thEdition) – page no: 4.3 and 8.2

5. Answer: CExplanation: Article 37 of theConstitution of India says that theDirective Principles of State Policy are non-justiciable in nature. However, it alsomakes it clear that the DPSP arefundamental to the governance of thecountry and it shall be the duty of thestate to apply these principles in makinglaws. They impose a moral obligation onthe state authorities for their application,but the real force behind is political i.e.public opinion. The makers of theConstitution believed more in an awakenedpublic opinion rather than in courts as theultimate sanction for the fulfilment of theseprinciples.Educational Objective: Article 37 of theConstitution of India states that the DPSPare fundamental to the governance of thecountry and it shall be the duty of the stateto apply these principles in making laws.Source: Indian Polity by Laxmikant (4thEdition) – page no: 8.1

6. Answer: DExplanation: The Directive Principles ofState Policy (DPSP) contains the idealsthat the state should keep in mind whileformulating policies and enacting laws.They aim at realising the high ideals ofjustice (social, economic and political),liberty, equality and fraternity as outlinedin the Preamble. The DPSP strives toestablish social and economic democracy inthe country and the political democracy isestablished by the Fundamental Rights.Educational Objective: The DPSP strivesto establish social and economic democracyin the country.Source: Indian Polity by Laxmikant (4thEdition) – page no: 8.1

7. Answer: BExplanation: The Right againstExploitation given by the Constitutionunder Fundamental Rights includes theProhibition of traffic in human beings,beggar (forced labour) and other formsof forced labour (Article 23 of theConstitution of India).

Some of the ideas fromthe Gandhianprogramme for the reconstruction of Indiaenunciated during the freedom movementare included in the Directive Principles ofState Policy. They are –

Article 40 – to organize villagepanchayats and to enable them tofunction as units of self-government.

Article 43 – to promote cottageindustries in the rural areas.

Article 43B – to promote voluntaryformation and democratic control ofcooperative societies.

Article 46 – to promote theeducational and economic interestsof SCs, STs and other weakersections of the society and to protectthem from social injustice andexploitation.

Article 47 – to prohibitconsumption of intoxicatingdrinks and drugs which areinjurious to health.

Article 48 – to prohibit theslaughter of cows, calves and othermilch and draught cattle and toimprove their breeds.

Educational Objective: Article 23 of theConstitution of India - Prohibition of trafficin human beings, beggar (forced labour)and other forms of forced labour.Article 47of the Constitution of India – toprohibit consumption of intoxicating drinksand drugs which are injurious to health.Source: Indian Polity by Laxmikant (4thEdition) – page no: 7.14 and 8.2

8. Answer: BExplanation: The 42ndConstitutionalAmendment Act of 1976 has added the

Page 33: PolityPreviousYearQuestionBank · 1979. 8. 31.  · PolityPreviousYearQuestionBank PageNo.5 Company can give licence of trade to individuals and its own employees to trade in India:

Polity Previous Year Question Bank

www.laex.in https://elearn.laex.inPage No. 30

following provisions to the DirectivePrinciples of State Policy – Article 39(f) – the state shall strive

to promote opportunities for healthydevelopment of children.

Article 39A – to promote equaljustice and to provide free legal aidto the poor.

Article 43A – to secure theparticipation of workers in themanagement of industries.

Educational Objective: Articles 39(f),Article 39A and Article 43A were added tothe DPSP by the 42ndConstitutionalAmendment Act of 1976.Source: Indian Polity by Laxmikant (4thEdition) – page no: 8.2

9. Answer: BExplanation: Article 51 of theConstitution of India states that thestate shall strive to - promote international peace and

security and to maintain just andhonourable relations betweennations.

foster respect for international lawand treaty obligations.

encourage settlement ofinternational disputes byarbitration.

Educational Objective: Article 51 ofthe Constitution of Indiadeals with thepromotion of international peace andsecurity.Source: Indian Polity by Laxmikant(4th Edition) – page no: 8.3

10. Answer: BExplanation: The Directive Principlesof State Policy can be classified intothree broad categories –i. Socialistic,ii. Gandhian, andiii. Liberal-intellectual

The following are the Gandhianprinciples included in the DirectivePrinciples of State Policy– Article 40 – to organize village

panchayats and to enable them tofunction as units of self-government.

Article 43 – to promote cottageindustries in the rural areas.

Article 43B – to promote voluntaryformation and democratic control ofcooperative societies.

Article 46 – to promote theeducational and economic interestsof SCs, STs and other weakersections of the society and to protectthem from social injustice andexploitation.

Article 47 – to prohibitconsumption of intoxicating drinksand drugs which are injurious tohealth.

Article 48 – to prohibit theslaughter of cows, calves and othermilch and draught cattle and toimprove their breeds.

Article 43 – the state shall endeavour tosecure a living wage, a decent standard oflife and full enjoyment of leisure andsocial and cultural opportunities.This directive principle is classified as asocialistic principle.Article 44 – to secure for all citizens auniform civil code throughout thecountry.This directive principle is classified as aliberal-intellectual principle.Educational Objective: Articles 40, 43,43B, 46, 47, and 48 of the Constitution ofIndia contains the directive principlesformulated on the basis of Gandhianideology.Source: Indian Polity by Laxmikant (4thEdition) – page no: 8.2

11. Answer: BExplanation: Article 39(d) of theConstitution of India directs the state tosecure equal pay for equal work for menand women.

Page 34: PolityPreviousYearQuestionBank · 1979. 8. 31.  · PolityPreviousYearQuestionBank PageNo.5 Company can give licence of trade to individuals and its own employees to trade in India:

Polity Previous Year Question Bank

www.laex.in https://elearn.laex.inPage No. 31

The constitution of India does notdefine the term backward classes. Theexpression ‘Backward Classes’ meanssuch backward classes of citizens otherthan SCs and STs as may be specifiedby the Central and the StateGovernments. Similarly, theConstitution also doesn’t specify thecastes and tribes that constitutes SCsand STs. It gives the President thepower to specify the castes and tribes ineach state and union territory that areclassified as SCs and STs.Educational Objective: Article 39(d)of the constitution of India provides forequal pay for equal work to both menand women.Source: Indian Polity by Laxmikant(4th Edition) – page no: 8.2 and 63.1

12. Answer: AExplanation: Article 51 of theConstitution of India states that thestate shall strive to - Promote international peace and

security and to maintain just andhonourable relations betweennations.

Foster respect for international lawand treaty obligations.

Encourage settlement ofinternational disputes byarbitration.

Educational Objective: Article 51 ofthe Constitution of India deals with thepromotion of international peace andsecurity.Source: Indian Polity by Laxmikant(4th Edition) – page no: 8.3

13. Answer: CExplanation: The National SocialAssistance Programme (NSAP) is awelfare programme administered by theMinistry of Rural Development. Thisprogramme is being implemented inrural areas well as urban areas. NSAPrepresents a significant step towardsthe fulfilment of the DirectivePrinciples of State Policy enshrined in

the Constitution of India which enjoinupon the State to undertake a numberof welfare measures. In particular,Article 41 of the Constitution of Indiadirects the State to provide publicassistance to its citizens in case ofunemployment, old age, sickness anddisablement. The programme was firstlaunched on 15th August 1995 as aCentrally Sponsored Scheme with theobjective of providing financial supportto the poor.It currently covers more than 3 crorepeople (including 80 lakh widows, 10lakh disabled and 2.2 crore elderly) whoare below the poverty line (BPL).Presently NSAP comprises of fiveschemes, namely:i. Indira Gandhi National Old Age

Pension Scheme (IGNOAPS).ii. Indira Gandhi National Widow

Pension Scheme (IGNWPS).iii. Indira Gandhi National

Disability Pension Scheme(IGNDPS).

iv. National Family Benefit SchemeNFBS).

v. AnnapurnaEducational Objective: Article 41 ofthe Constitution of India – to secure theright to work, to education and to publicassistance in cases of unemployment,old age, sickness and disablement.The National Social AssistanceProgramme (NSAP) is a CentrallySponsored Scheme under the Ministryof Rural DevelopmentSource: Indian Polity by Laxmikant(4th Edition) – page no: 8.2

05. Fundamental Duties

5.1 Common features ofFundamental Duties

1. Which of the followingstatements is/are true of theFundamental Duties of an Indiancitizen?

Page 35: PolityPreviousYearQuestionBank · 1979. 8. 31.  · PolityPreviousYearQuestionBank PageNo.5 Company can give licence of trade to individuals and its own employees to trade in India:

Polity Previous Year Question Bank

www.laex.in https://elearn.laex.inPage No. 32

1. A legislative process has beenprovided to enforce these duties.

2. They are correlative to legal duties.Select the correct answer using thecode given below:a) 1 onlyb) 2 onlyc) Both 1 and 2d) Neither 1 nor 2

2. In the context of India, which oneof the following is the correctrelationship between Rights andDuties?

a) Rights are correlative with Duties.b) Rights are personal and hence

independent of society and Duties.c) Rights, not Duties, are important for

the advancement of the personality ofthe citizen.

d) Duties, not Rights, are important forthe stability of the State.

5.2 List of Fundamental Duties

3. To uphold and protect thesovereignty, unity and integrityof India, a provision has beenmade in the?

a) Directive Principles of State Policyb) Preamble to the Constitutionc) Fundamental Dutiesd) Fundamental Rights

4. Which of the following is/areamong the Fundamental Dutiesof citizens laid down in theIndian Constitution?

1. To preserve the rich heritage of ourcomposite culture.

2. To protect the weaker sections fromsocial injustice.

3. To develop the scientific temper andspirit of inquiry.

4. To strive towards excellence in allspheres of individual and collectiveactivity.

Select the correct answer using thecodes given below:

a) 1 and 2 onlyb) 2 onlyc) 1, 3 and 4 onlyd) 1, 2, 3 and 4

5. Under the constitution of India,which one of the following is nota fundamental duty?

a) To vote in public elections.b) To develop scientific temper.c) To safeguard public property.d) To abide by the constitution and

respect its ideals.Key & Explanation

1. Answer: DExplanation: Like the DPSP, theFundamental Duties are also non-justiciable in nature. The Constitution doesnot provide for their direct enforcement bythe courts. Moreover, there is no legalsanction against their violation. However,the Parliament is free to enforce them by asuitable legislation but until now, no suchlaw has been passed by the Parliament.Fundamental duties do not correlate withlegal duties because a law was not enactedto enforce them.Educational Objective: TheFundamental Duties are non-justiciable innature. However, the Parliament is free toenforce them by a suitable legislation butuntil now, no such law has been passed bythe Parliament.Source: Indian Polity by Laxmikant (4thEdition) – page no: 9.22. Answer: AExplanation: Rights are correlative withduties and cannot be separated from oneanother. Both go side by side. If the stategives the right to life to a citizen, it alsoimposes an obligation on him to not to

Page 36: PolityPreviousYearQuestionBank · 1979. 8. 31.  · PolityPreviousYearQuestionBank PageNo.5 Company can give licence of trade to individuals and its own employees to trade in India:

Polity Previous Year Question Bank

www.laex.in https://elearn.laex.inPage No. 33

expose his life to dangers, as well as torespect the life of others.

Rights can be enjoyed only in theworld of duties. For every rightthere is corresponding duty. Whenthe people fail to discharge theirduties properly, the rights becomemeaningless.

Rights are not the monopoly of asingle individual. Everybody getsthese equally and it is their duty tosee that others also enjoy theirrights.

Rights originate in society.Therefore, while enjoying rights, wemust always try to promote thewelfare of the society as a whole.

Since state protects and enforcesrights, it also becomes the duty ofall citizens to be loyal to the state. Itis their duty to obey the laws of thestate and to pay taxes honestly.Thus, a citizen has both Rights andDuties.

Educational Objective: Rights arecorrelative with duties. For every rightthere is corresponding duty. When thepeople fail to discharge their dutiesproperly, the rights become meaningless.Source: Indian Polity by Laxmikant (4thEdition) – page no: 9.1

3. Answer: CExplanation: The 42nd ConstitutionAmendment Act of 1976, has added anew part-IVA to the Constitution of India.Article 51A under Part-IVA of theConstitution deals with the Fundamentalduties. Fundamental Duties wereincorporated in the Constitution based onthe recommendations of the Swaran SinghCommittee (1976) however, all of itsrecommendations were not incorporated.The Fundamental Duties are –

a) to abide by the Constitution andrespect its ideals and institutions,the National Flag and the NationalAnthem;

b) to cherish and follow the nobleideals which inspired our nationalstruggle for freedom;

c) to uphold and protect thesovereignty, unity and integrityof India;

d) to defend the country and rendernational service when called upon todo so;

e) to promote harmony and the spiritof common brotherhood amongst allthe people of India transcendingreligious, linguistic and regional orsectional diversities; to renouncepractices derogatory to the dignityof women;

f) to value and preserve the richheritage of our composite culture;

g) to protect and improve the naturalenvironment including forests, lakes,rivers and wild life, and to havecompassion for living creatures;

h) to develop the scientific temper,humanism and the spirit of inquiryand reform;

i) to safeguard public property and toabjure violence;

j) to strive towards excellence in allspheres of individual and collectiveactivity so that the nationconstantly rises to higher levels ofendeavour and achievement.

k) to provide opportunities foreducation by the parent theguardian, to his child, or a wardbetween the age of 6-14 years as thecase may be.

Educational Objective: FundamentalDuties were added to the Constitutionunder Article 51A (Part IVA) by the 42ndConstitutional Amendment Act of 1976based on the recommendations of theSwaran Singh Committee.Source: Indian Polity by Laxmikant (4thEdition) – page no: 9.1-24. Answer: C

Explanation: The 42nd ConstitutionAmendment Act of 1976, has added anew part-IVA to the Constitution ofIndia. Article 51A under Part-IVA ofthe Constitution deals with theFundamental duties. FundamentalDuties were incorporated in the

Page 37: PolityPreviousYearQuestionBank · 1979. 8. 31.  · PolityPreviousYearQuestionBank PageNo.5 Company can give licence of trade to individuals and its own employees to trade in India:

Polity Previous Year Question Bank

www.laex.in https://elearn.laex.inPage No. 34

Constitution based on therecommendations of the Swaran SinghCommittee (1976) however, all of itsrecommendations were not incorporated.The Fundamental Duties are –a) to abide by the Constitution and

respect its ideals and institutions,the National Flag and the NationalAnthem;

b) to cherish and follow the nobleideals which inspired our nationalstruggle for freedom;

c) to uphold and protect thesovereignty, unity and integrity ofIndia;

d) to defend the country and rendernational service when called upon todo so;

e) to promote harmony and the spiritof common brotherhood amongst allthe people of India transcendingreligious, linguistic and regional orsectional diversities; to renouncepractices derogatory to the dignityof women;

f) to value and preserve the richheritage of our compositeculture;

g) to protect and improve the naturalenvironment including forests, lakes,rivers and wild life, and to havecompassion for living creatures;

h) to develop the scientific temper,humanism and the spirit ofinquiry and reform;

i) to safeguard public property and toabjure violence;

j) to strive towards excellence inall spheres of individual andcollective activity so that thenation constantly rises to higherlevels of endeavour andachievement.

k) to provide opportunities foreducation by the parent theguardian, to his child, or a wardbetween the age of 6-14 years as thecase may be.

Protection of weaker sections fromsocial injustice is not a Fundamentalduty under Article 51A.Educational Objective: Protection ofweaker sections from social injustice isnot a Fundamental duty under Article51A.Source: Indian Polity by Laxmikant(4th Edition) – page no: 9.1-2

5. Answer: AExplanation: The 42nd ConstitutionAmendment Act of 1976, has added anew part-IVA to the Constitution ofIndia. Article 51A under Part-IVA ofthe Constitution deals with theFundamental duties. FundamentalDuties were incorporated in theConstitution based on therecommendations of the Swaran SinghCommittee (1976) however, all of itsrecommendations were not incorporated.The Fundamental Duties are –a) to abide by the Constitution and

respect its ideals andinstitutions, the National Flag andthe National Anthem;

b) to cherish and follow the nobleideals which inspired our nationalstruggle for freedom;

c) to uphold and protect thesovereignty, unity and integrity ofIndia;

d) to defend the country and rendernational service when called upon todo so;

e) to promote harmony and the spiritof common brotherhood amongst allthe people of India transcendingreligious, linguistic and regional orsectional diversities; to renouncepractices derogatory to the dignityof women;

f) to value and preserve the richheritage of our composite culture;

g) to protect and improve the naturalenvironment including forests, lakes,rivers and wild life, and to havecompassion for living creatures;

Page 38: PolityPreviousYearQuestionBank · 1979. 8. 31.  · PolityPreviousYearQuestionBank PageNo.5 Company can give licence of trade to individuals and its own employees to trade in India:

Polity Previous Year Question Bank

www.laex.in https://elearn.laex.inPage No. 35

h) to develop the scientific temper,humanism and the spirit of inquiryand reform;

i) to safeguard public propertyand to abjure violence;

j) to strive towards excellence in allspheres of individual and collectiveactivity so that the nationconstantly rises to higher levels ofendeavour and achievement.

k) to provide opportunities foreducation by the parent theguardian, to his child, or a wardbetween the age of 6-14 years as thecase may be.

Voting in public elections is not aFundamental Duty under Article 51A.Educational Objective: Voting inpublic elections is not a FundamentalDuty under Article 51A.

06. Executive

6.1 President6.1.1Presidential Powers andFunctions

1. If the President of Indiaexercises his power as providedunder Article 356 of theConstitution in respect of aparticular State, then (2014)

(a) The Assembly of the State isautomatically dissolved.

(b) The powers of the Legislature of thatState shall be exercisable by or underthe authority of the Parliament

(c) Article 19 is suspended in that State.(d) The President can make laws

relating to that State

2. Consider the followingstatements:

1. The President shall make rules forthe more convenient transaction ofthe business of the Government ofIndia, and for the allocation amongMinisters of the said business.

2. All executive actions of theGovernment of India shall beexpressed to be taken in the name ofthe Prime Minister.

Which of the statements given aboveis/are correct? (2014)

(a) 1 only(b) 2 only(c) Both 1 and 2(d) Neither 1 nor 2

3. According to the Constitution ofIndia, it is the duty of thePresident of India to cause to belaid before the Parliament whichof the following? (2012)

1. The Recommendations of the UnionFinance Commission

2. The Report of the Public AccountsCommittee

3. The Report of the Comptroller andAuditor General

4. The Report of the NationalCommission for Scheduled Castes

Select the correct answer using thecodes given below:(a) 1 only(b) 2 and 4 only(c) 1, 3 and 4 only(d) 1,2,3 and 4

4. Under which Article of theIndian Constitution did thePresident make a reference tothe Supreme Court to seek theCourt's opinion on theConstitutional validity of theElection Commission's decisionon deferring the GujaratAssembly Elections (in the year2002)? [2003]

(a) Article 142(b) Article 143(c) Article 144(d) Article 145

Page 39: PolityPreviousYearQuestionBank · 1979. 8. 31.  · PolityPreviousYearQuestionBank PageNo.5 Company can give licence of trade to individuals and its own employees to trade in India:

Polity Previous Year Question Bank

www.laex.in https://elearn.laex.inPage No. 36

5. Which one of the followingamendments to the IndianConstitution empowers thePresident to send back anymatter for reconsideration by theCouncil of Ministers? [2002]

(a) 39th(b) 40th(c) 42nd(d) 44th6. With reference to Indian polity,

which one of the followingstatements is correct? (2002)

(a) Planning Commission is accountableto the Parliament

(b) President can make ordinance onlywhen either of the two Houses ofParliament is not in session

(c) The minimum age prescribed forappointment as a Judge of theSupreme Court is 40 years

(d) National Development Council isconstituted of Union FinanceMinister and the Chief Ministers ofall the States

6.1.2 Election of the President

7. Consider the followingstatements: In the electoralcollege for Presidential Electionin India

1. The value of the vote of an electedMember of Legislative Assemblyequals State Population? 100 Numberof Elected MLAs of the State

2. The value of the vote of an electedMember of Parliament equals to totalvalue of the votes of all elected MLA’sand total number of elected MPs

3. There were more than 5000 membersin the latest elections.

Which of these statements is/arecorrect? (2003)

(a) 1 and 2

(b) Only 2(c) 1 and 3(d) Only 3

6. 2 The Governor6.2 .1 Cnditions of Governor Office

8. Consider the followingstatements:

1. No criminal proceedings shall beinstituted against the Governor of aState in any court during his term ofoffice.

2. The emoluments and allowances ofthe Governor of a State shall not bediminished during his term of office.

Which of the statements given aboveis/are correct? (2018)

(a) 1 only(b) 2 only(c) Both 1 and 2(d) Neither 1 nor 2

6.2.2 Appointment and Removal ofGovernor

9. In India, the same person cannotbe appointed as Governor for twoor more States at the same time(2013)

(a) In India, the same person cannot beappointed as Governor for two ormore States at the same time.

(b) The Judges of the High Court of theStates in India are appointed by theGovernor of the State just as theJudges of Supreme Court areappointed by the President

(c) No procedure has been laid down inthe Constitution of India for theremoval of a Governor from his/herpost

(d) In the case of a Union Territoryhaving a legislative setup, the ChiefMinister is appointed by the Lt.Governor on the basis of majoritysupport

Page 40: PolityPreviousYearQuestionBank · 1979. 8. 31.  · PolityPreviousYearQuestionBank PageNo.5 Company can give licence of trade to individuals and its own employees to trade in India:

Polity Previous Year Question Bank

www.laex.in https://elearn.laex.inPage No. 37

6.2.3 Constitutional position ofGovernor

10. Which of the following are thediscretionary powers given to theGovernor of a State?

1. Sending a report to the President ofIndia for imposing the President'srule

2. Appointing the Ministers3. Reserving certain bills passed by the

State Legislature for consideration ofthe President of India

4. Making the rules to conduct thebusiness of the State Government

Select the correct answer using thecode given below.

(a) 1 and 2 only(b) 1 and 3 only(c) 2, 3 and 4 only(d) 1, 2, 3 and 4

6.3 Prime Minister6.3.1 Powers and Functions of the

Prime Minister

11. Consider the followingstatements

1. The Executive power of the Union ofIndia is vested in the Prime Minister.

2. The Prime Minister is the ex officioChairman of the Civil Services Board.

Which of the statements given above is/ are correct?

(a) 1 only(b) 2 only(c) Both 1 and 2(d) Neither 1 nor 2

12. With reference to UnionGovernment, consider thefollowing statements:

1. The Ministries and Departments ofthe Government of India are createdby the Prime Minister on the adviceof the Cabinet Secretary.

2. Each of the ministries is assigned toa Minister by the President of Indiaon the advice of the Prime Minister.

Which of the statements given aboveis/are correct? (2009)

(a) 1 only(b) 2 only(c) Both 1 and 2(d) Neither 1 nor 2

6.3.2 Appointment of the PrimeMinister

13. The Prime Minister of India, atthe time of his/her appointment(2010)

(a) Need not necessarily be a member ofone of the Houses of the Parliamentbut must become a member of one ofthe Houses within six months

(b) Must be a member of the Lok Sabha(c) Need not necessarily be a member of

one of the Houses of the Parliamentbut must become a member of theLok Sabha within six months

(d) Must be a member of one of theHouses of the Parliament

6.3.3 Factual

14. Consider the followingstatements:

1. Jawaharlal Nehru was in his fourthterm as the Prime Minister of Indiaat the time of his death.

2. Jawaharlal Nehru represented RaeBareilly constituency as a Member ofParliament.

3. The first non-Congress PrimeMinister of India assumed the officein the year 1977.

Page 41: PolityPreviousYearQuestionBank · 1979. 8. 31.  · PolityPreviousYearQuestionBank PageNo.5 Company can give licence of trade to individuals and its own employees to trade in India:

Polity Previous Year Question Bank

www.laex.in https://elearn.laex.inPage No. 38

Which of the statements given aboveis/are correct? (2007)

(a) 1 and 2(b) 3 only(c) 1 only(d) 1 and 3

15. The Ninth Schedule wasintroduced in the Constitution ofIndia during the PrimeMinistership of

(a) Jawaharlal Nehru(b) Lal Bahadur Shastri(c) Indira Gandhi(d) Morarji Desai

6.4 The Vice President of India6.4 .1 Factual

16. Who among the following haveheld the office of the Vice-President of India?

1. Mohammad Hidayatullah2. Fakhruddin Ali Ahmed3. NeelamSanjiva Reddy4. Shankar Dayal SharmaSelect the correct answer using the

code given below:(a) 1, 2, 3 and 4(b) 1 and 4 only(c) 2 and 3 only(d) 3 and 4 only

6.4.2 Term of Office of the VicePresident of India

17. The resolution for removing theVice-President of India can bemoved in the (2003).

(a) Lok Sabha alone(b) Either House of Parliament(c) Joint Sitting of Parliament(d) Rajya Sabhaalon.

6.5 Secretariats6.5 .1 General

18. Which of the following is/are thefunction/functions of the CabinetSecretariat?

1. Preparation of agenda for CabinetMeetings

2. Secretarial assistance to CabinetCommittees

3. Allocation of financial resources tothe Ministries

Select the correct answer using thecode given below. (2014)

(a) 1 only(b) 2 and 3 only(c) 1 and 2 only(d) 1, 2 and 319. Consider the following

statements:1. The Chief Secretary in a State is

appointed by the Governor of thatState.

2. The Chief Secretary in a State has afixed tenure.

Which of the statements given aboveis/are correct? (2017)

(a) 1 only(b) 2 only(c) Both 1 and 2(d) Neither 1 nor 2

6.6 Executive Council of Ministers6.6 .1 Factual

20. Which one of the followingConstitutional Amendmentsstates that the total number ofMinisters, including the PrimeMinister, in the Council ofMinister shall not exceed fifteenpercent of the total number of

Page 42: PolityPreviousYearQuestionBank · 1979. 8. 31.  · PolityPreviousYearQuestionBank PageNo.5 Company can give licence of trade to individuals and its own employees to trade in India:

Polity Previous Year Question Bank

www.laex.in https://elearn.laex.inPage No. 39

members of the House of thePeople? (2009).

(a) 90th

(b) 91st

(c) 92nd

(d) 93rd

21. With reference to UnionGovernment, consider thefollowing statements:

1. Number of Ministries at the Centeron 15th August 1947 was 18.

2. Number of Ministries at the Centerat present is 36.

Which of the statements given aboveis/are correct? (2009)

(a) 1 only(b) 2 only(c) Both 1 and 2(d) Neither 1 nor 2

6.7 Executive and the Parliament ofIndia relations

22. Consider the followingstatements:

1. The Council of Ministers in theCentre shall be collectivelyresponsible to the Parliament.

2. The Union Ministers shall hold theoffice during the pleasure of thePresident of India.

3. The Prime Minister shallcommunicate to the President aboutthe proposals for legislation.

Which of the statements given aboveis/are correct? (2013)

(a) 1 only(b) 2 and 3 only(c) 1 and 3 only(d) 1, 2 and 3

6.8 The Attorney General andAdvocate General of India

6.8 .1 Appointments

23. Consider the followingstatements:

1. The Advocate General of a State inIndia is appointed by the President ofIndia upon the recommendation ofthe Governor of the concerned State.

2. As provided in Civil Procedure Code,High Courts have original, appellateand advisory jurisdiction at the Statelevel. Which of the statements givenabove is/are correct? (2009)

(a) 1 only(b) 2 only(c) Both 1 and 2(d) Neither 1 nor 224. Consider the following

statements about the AttorneyGeneral of India:

1. He is appointed by the President ofIndia

2. He must have the same qualificationsas are required for a judge of theSupreme Court

3. He must be a member of either Houseof Parliament

4. He can be removed by impeachmentby Parliament.

Which of these statements are correct?(2000)

(a) 1 and 2(b) 1 and 3(c) 1 and 3(d) 3 and 4

6.8 .2 Rights and Limitations

25. Consider the followingstatements: Attorney General ofIndia can

1. Take part in the proceedings of theLok Sabha

Page 43: PolityPreviousYearQuestionBank · 1979. 8. 31.  · PolityPreviousYearQuestionBank PageNo.5 Company can give licence of trade to individuals and its own employees to trade in India:

Polity Previous Year Question Bank

www.laex.in https://elearn.laex.inPage No. 40

2. Be a member of a committee of theLok Sabha

3. Speak in the Lok Sabha4. Vote in the Lok SabhaWhich of the statements given aboveis/are correct?(a) 1 only(b) 2 and 4(c) 1, 2 and 3(d) 1 and 3 only

6.9 Miscellaneous

26. Match List-I with List-II andselect the correct answer usingthe codes given below the lists:List-I List-II (Article ofthe (Content) Constitution)

A. Article 54 1. Election of thePresident of India

B. Article 75 2. Appointment of thePrime Minister

C. Article 155 3. Appointment of theGovernor of a State

D. Article 164 4. Appointment of theChief Minister and Council of Ministersof a State

Codes:(a) A-1, B-2, C-3, D-4(b) A-1, B-2, C-4, D-5(c) A-2, B-1, C-3, D-5(d) A-2, B-1, C-4, D-3

27. As per Indian Protocol, whoamong the following ranks highest inthe order of precedence? (2003).(a) Deputy Prime Minister(b) Former Presidents(c) Governor of a State within his State(d) Speaker of the Lok Sabha

Key & Explanation1. Answer: B

Explanation: Article 356 deals withPresident’s rule in the state. OncePresident’s rule is imposed, LegislativeAssembly of the state goes in to suspendedanimation till parliament approves it.The power to legislate on state list isdelegated to Parliament of India.Article 19 is suspended automaticallyunder National Emergency not President’srule.President can enforce the laws with thehelp of the Governor of the state. He willnot legislate on state matters.Educational Objectives: President’s rulewill not disturb Fundamental Rights. OnlyNational Emergency does it.

a) In S R Bommai Judgement, theSupreme Court held that Statelegislative assembly will not bedissolved till parliament approvesPresident’s rule.

Source: Indian Polity (Laxmikanth) – Pageno: 16.7

2. Answer: AExplanation: All executive actions of theGovernment of India are formally taken inthe name of the President only.He can make rules specifying the mannerin which the orders and other instrumentsmade and executed in his name shall beauthenticated.He can make rules for more convenienttransaction of business of the UnionGovernment, and for allocation among theMinisters, of the said business.Educational Objective: President canseek any information relating to theadministration of affairs of the Union, andproposals for legislation from the PrimeMinister.Source: Indian Polity (Laxmikanth) – Pageno: 17.7

3. Answer: CExplanation: The President of Indiaconstitutes a Finance Commission afterevery 5 years to recommend thedistribution of revenues between the

Page 44: PolityPreviousYearQuestionBank · 1979. 8. 31.  · PolityPreviousYearQuestionBank PageNo.5 Company can give licence of trade to individuals and its own employees to trade in India:

Polity Previous Year Question Bank

www.laex.in https://elearn.laex.inPage No. 41

Centre and the States. The commissionsubmits its report to the President. He laysit before both the Houses of Parliamentalong with an explanatory memorandum asto the action taken on its recommendations.The CAG submits three audit reports to thePresident and he lays these reports beforeboth the Houses of Parliament. After this,the Public Accounts Committee examinesthem and reports it findings directly to theParliament.The National Commission for ScheduledCastes presents an annual report to thePresident. The president places the reportbefore the Parliament.Educational Objectives: The President ofIndia enjoys the executive, legislative,financial, judicial, diplomatic, military andemergency powers.Source: Indian Polity (Laxmikanth) Pages:45, 49.2, and 46.2.

4. Answer: BExplanation: The Article 143 of theConstitution authorises the President toseek the opinion of the Supreme Court inthe two categories of matters: (a) On anyquestion of law or fact of public importancewhich has arisen or which is likely to arise.(b) On any dispute arising out of any pre-constitution treaty, agreement, covenant,engagement, sanad or other similarinstruments.Educational Objective: Until 2013, thePresident has made 15 references to theSupreme Court under its advisoryjurisdiction (also known consultativejurisdiction), which include theconstitutional validity of the ElectionCommission’s decision on deferring theGujarat Assembly Elections in 2002.Source: Indian Polity (Laxmikanth) Page:26.7

5. Answer: DExplanation: Article 74 provides for aCouncil of Ministers with the PrimeMinister at the head to aid and advice thePresident in the exercise of his functions.The 42nd and 44th Constitutional

Amendment Acts have made the advicebinding on the President.Educational Objective: The Presidentcannot exercise the executive powerwithout the aid and advise of the Council ofMinisters.Source: Indian Polity (Laxmikanth) Page:20.2

6. Answer: BExplanation: The erstwhile PlanningCommission was only an advisory body andhad no executive responsibility. It was notresponsible for taking and implementingdecisions. This responsibility rested withthe Central and State Governments. So it isaccountable to Government not Parliament.President can promulgate ordinances whenthe Parliament is not in session. Theseordinances must be approved by theParliament within 6 weeks from itsreassembly. He can also withdraw anordinance at any time.The Constitution of India has notprescribed a minimum age for appointmentas a judge of the Supreme Court.The National Development Council iscomposed of the Prime Minister, All UnionCabinet Ministers, the Chief Ministers ofall the states, ChiefMinisters/Administrators of all UT’s, andmembers of the Planning Commission.Educational Objective: NITI Aayog wasformed in 2015 in the place of PlanningCommission.National Development Council proposed tobe abolished and transfer its powers to theGoverning Council of NITI Aayog.Source: Indian Polity (Laxmikanth) Pages:52.8, 17.8, 26.2 and 52.10

7. Answer: BExplanation: The President is elected notdirectly by the people but by members ofelectoral college. Every elected member ofthe legislative assembly of a state shallhave as many votes as there are multiplesof 1000 in the quotient obtained by dividingthe population of the state by the total

Page 45: PolityPreviousYearQuestionBank · 1979. 8. 31.  · PolityPreviousYearQuestionBank PageNo.5 Company can give licence of trade to individuals and its own employees to trade in India:

Polity Previous Year Question Bank

www.laex.in https://elearn.laex.inPage No. 42

number of the elected members of theassembly.Every elected member of either House ofParliament shall have such number of votesas may be obtained by dividing the totalnumber of votes assigned to members of thelegislative assemblies of the states by thetotal number of the elected members ofboth the Houses of Parliament.Educational Objective: The Constitutionof India provides that there shall beuniformity in the scale of representation ofdifferent states as well as parity betweenthe states as a whole and the Union at theelection of the President.Source: Indian Polity (Laxmikanth) Page:17

8. Answer: CExplanation: Under Article 361 of theIndian Constitution, no criminalproceedings shall be instituted or continuedagainst the President or the Governor inany court during his term of office.The emoluments and allowances of theGovernor of a State cannot be diminishedduring his term of office.Educational Objective: Like thePresident, the Governor is also entitled to anumber of privileges and immunities.Source: Indian Polity (Laxmikanth) Page:30.4

9. Answer: DExplanation: The 7th ConstitutionalAmendment Act of 1956 facilitated theappointment of the same person as agovernor for two or more states.The Judges of the Supreme Court and HighCourts are appointed by the President only.The Governor has no security of tenure andno fixed term of office. He may be removedby the President at any time.Educational Objective: The council ofministers headed by the Chief Minister aidand advises the lt. governor in the exerciseof his functions except in so far as he isrequired to act in his discretion.

Source: Indian Polity (Laxmikanth) Pages:30, 34, 30.4

10. Answer: BExplanation: The Governor can usediscretionary powers: when no party gets aclear majority, he can use his discretion inthe selection of Chief Ministerial candidateto prove the majority as soon as possible.He can impose President's rule.When a bill is sent to the Governor after itis passed by State Legislature, he canreserve the bill for the consideration of thePresident.Educational Objective: The discretionarypowers enjoyed by the Governor reduce tosome extent the power, authority, influence,prestige and role of the Chief Minister inthe State Administration.

Source:https://en.wikipedia.org/wiki/Governor_(India)#Discretionary_powers

11. Answer: DExplanation: Executive power of theUnion is vested in the President, and isexercised by him either directly or throughofficers subordinates to him in accordancewith the Constitution.The Cabinet Secretary is the ex-officioChairman of the Civil Services Board; thechief of the Indian Administrative Serviceand head of all civil services under therules of business of the Government ofIndia.Educational Objective: All the executiveactions of the Government of India areformally taken in President’s name.The Cabinet Secretary is the highestranking civil servant in India.Source: Indian Polity (Laxmikanth) Pages:17.7 andhttps://en.wikipedia.org/wiki/Cabinet_Secretary_of_India

12. Answer: BExplanation: The Government ofIndia (Allocation of Business) Rules 1961are made by the President of India underArticle 77 of the Constitution for the

Page 46: PolityPreviousYearQuestionBank · 1979. 8. 31.  · PolityPreviousYearQuestionBank PageNo.5 Company can give licence of trade to individuals and its own employees to trade in India:

Polity Previous Year Question Bank

www.laex.in https://elearn.laex.inPage No. 43

allocation of business of the Government ofIndia. The Ministries/Departments of theGovernment are created by the Presidenton the advice of the Prime Minister underthese Rules.Educational Objective: The PrimeMinister is appointed by the President whoalso appoints other ministers on theadvice of Prime Minister. The Council iscollectively responsible to the Lok Sabha.Source:

https://knowindia.gov.in/profile/the-union/administrative-set-up.php

13. Answer: AExplanation: Article 75 of theConstitution says only that the PrimeMinister shall be appointed by thepresident.In accordance with the conventions of theparliamentary system of government, thePresident has to appoint the leader of themajority party in the Lok Sabha as thePrime Minister.In 1997, the Supreme Court held that aperson who is not a member of either Houseof Parliament can be appointed as PrimeMinister for six months, within which, heshould become a member of either House ofParliament; otherwise, he ceases to be thePrime Minister. Constitutionally, the PrimeMinister may be a member of any of thetwo Houses of parliament.Educational Objective: The Constitutiondoes not contain any specific procedure forthe selection and appointment of the PrimeMinister.Source: Indian Polity (Laxmikanth) Page:19.1

14. Answer: DExplanation: Jawaharlal Nehru emergedas an eminent leader of the Indianindependence movement under the tutelageof Mahatma Gandhi and served India asPrime Minister from its establishment asan independent nation in 1947 until hisdeath in 1964.The first Prime Minister of Indiaof Jawaharlal Nehru died in office in 1964

while holding the seat in Phulpurconstituency. Hence it is also called"Nehru's constituency”.Morarji Desai was selected by the Janataalliance, later Janata Party as theirparliamentary leader, and thus became thefirst non-Congress Prime Minister of India.Educational Objectives: Phulpur is aprestigious constituency and two IndianPrime Ministers Jawaharlal Nehru and V PSingh have been elected fromthis constituency.Source:https://en.wikipedia.org/wiki/Jawaharlal_Nehruhttps://en.wikipedia.org/wiki/Phulpur_(Lok_Sabha_constituency)https://en.wikipedia.org/wiki/Morarji_Desai

15. Answer: AExplanation: Ninth Schedule was addedby the 1st Amendment (1951) to protect thelaws included in it from judicial scrutiny onthe ground of violation of fundamentalrights. However, in 2007, the SupremeCourt ruled that the laws included in thisschedule after April 24, 1973, are now opento judicial review.Educational objective: Article 31B alongwith the Ninth Schedule was added by the1st Constitutional Amendment Act of 1951.Source: Indian Polity (Laxmikanth) Page:27.3

16. Answer: BExplanation: Mohammad Hidayatullahwas the sixth Vice President of India,serving from 31 August 1979 to 30 August1984. He had also served as theActing President of India. Shankar DayalSharma was the ninth President of India,serving from 1992 to 1997. Prior to hispresidency, he had been the eighth VicePresident of India, serving under R.Venkataraman.Educational Objective: On 11 August2017, Venkaiah Naidu was sworn in as the13th Vice President of India.

Page 47: PolityPreviousYearQuestionBank · 1979. 8. 31.  · PolityPreviousYearQuestionBank PageNo.5 Company can give licence of trade to individuals and its own employees to trade in India:

Polity Previous Year Question Bank

www.laex.in https://elearn.laex.inPage No. 44

Source:https://en.wikipedia.org/wiki/Mohammad_Hidayatullahhttps://en.wikipedia.org/wiki/Shankar_Dayal_Sharma

17. Answer: DExplanation: Article 67(b) of theConstitution states that the Vice Presidentcan be removed by a resolution of the RajyaSabha passed by an effective majority(i.e.,majority of all the then member of thehouse) and agreed by the Lok Sabha withsimple majority.Educational Objective: No ground hasbeen mentioned in the Constitution for theremoval of the Vice President of India.Source: Indian Polity (Laxmikanth) Page:18.2

18. Answer: CExplanation: The Cabinet Secretariatfunctions directly under the Prime Minister.He is responsible for the administration ofthe Government of India (Transaction ofBusiness) Rules 1961. The businessallocated to Cabinet Secretariat underthese rules includes (i) Secretarialassistance to the Cabinet and CabinetCommittees; and (ii) Rules of Business.Educational Objective: Wherever theMinistries face any difficulties, the CabinetSecretariat and the Prime Minister’s Officewill facilitate the decision-making process.Source:https://cabsec.gov.in/about/functions/

19. Answer: DExplanation: The Chief Secretary is thesenior-most position held in the civilservices of the states and union territoriesof India. In a State, he is appointed bythe Chief Minister under the seal of theGovernor.No fixed tenure is imposed on the ChiefSecretary office but term can be extended.Educational Objective: The ChiefSecretary is usually the senior most IASofficer of the senior most batch in the state.

Source:https://en.wikipedia.org/wiki/Chief_secretary_(India)

20. Answer: BExplanation: The 91st Amendment Act of2003 has made some provisions to limit thesize of Council of Ministers, to debardefectors from holding public offices, and tostrengthen the anti-defection law.Article 75 of the Constitution states thatthe total number of ministers, including thePrime Minister, in the Central Council ofMinisters shall not exceed 15 percent of thetotal strength of the Lok Sabha.Educational Objective: The 91stAmendment Act of 2003 made one changein the provisions of the Tenth Schedule. Itomitted an exception provision i.e.,disqualification on ground of defection notto apply in case of split.Source: Indian Polity (Laxmikanth) Page:72.3

21. Answer: AExplanation: The Government consists ofa number of Ministries/Departments. Thenumber and character varying from time totime on factors such as volume of workimportance attached to certain items,changes of orientation, political expediency,etc. On 15 August 1947, the number ofMinistries at the Centre was 18.Educational Objective: The 91stAmendment Act of 2003 made one changein the provisions of the Tenth Schedule. Itomitted an exception provision i.e.,disqualification on ground of defection notto apply in case of split.Source:https://knowindia.gov.in/profile/the-union/administrative-set-up.php22. Answer: BExplanation: Article 75 of theConstitution clearly states that the Councilof Ministers is responsible to theParliament in general and to the LokSabha in particular.Article 75 also contains the principle ofindividual responsibility. It states that the

Page 48: PolityPreviousYearQuestionBank · 1979. 8. 31.  · PolityPreviousYearQuestionBank PageNo.5 Company can give licence of trade to individuals and its own employees to trade in India:

Polity Previous Year Question Bank

www.laex.in https://elearn.laex.inPage No. 45

Ministers hold office during the pleasure ofthe President.Article 78 states that it shall be the duty ofthe Prime Minister to communicate to thePresident all decisions of the Council ofMinisters relating to the administration ofthe affairs of the Union and proposals forlegislation.

Educational Objective: Article 164 ofthe Constitution also contains theprinciple of individual responsibility. Itstates that the Council of Ministers ofState holds office during the pleasure ofthe Governor of that particular State.Source: Indian Polity (Laxmikanth)Pages: 12.4, 20.3 and 20.2

23. Answer: DExplanation: Governor appoints theAdvocate General of a State anddetermines his remuneration. The AdvocateGeneral holds office during the pleasure ofthe Governor.High Courts have original, writ, appellate,supervisory jurisdiction at the State level.Educational Objective: Only SupremeCourt has advisory jurisdiction but HighCourts do not.Source: Indian Polity (Laxmikanth) Pages:51.1 and 34.5

24. Answer: AExplanation: The Attorney General ofIndia is appointed by the President. Hemust be a person who is qualified to beappointed a judge of the Supreme Court.The Constitution does not contain theprocedure and grounds for his removal. Heholds office during the pleasure of thePresident.Educational Objective: Attorney Generalis the highest law officer in the country. Itis the duty of Attorney General to appearon behalf of the Government of India in allcases in the Supreme Court in which theGovernment of India is concerned.Source: Indian Polity (Laxmikanth) Page:50.1

25. Answer: CExplanation: Attorney General has theright to speak and to take part in theproceedings of both the Houses ofParliament or their joint sitting and anycommittee of the Parliament of which hemay be named a member, but without aright to vote.He enjoys all the privileges and immunitiesthat are available to a Member ofParliament.Educational Objective: Attorney Generalhas the right of audience in all courts in theterritory of India.

Source: Indian Polity (Laxmikanth)Page: 50.1

26. Answer: AExplanation: Article 54 of theConstitution states that the President shallbe elected by the members of an electoralcollege consisting of the elected members ofboth Houses of Parliament; and the electedmembers of the Legislative Assemblies ofthe States.Article 75 says only that the PrimeMinister shall be appointed by thepresident.Article 164 states that the Chief Ministershall be appointed by the Governor and theother Ministers shall be appointed by theGovernor on the advice of the ChiefMinister, and the Ministers shall hold officeduring the pleasure of the Governor.Educational Objective: The LegislativeAssembly consists of representativesdirectly elected by the people on the basis ofUniversal Adult Franchise. Its maximumstrength is fixed at 500 and minimumstrength at 60.Source: Indian Polity (Laxmikanth) Pages:17.1, 30.1, 19.4 and 31.1

27. Answer: CExplanation: The order of precedence isthe protocol list in which the functionaries,dignitaries and officials are listed accordingto their rank and office in the Governmentof India. The order is established by

Page 49: PolityPreviousYearQuestionBank · 1979. 8. 31.  · PolityPreviousYearQuestionBank PageNo.5 Company can give licence of trade to individuals and its own employees to trade in India:

Polity Previous Year Question Bank

www.laex.in https://elearn.laex.inPage No. 46

the President of India and is maintained bythe Ministry of Home Affairs.The Order of Precedence includes (1)President (2) Vice President (3) PrimeMinister (4) Governors of States (5) FormerPresidents (6) Chief Justice and Speaker ofthe Lok Sabha, etc.Educational Objective: The Order ofPrecedence is not applicable for the day-to-day functioning of the Government of India.Source:https://en.wikipedia.org/wiki/Indian_order_of_precedence

07. Legislature

7.1 Structure of Legislature Lok Sabhaand Rajya Sabha

7.1 .1 Privileges and Limitations

1. Consider the followingstatements:

1. The Rajya Sabha has no power eitherto reject or to amend a Money Bill.

2. The Rajya Sabha cannot vote on theDemands for Grants.

3. The Rajya Sabha cannot discuss theAnnual Financial Statement.

Which of the statements given above is/ are correct?

a) 1 onlyb) 1 and 2 onlyc) 2 and 3 onlyd) 1, 2 and 3

2. Assertion (A): The Council ofMinisters in the Union of India iscollectively responsible both to theLok Sabha and Rajya Sabha.Reason (R): The Members of boththe Lok Sabha and the Rajya Sabhaare eligible to be the Ministers of theUnion Government.

a) Both A and Rare true and R is thecorrect explanation of A

b) Both A and R are true but R is not acorrect explanation of A

c) A is true but R is falsed) A is false but R is true3. Consider the following

statements:1. The Rajya Sabha alone has the power

to declare that it would be in nationalinterest for the Parliament tolegislate with respect to a matter inthe State List.

2. Resolutions approving theproclamation of Emergency arepassed only by the Lok Sabha.

Which of the statement(s) given aboveis/are correct?a) 1 onlyb) 2 onlyc) Both 1 and 2d) Neither 1 nor 2

4. Which one of the followingstatements is correct?

a) Only the Rajya Sabha and not theLok Sabha can have nominatedmembers.

b) There is a constitutional provision fornominating two members belongingto the Anglo-Indian community to theRajya Sabha.

c) There is no constitutional bar for anominated member to be appointedas a Union minister.

d) A nominated member can vote bothin the Presidential and Vice-Presidential elections.

5. Which of the following specialpowers have been conferred onthe Rajya Sabha by theConstitution of India?

a) To change the existing territory of aState and to change the name of aState.

b) To pass a resolution empowering theParliament to make laws in the StateList and to create one or more AllIndia Services.

Page 50: PolityPreviousYearQuestionBank · 1979. 8. 31.  · PolityPreviousYearQuestionBank PageNo.5 Company can give licence of trade to individuals and its own employees to trade in India:

Polity Previous Year Question Bank

www.laex.in https://elearn.laex.inPage No. 47

c) To amend the election procedure ofthe President and to determine thepension of the President after his/herretirement.

d) To determine the functions of theElection Commission and todetermine the number of ElectionCommissioners.

7.1.2 Joint Sitting

6. A deadlock between the LokSabha and the Rajya Sabha callsfor a joint sitting of theParliament during the passage of

1. Ordinary Legislation2. Money Bill3. Constitution Amendment Bill

Select the correct answer using thecodes given below:

a) 1 onlyb) 2 and 3 onlyc) 1 and 3 onlyd) 1, 2 and 3

7. Consider the followingstatements:

1. The joint sitting of the two houses ofthe Parliament in India is sanctionedunder Article 108 of the Constitution.

2. The first joint sitting of Lok Sabhaand Rajya Sabha was held in the year1961.

3. The second joint sitting of the twoHouses of Indian Parliament washeld to pass the Banking ServiceCommission (Repeal) Bill.

Which of these statements is correct?a) 1 and 2b) 2 and 3c) 1 and 3d) 1, 2 and

7.1.3 Miscellaneous

8. Consider the followingstatements:

1. Union Territories are not representedin the Rajya Sabha.

2. It is within the purview of the ChiefElection Commissioner to adjudicatethe election disputes.

3. According to the Constitution of India,the Parliament consists of the LokSabha and the Rajya Sabha only.

Which of the statements given aboveis/are correct?

a) 1 onlyb) 2 and 3 onlyc) 1 and 3 onlyd) None

9. The term of the Lok Sabha:a) Cannot be extended under any

circumstances.b) Can be extended by six months at a

time.c) Can be extended by one year at a

time during the proclamationofemergency.

d) Can be extended for two years at atime during the proclamation ofemergency.

7.2 Membership of the Houses7.2 .1 Election to the Lok Sabha

10. Consider the following statements:1. In the election for Lok Sabha or State

Assembly, the winning candidatemust get at least 50 percent of thevotes polled, to be declared elected.

2. According to the provisions laid downin the Constitution of India, in LokSabha, the Speaker’s post goes to themajority party and the DeputySpeaker’s to the Opposition.

Which of the statements given aboveis/are correct?a) 1 only

Page 51: PolityPreviousYearQuestionBank · 1979. 8. 31.  · PolityPreviousYearQuestionBank PageNo.5 Company can give licence of trade to individuals and its own employees to trade in India:

Polity Previous Year Question Bank

www.laex.in https://elearn.laex.inPage No. 48

b) 2 onlyc) Both 1 and 2d) Neither 1 nor 2

11. For election to the Lok Sabha, anomination paper can be filed by

a) Anyone residing in India.b) A resident of the constituency from

which the election is to be contested.c) Any citizen of India whose name

appears in the electoral roll of aconstituency.

d) Any citizen of India.

7.2 .2 Disqualification of Members

12. Consider the followingstatements:

1. The Parliament (Prevention ofDisqualification) Act, 1959 exemptsseveral posts from disqualification onthe grounds of ‘Office of Profit’.

2. The above-mentioned Act wasamended five times.

3. The term ‘Office of Profit’ is well-defined in the Constitution of India.

Which of the statements given aboveis/ are correct?

(a) 1 and 2 only(b) 3 only(c) 2 and 3 only(d) 1, 2 and 3

7.2 .3 Leader of Opposition

13. Consider the followingstatements:

1. In the first Lok Sabha, the singlelargest party in the opposition wasthe Swatantra Party.

2. In the Lok Sabha, a "Leader of theOpposition" was recognized for thefirst time in 1969.

3. In the Lok Sabha, if a party does nothave a minimum of 75 members, its

leader cannot be recognized as theLeader of the Opposition.

Which of the statements given aboveis/are correct?a) 1 and 3 onlyb) 2 onlyc) 2 and 3 onlyd) 1, 2 and 37.3 Power and functions of Parliament

7.3 .1 Emergency Powers

14. Consider the followingstatements in respect of financialemergency under Article 360 ofthe Constitution of India:

1. A proclamation of financialemergency issued shall cease tooperate at the expiration of twomonths, unless before the expirationof that period it has been approved bythe resolutions of both Houses ofParliament.

2. If any proclamation of financialemergency is in operation, it iscompetent for the President of Indiato issue directions for the reduction ofsalaries and allowances of all or anyclass of persons serving in connectionwith the affairs of the Union butexcluding the Judges of SupremeCourt and the High Courts.

Which of the statements given aboveis/are correct?a) 1 onlyb) 2 onlyc) Both 1 and 2d) Neither 1 nor 2

7.3.2 - In relation to States

15. The Parliament of India acquiresthe power to legislate on anyitem in the State List in thenational interest if a resolutionto that effect is passed by the

a) Lok Sabha by a simple majority of itstotal membership.

Page 52: PolityPreviousYearQuestionBank · 1979. 8. 31.  · PolityPreviousYearQuestionBank PageNo.5 Company can give licence of trade to individuals and its own employees to trade in India:

Polity Previous Year Question Bank

www.laex.in https://elearn.laex.inPage No. 49

b) Lok Sabha by a majority of not lessthan two-thirds of its totalmembership.

c) Rajya Sabha by a simple majority ofits total membership.

d) Rajya Sabha by a majority of not lessthan two-thirds of its memberspresent and voting.

16. The Parliament can make anylaw for whole or any part of Indiafor implementing internationaltreaties.

a) with the consent of all the Statesb) with the consent of the majority of

Statesc) with the consent of the States

concernedd) without the consent of any State

7.3.3 - Judicial Powers

17. The power to increase thenumber of judges in the SupremeCourt of India is vested in

a) the President of Indiab) the Parliamentc) the Chief Justice of Indiad) the Law Commissione)

7.3.4 -9th Schedule

18. Consider the followingstatements:

1. The Parliament of India can place aparticular law in the Ninth Scheduleof the Constitution of India.

2. The validity of a law placed in theNinth Schedule cannot be examinedby any court and no judgement canbe made on it.

Which of the statements given aboveis/are correct?

a) 1 onlyb) 2 only

c) Both 1 and 2d) Neither 1 nor 2

7.4 Legislative Control over Executive7.4.1 Devices of Parliamentary Control

19. The Parliament of Indiaexercises control over thefunctions of the Council ofMinisters through

1. Adjournment motion2. Question hour3. Supplementary questionsSelect the correct answer using the

code given below:a) 1 onlyb) 2 and 3 onlyc) 1 and 3 onlyd) 1,2 and 3

20. Consider the followingstatements regarding a No-Confidence Motion in India:

1. There is no mention of a No-Confidence Motion in theConstitution of India.

2. A Motion of No-Confidence can beintroduced in the Lok Sabha only.

Which of the statements given aboveis/are correct?a) 1 onlyb) 2 onlyc) Both 1 and 2d) Neither 1 nor 2

21. In the Parliament of India, thepurpose of an adjournmentmotion is

a) to allow a discussion on a definitematter of urgent public importance

b) to let opposition members, collectinformation from the ministers

c) to allow a reduction of specificamount in demand for grant

Page 53: PolityPreviousYearQuestionBank · 1979. 8. 31.  · PolityPreviousYearQuestionBank PageNo.5 Company can give licence of trade to individuals and its own employees to trade in India:

Polity Previous Year Question Bank

www.laex.in https://elearn.laex.inPage No. 50

d) to postpone the proceedings to checkthe inappropriate or violent behavioron the part of some members

22. Which of the followingstatements is not correct? (2004)

a) In Lok Sabha a no confidence motionhas to set out the grounds on which itis based.

b) In the case of a no confidence motionin Lok Sabha no conditions ofadmissibility have been laid down inthe rules.

c) A motion of no confidence onceadmitted has to be taken up within10 days of the leave being granted.

d) Rajya Sabha is not empowered toentertain a motion of no confidence.

23. In what way does the IndianParliament exercise control overthe administration?

a) Through Parliamentary Committeesb) Through Consultative Committees in

various ministriesc) By making the administrators send

periodic reportsd) By compelling the executive to issue

writs

24. Which of the following are themethods of Parliamentarycontrol over public finance inIndia?

1. Placing Annual Financial Statementbefore the Parliament.

2. Withdrawal of moneys fromConsolidated Fund of India only afterpassing the Appropriation Bill.

3. Provisions of supplementary grantsand vote-on account.

4. A periodic or at least a mid-yearreview of programmes of theGovernment against themacroeconomic forecasts andexpenditure by a ParliamentaryBudget Office.

5. Introducing Finance Bill in theParliament.

Select the correct answer using thecodes given below:(2012)

a) 1, 2, 3 and 5 onlyb) 1, 2 and 4 onlyc) 3, 4 and 5 onlyd) 1, 2, 3, 4 and 5

7.4 .2 Budget, Funds and Grants

25. With reference to the UnionGovernment, consider thefollowing statements:

1. The Department of Revenue isresponsible for the preparation ofUnion Budget that is presented to theParliament.

2. No amount can be withdrawn fromthe Consolidated Fund of Indiawithout the authorization from theParliament of India

3. All the disbursements made fromPublic Account also need theauthorization from the Parliament ofIndia.

Which of the statements given aboveis/are correct?

a) 1 and 2 onlyb) 2 and 3 onlyc) 2 onlyd) 1, 2 and 3

26. What is the difference between'vote-on-account' and 'interimbudget'?

1. The provision of a vote-on-account isused by a regular Government, whilean 'interim budget' is a provisionused by a caretaker Government.

2. A 'vote-on-account' only deals withthe expenditure in Governmentbudget, while an interim budgetincludes both expenditure andreceipts.

Which of the statements given aboveis/are correct?

a) 1 only

Page 54: PolityPreviousYearQuestionBank · 1979. 8. 31.  · PolityPreviousYearQuestionBank PageNo.5 Company can give licence of trade to individuals and its own employees to trade in India:

Polity Previous Year Question Bank

www.laex.in https://elearn.laex.inPage No. 51

b) 2 onlyc) Both 1 and 2d) Neither 1 nor 2

27. All revenues received by theUnion Government by way oftaxes and other receipts for theconduct of Government businessare credited to the

a) Contingency Fund of Indiab) Public Accountc) Consolidated Fund of Indiad) Deposits and Advances Fund

28. The authorization for thewithdrawal of funds from theConsolidated Fund of India mustcome from

a) The President of Indiab) The Parliament of Indiac) The Prime Minister of Indiad) The Union Finance Minister

29. When the annual Union Budget isnot passed by the Lok Sabha?

a) The Budget is modified andpresented again

b) The Budget is referred to the RajyaSabha for suggestions

c) The Union Finance Minister is askedto resign

d) The Prime Minister submits theresignation of Council of Ministers.

30. Which one of the following isresponsible for the preparationand presentation of union budgetto the parliament?

a) Department of Revenueb) Department of Economic Affairsc) Department of Expenditured) Department of Financial Affairs

31. With reference to Indian publicfinance, consider the followingstatements:

1. Disbursements from Public Accountsof India are subject to the Vote of theParliament.

2. The Indian Constitution provides forthe establishment of a ConsolidatedFund, a Public Account and aContingency Fund for each State.

3. Appropriations and disbursementsunder the Railway Budget are subjectto the same form of parliamentarycontrol as other appropriations anddisbursements.

Which of the statements given aboveare correct?

a) 1 and 2b) 2 and 3c) 1 and 3d) 1, 2 and 3

7.4.3 Committees

32. With reference to the Parliamentof India, which of the followingParliamentary Committeesscrutinizes and reports to theHouse whether the powers tomake regulations, rules, sub-rules, by-laws, etc. delegated bythe Parliament are beingproperly exercised by theExecutive within the scopeofsuch delegation?

a) Committee on GovernmentAssurances

b) Committee on SubordinateLegislation

c) Rules Committeed) Business Advisory Committee

33. Which one of the following is thelargest Committee of theParliament?

a) The Committee on Public Accountsb) The Committee on Estimatesc) The Committee on Public

Undertakingsd) The Committee on Petitions

Page 55: PolityPreviousYearQuestionBank · 1979. 8. 31.  · PolityPreviousYearQuestionBank PageNo.5 Company can give licence of trade to individuals and its own employees to trade in India:

Polity Previous Year Question Bank

www.laex.in https://elearn.laex.inPage No. 52

34. Consider the followingstatements about theparliamentary Committee onpublic accounts:

1. Consists of not more than 25Members of the Lok Sabha

2. Scrutinizes appropriation and financeaccounts of the Government

3. Examines the report of theComptroller and Auditor General ofIndia.

Which of the statements given aboveis/are correct?

a) 1 onlyb) 2 and 3 onlyc) 3 onlyd) 1, 2 and 3

35. Consider the followingstatements:

1. The Chairman of the Committee onPublic Accounts is appointed by theSpeaker of the Lok Sabha.

2. The Committee on Public Accountscomprises Members of Lok Sabha,Members of Rajya Sabha and feweminent persons of industry andtrade.

Which of the statements given aboveis/are correct?

a) 1 onlyb) 2 onlyc) Both 1 and 2d) Neither 1 nor 2

36. Consider the followingstatements:

1. While members of the Rajya Sabhaare associated with Committees onPublic Accounts and PublicUndertakings, members ofCommittee on Estimates are drawnentirely from the Lok Sabha.

2. The Ministry of ParliamentaryAffairs works under the overalldirection of Cabinet Committee onParliamentary Affairs.

3. The Minister of ParliamentaryAffairs nominates Members ofParliament on Committees, Councils,Board and Commissions etc. set upby the Government of India in thevarious ministries.

Which of these statements are correct?a) 1 and 2b) 2 and 3c) 2 and 3d) 1, 2 and 3

7.5 Law Making Process7.5.1 Comparison of Different Bills

37. Regarding Money Bill, which ofthe following statements is notcorrect?

a) A bill shall be deemed to be a MoneyBill if it contains only provisionsrelating to imposition, abolition,remission, alteration or regulation ofany tax.

b) A Money Bill has provisions for thecustody of the Consolidated Fund ofIndia or the Contingency Fund ofIndia.

c) A Money Bill is concerned with theappropriation of money out of theContingency Fund of India.

d) A Money Bill deals with theregulation of borrowing of money orgiving of any guarantee by theGovernment of India.

38. With reference to the Parliamentof India, consider the followingstatements:

1. A private member’s bill is a billpresented by a Member of Parliamentwho is not elected but onlynominatedby the President of India.

2. Recently, a private member’s bill hasbeen passed in the Parliament ofIndia for the first time in its history.

Which of the statements given aboveis/are correct?

Page 56: PolityPreviousYearQuestionBank · 1979. 8. 31.  · PolityPreviousYearQuestionBank PageNo.5 Company can give licence of trade to individuals and its own employees to trade in India:

Polity Previous Year Question Bank

www.laex.in https://elearn.laex.inPage No. 53

a) 1 onlyb) 2 onlyc) Both 1 and 2d) Neither 1 nor 2

39. Which of the followingstatements is/are correct?

1. A Bill pending in the Lok Sabhalapses on its prorogation.

2. A Bill pending in the Rajya Sabha,which was not been passed by theLok Sabha, shall not lapse ondissolution of the Lok Sabha.

Select the correct answer using thecode given below.

a) 1 onlyb) 2 onlyc) Both 1 and 2d) Neither 1 nor 2

40. What will follow if a Money Bill issubstantially amended by theRajya Sabha?

a) The Lok Sabha may still proceed withthe Bill, accepting or not acceptingthe recommendations of the RajyaSabha.

b) The Lok Sabha cannot consider thebill further.

c) The Lok Sabha may send the Bill tothe Rajya Sabha for reconsideration.

d) The President may call a joint sittingfor passing the Bill.

41. With reference to IndianParliament, which one of thefollowing is not correct? [2004]

a) The Appropriation Bill must bepassed by both the Houses ofParliament before it can be enactedinto law.

b) No money shall be withdrawn fromthe Consolidated Fund of Indiaexcept under the appropriation madeby the Appropriation Act.

c) Finance Bill is required for proposingnew taxes but no additional Bill/Actis required for making changes in therates of taxes which are alreadyunder operation.

d) No Money Bill can be introducedexcept on the recommendation of thePresident

42. Which one of the following Billsmust be passed by each House ofthe Indian Parliamentaryseparately by special majority?[2003]

a) Ordinary Billb) Money Billc) Finance Billd) Constitution Amendment Bill

43. Which of the followingstatements about money bill isnot correct?

a) A money bill can be tabled in eitherhouse of parliament.

b) The speaker of the Lok Sabha is thefinal authority to decide whether abill is money bill or not.

c) The Rajya Sabha must return amoney bill passed by the Lok Sabhaand send it for consideration within14 days.

d) The President cannot return a moneybill to the Lok Sabha forreconsideration.

7.6 Presiding Officers of the Houses.

44. Consider the following statements:

1. The Chairman and the DeputyChairman of the Rajya Sabha are notthe members of that House.

2. While the nominated members of thetwo Houses of the Parliament haveno voting right in the presidentialelection, they have the right to votein the election of the Vice President.

Page 57: PolityPreviousYearQuestionBank · 1979. 8. 31.  · PolityPreviousYearQuestionBank PageNo.5 Company can give licence of trade to individuals and its own employees to trade in India:

Polity Previous Year Question Bank

www.laex.in https://elearn.laex.inPage No. 54

Which of the statements given aboveis/are correct?

a) 1 onlyb) 2 onlyc) Both 1 and 2d) Neither 1 nor 2

45. Regarding the office of the LokSabha Speaker, consider thefollowing statements:

1. He/She holds the office during thepleasure of the President.

2. He/She need not be a member of theHouse at the time of his/her electionbut has to become a member of theHouse within six months from thedate of his/her election.

3. If he/she intends to resign, the letterof his/her resignation has to beaddressed to the Deputy Speaker.

Which of the statements given above is/are correct?

a) 1 and 2 onlyb) 3 onlyc) 1, 2 and 3d) Nonee)46. Consider the following

statements:1. The Speaker of Lok Sabha has the

power to adjourn the House sine diebut, on prorogation, it is only thePresident who can summon theHouse.

2. Unless sooner dissolved or there is anextension of the term, there is anautomatic dissolution of the LokSabha by efflux of time, at the end ofthe period of five years, even if noformal order ofdissolution is issuedby the President.

3. The Speaker of Lok Sabha continuesin office even after the dissolution ofthe House and until immediatelybefore the first meeting of the House.

Which of the statements given aboveare correct?

a) 1 and 2b) 2 and 3c) 1 and 3d) 1, 2 and 3

47. The Speaker can ask a member ofthe House to stop speaking andlet another member speak. Thisphenomenon is known as:

a) Decorumb) crossing the floorc) interpellationd) yielding the floor

7.7 State Legislature

48. With reference to the LegislativeAssembly of a State in India,consider the followingstatements:

1. The Governor makes a customaryaddress to Members of the House atthe commencement of the firstsession of the year.

2. When a State Legislature does nothave a rule on a particular matter, itfollows the Lok Sabha rule on thatmatter.

Which of the statements given aboveis/ are correct?(a) 1 only(b) 2 only(c) Both 1 and 2(d) Neither 1 nor 2

49. Consider the followingstatements with respect to theConstitution of India:

a) The Legislative Assembly of eachState shall consist of not more than450 members chosen by directelection from territorialconstituencies in the State.

b) A person shall not be qualified to bechosen to fill a seat in the Legislative

Page 58: PolityPreviousYearQuestionBank · 1979. 8. 31.  · PolityPreviousYearQuestionBank PageNo.5 Company can give licence of trade to individuals and its own employees to trade in India:

Polity Previous Year Question Bank

www.laex.in https://elearn.laex.inPage No. 55

Assembly of a State if he/ she is lessthan 25 years of age.

Which of the statements given aboveis/are correct?

a) 1 onlyb) 2 onlyc) Both 1 and 2d) Neither 1 nor 2

50. Consider the followingstatements:

1. The Speaker of the LegislativeAssembly Shall vacate his/her office ifhe/she ceases to be a member of theAssembly.

2. Whenever the Legislative Assemblyis dissolved, the Speaker shall vacatehis/her immediately.

Which of the statements given aboveis/are correct?

a) 1 onlyb) 2onlyc) Both 1 and 2d) Neither 1 nor 2

51. Consider the followingstatements:

1. The Legislative Council of a State inIndia can be larger in size than halfof the Legislative Assembly of thatparticular State.

2. The Governor of a State nominatesthe Chairman of Legislative Councilof that particular State.

Which of the statements given aboveis/are correct?

a) 1 onlyb) 2onlyc) Both 1 and 2d) Neither 1 nor 2

Key & Explanation

1. Answer: B

Explanation: Article 110 of theConstitution of India deals with money bills.Money bills deal with expenditure from theconsolidated fund of India therefore onlyLok Sabha is given exclusive power to enactthem. They can only be introduced in theLok Sabha. Rajya Sabha has no power toeither reject or amend a money bill. Itshould return the bill to the Lok Sabhawithin 14 days with or withoutrecommendations. Lok Sabha can eitheraccept or reject these recommendations.Since Annual Financial Statement is amoney bill, the Rajya Sabha can onlydiscuss it.The Rajya Sabha has no power tovote on the Demand for Grants since it ispart of budget (money bill) enactmentprocess.Article 117 of the Constitution of Indiadeals with financial bills. They can beintroduced only in the Lok Sabha but RajyaSabha has equal powers with the LokSabha in their enactment process.Educational Objective: Constitutionalposition of the Rajya Sabha with respect toLok Sabha is not equal in the case ofenactment of money bills.Source: Indian Polity by Laxmikant (4thEdition) – page no: 22.29

2. Answer: DExplanation: Lok Sabha is also known asHouse of People’s Representatives. Thecouncil of ministers are held accountable tothe public by people’s representativeselected to the Lok Sabha. According toArticle 75 of the constitution of India, theCouncil of Ministers are collectively (notindividually) responsible to the Lok Sabhaonly.Members of the both houses of theparliament are eligible to become Ministers.Even a non-member of the both houses canbecome a Minister for a period notexceeding 6 months. They have to become amember of the either house of theparliament to continue as Minister beyond6 months.Educational Objective: Article 75 -Prime Minister and his Council of

Page 59: PolityPreviousYearQuestionBank · 1979. 8. 31.  · PolityPreviousYearQuestionBank PageNo.5 Company can give licence of trade to individuals and its own employees to trade in India:

Polity Previous Year Question Bank

www.laex.in https://elearn.laex.inPage No. 56

Ministers are collectively responsible onlyto the Lok Sabha.Source: Indian Polity by Laxmikant (4thEdition) – page no: 20.3

3. Answer: AExplanation: According to Article 249 ofthe Constitution of India, only the RajyaSabha has the power to authorize theParliament to make a law on a subjectenumerated in the state list of the 7thSchedule. This exclusive power has beenconferred on the Rajya Sabha because of itsfederal character.The proclamation of Emergency underArticle 352 of the Indian Constitutionmust be approved by both the houses of theParliament. The 44th Constitutionalamendment act of 1978, has reduced theperiod of approval from 2 months to 1month. Once approved the emergencycontinues for 6 months. Approval of theparliament is necessary every 6 months.Educational Objective: Article 249,Article 352 and 44th Constitutionalamendment act,1978.Source: Indian Polity by Laxmikant (4thEdition) – page no: 22.29 and 16.2

4. Answer: CExplanation: Article 80 of the IndianConstitution deals with the composition ofthe Rajya Sabha. According to it, 12members are to be nominated by thePresident from those who have specialknowledge in art, literature, science andsocial service.Article 334 of the Indian Constitutiondeals with the provisions relating toreservation of seats for SC’s, ST’s and therepresentation of Anglo-Indian community.The President can nominate two membersfrom the Anglo-Indian community to theLok Sabha only, if they are not adequatelyrepresented.There is no bar on a nominated member ofthe Parliament from becoming a Minister.A nominated member of the Parliament canvote only in the Vice-Presidential electionsbut not in the Presidential elections.

Educational Objective: Nominatedmembers are present in both houses of theParliament and they can vote only in theVice-Presidential elections.Source: Indian Polity by Laxmikant (4thEdition) – page no: 22.2, 17.1 and 18.1

5. Answer: BExplanation: According to Article 249 ofthe Constitution of India, only the RajyaSabha has the power to authorize theParliament to make a law on a subjectenumerated in the state list of the 7thSchedule.Article 312 of the Indian Constitution saysthat only Rajya Sabha has the power toauthorize the creation of a new All-IndiaService.These two special powers have beenconferred only on the Rajya Sabha becauseof its federal character.Educational Objective: Rajya Sabha hasthe exclusive power to authorize legislationon a state subject and to authorize creationof a new All-India Service.Source: Indian Polity by Laxmikant (4thEdition) – page no: 22.29

6. Answer: AExplanation: Article 108 of theConstitution of India, the President can callfor the joint sitting of the both houses of theParliament only in the case of a deadlockwith an ordinary bill. A joint sitting is notallowed in case of a ConstitutionalAmendment Bill.In the case of a Money Bill, the situation ofa deadlock doesn’t arise because the RajyaSabha can only recommend amendments tothe money bill. The Lok Sabha can eitheraccept or reject them.Educational Objective: Joint sitting ofboth houses of the parliament only in thecase of ordinary bills.Source: Indian Polity by Laxmikant (4thEdition) – page no: 22.21

Page 60: PolityPreviousYearQuestionBank · 1979. 8. 31.  · PolityPreviousYearQuestionBank PageNo.5 Company can give licence of trade to individuals and its own employees to trade in India:

Polity Previous Year Question Bank

www.laex.in https://elearn.laex.inPage No. 57

Page 61: PolityPreviousYearQuestionBank · 1979. 8. 31.  · PolityPreviousYearQuestionBank PageNo.5 Company can give licence of trade to individuals and its own employees to trade in India:

Polity Previous Year Question Bank

www.laex.in https://elearn.laex.inPage No. 58

7. Answer: DExplanation: Article 108 of theConstitution of India, the President can callfor the joint sitting of the both houses of theParliament only in the case of a deadlockwith an ordinary bill. A joint sitting is notallowed in case of a ConstitutionalAmendment Bill.Since 1950, the provision regarding thejoint sitting of the both houses has beeninvoked only 3 times. The bills are

1. Dowry Prohibition Bill, 1960 (Jointsitting held on 6th May, 1961)

2. Banking Service Commission(Repeal) Bill, 1977 (Joint sittingheld on 16th May, 1978)

3. Prevention of Terrorism Bill,2002(Joint sitting held on 26th March,2002)

Educational Objective: Article 108 –Joint sitting of the both houses of theParliament.Source: Indian Polity by Laxmikant (4thEdition) – page no: 22.21 and 22.43Structure of Legislature Lok Sabha andRajya Sabha – Miscellaneous

8. Answer: DExplanation: According to Article 80 ofthe Constitution of India, the allocation ofseats in the Rajya Sabha to both therepresentatives of states and of unionterritories shall be in accordance with 4thSchedule. Only 2 union territories (Delhiand Puducherry) have representation in theRajya Sabha.Article 79 of the Constitution of India saysthat Parliament shall consist of thePresident, the Lok Sabha and the RajyaSabha. The President is a part of legislationmaking process.According to Article 103 of theConstitution of India, question on thedisqualification of a Member of Parliamentis referred to the President. The Presidentshall act according to the opinion of theElection Commission on such matters.Conditions for disqualification arementioned in Article 102 of the

Constitution of India and theRepresentation of People Act, 1951.The power to adjudicate election disputeslies with the respective state High Courtand the Supreme Court on appeal.Educational Objective: Article 79 –Parliament consists of the President, theLok Sabha and the Rajya Sabha.Union territories are represented in theRajya Sabha.The jurisdiction to adjudicate electiondisputes lies with the High Court and theSupreme court on appeal.Source: Indian Polity by Laxmikant (4thEdition) – page no: 22.5

9. Answer: CExplanation: During a NationalEmergency (Article 352), the life of the LokSabha can be extended beyond its normalterm (5years) by a law of the Parliament for1 year at a time (for any length of time).However, this extension cannot continuebeyond 6 months from the revocation ofNational Emergency.Educational Objective: Term of the LokSabha can be extended by 1 year at a timeby a law of the Parliament.Source: Indian Polity by Laxmikant (4thEdition) – Page No: 16.3

10. Answer: DExplanation: The Constitution of Indiahas adopted the system of proportionalrepresentation (single transferable votesystem) in the case of the Rajya Sabha orVidhan Parishad and for electing thePresident and Vice-President.However, in the case of the Lok Sabha orVidhan Sabha the system of territorialrepresentation (single member constituency)with first-past-the-post system is preferred.In this system the candidate with thehighest number of votes is declared as thewinner. This system was preferred becauseof its simplicity and stability in government.The Speaker is responsible for the smoothconduct of business in the Lok Sabha.There is no provision in the Constitution

Page 62: PolityPreviousYearQuestionBank · 1979. 8. 31.  · PolityPreviousYearQuestionBank PageNo.5 Company can give licence of trade to individuals and its own employees to trade in India:

Polity Previous Year Question Bank

www.laex.in https://elearn.laex.inPage No. 59

that the position of Speaker goes to themajority party and that the post of Deputy-Speaker goes to the opposition.Educational Objective: Proportionalrepresentation (single transferablevote system) – Elections of staterepresentatives in the Rajya Sabha orVidhan Parishad, the President and theVice-President.Territorial representation (singlemember constituency) with first-past-the-post system– Election of peoplerepresentatives to the Lok Sabha.Source: Indian Polity by Laxmikant (4thEdition) – Page No: 22.311. Answer: CExplanation: According to Article 84 ofthe Indian Constitution, only a citizen ofIndia is qualified to become a Member ofParliament.The Representation of People Act, 1951places an additional qualification that theperson contesting for a Lok Sabha seatshould be a registered voter in thatconstituency. However, there is no bar on aperson contesting to become a member ofRajya Sabha to be from the same state(from 2003 onwards).Educational Objective: Article 84 ofthe Constitution of India –Qualifications to become a MP.The Representation of People Act,1951–Additional Qualifications to becomean MP.Source: Indian Polity by Laxmikant (4thEdition) – Page No: 22.5

12. Answer: AExplanation: According to Article 102 ofthe Constitution of India, a person shall bedisqualified from being a member ofParliament if he/she holds any office ofprofit under the Government of India or ofany state. This provision was made to makethe legislature free from the influence ofthe executive. However, the Parliamentby law can exempt certain offices fromcoming under office of profit.The term office of profit is not well definedin the Constitution of India.

The Parliament (Prevention ofDisqualification) Act, 1959 was enactedto prevent disqualification of the membersof Parliament holding certain offices on thegrounds of Office of Profit. The offices thatwere exempted are

1. Minister, Minister of State orDeputy Minister for the Union or forany State.

2. Chief Whip, Deputy Chief Whip orWhip in Parliament or of aParliamentary Secretary, etc.

This law has been amended 5 times and thelast amendment was in 2013.Educational Objective: Article 102 ofthe Constitution of India –Disqualifications to become a MP.The Parliament (Prevention ofDisqualification) Act, 1959 – certainoffices exempted from coming under officeof profit.Source: Indian Polity by Laxmikant (4thEdition) – Page No: 22.5

13. Answer: BExplanation: Indian political system wasdominated by the Indian National CongressParty upto 1967. However, this changedwith the 4th general election. The need togive voice to the opposition has resulted inthe creation of the position of Leader ofOpposition in both the houses of theParliament. In 1969, the Leader ofOpposition was first officially recognizedand the statutory recognition was given in1977.The leader of the largest opposition partyhaving not less than one-tenth seats ofthe total strength of the house isrecognized as the leader of the opposition inthat house.The Communist Party of India with 16seats was the single largest party in theopposition in the first Lok Sabha.Educational Objective: Condition tobecome Leader of Opposition – not lessthan one-tenth seats of the total strength ofthe houseSource: Indian Polity by Laxmikant (4thEdition) – page no: 22.11

Page 63: PolityPreviousYearQuestionBank · 1979. 8. 31.  · PolityPreviousYearQuestionBank PageNo.5 Company can give licence of trade to individuals and its own employees to trade in India:

Polity Previous Year Question Bank

www.laex.in https://elearn.laex.inPage No. 60

14. Answer: AExplanation: Article 360 of the IndianConstitution empowers the President toproclaim a Financial Emergency if he issatisfied that a situation has arisen due towhich the stability or credit of India or anypart of its territory is threatened.A proclamation declaring FinancialEmergency must be approved by both thehouses of the Parliament within 2 monthsfrom the date of its declaration. Onceapproved by both the houses of Parliament,the Financial Emergency continuesindefinitely till it is revoked.During the Financial Emergency –

I. The executive authority of thecentre extends to direct states toobserve canons of financialpropriety.

II. The President may issuedirections for reduction ofsalaries and allowances of all orany class of persons servingeither the Union or any Stateand those of the judges of theSupreme Court and the HighCourt.

Educational Objective: Article 360 ofthe Constitution of India – FinancialEmergency.Source: Indian Polity by Laxmikant (4thEdition) – page no: 16.9 and 16.10

15. Answer: DExplanation: According to Article 249 ofthe Constitution of India, only the RajyaSabha has the power to authorize theParliament to make a law on a subjectenumerated in the state list of the 7thSchedule. This exclusive power has beenconferred on the Rajya Sabha because of itsfederal character. Such a resolution in theRajya Sabha must be supported by two-thirds of the members present andvoting. The resolution remains in effect for1 year and it can be renewed any number oftimes. The laws so made shall cease to haveeffect on the expiry of 6 months after theresolution has ceased to be in force.

Educational Objective: Only the RajyaSabha has the power to authorize theParliament to make a law on a state subject(resolution by a majority of two-thirds ofthe members present and voting).Source: Indian Polity by Laxmikant (4thEdition) – page no: 20.3

16. Answer: DExplanation: According to Article 253 ofthe Constitution of India, the Parliamentcan make laws on any matter in the statelist for implementing the internationaltreaties, agreements or conventions. Thisprovision enables the union government tofulfil its international obligations. Consentof the states is not necessary.Educational Objective: Article 253 ofthe Constitution of India – Legislationgiving effect to international agreements.Source: Indian Polity by Laxmikant (4thEdition) – page no: 14.3

17. Answer: BExplanation: Article 124 of theConstitution of India enables theParliament to decide the number of judgesin the Supreme Court. Therefore, the powerto increase the number of judges in theSupreme Court vests with the Parliament.Educational Objective: Article 124 ofthe Constitution of India – Establishmentand constitution of the Supreme Court.Source: Indian Polity by Laxmikant (4thEdition) – Page No: 25.1

18. Answer: AExplanation: According to Article 31B ofthe Indian Constitution, acts andregulations included in the Ninth Scheduleare saved from being challenged andinvalidated on the contravention of any ofthe fundamental rights. However, theSupreme Court in 2007 adjudged that therecannot be any blanket immunity fromjudicial review of laws included in theNinth Schedule. It also said those laws thatwere placed in the Ninth Schedule after24th April, 1973 (Kesavananda Bharati

Page 64: PolityPreviousYearQuestionBank · 1979. 8. 31.  · PolityPreviousYearQuestionBank PageNo.5 Company can give licence of trade to individuals and its own employees to trade in India:

Polity Previous Year Question Bank

www.laex.in https://elearn.laex.inPage No. 61

judgment) are open to challenge in court ifthey violate fundamental rights.Some of the acts included in the 9thSchedule deal with land reforms, abolitionof zamindari system and reservation tobackward classes exceeding the 50% limitset by the Supreme Court.Educational Objective: Article 31B ofthe Indian Constitution – Validation of actsand regulations included in the 9thSchedule.Judicial review is allowed for laws includedin the 9th Schedule after 24th April, 1973.Source: Indian Polity by Laxmikant (4thEdition) – Page No: 7.2219. Answer: D

Explanation: The executive is heldaccountable to the public by the Parliament.Various means are available to themembers of the Parliament to achieve thispurpose.Some of them are - Question Hour,Zero Hour, Calling Attention Motion,Adjournment Motion, No-ConfidenceMotion, etc.Question Hour – The first hour of everyparliamentary sitting is called as QuestionHour. During this, the members can gettheir questions answered by the ministers.The questions can be of three types-i. Starred question – this question

requires the minister to answer thequestion orally and the member canask supplementary questions if thequestion was not answered by theminister to their satisfaction.

ii. Unstarred question – this questionrequires a written reply from theminister. So, supplementary questionsare not allowed for this type ofquestions.

iii. Short notice question – this type ofquestions can be asked by giving anotice of less than 10 days and theminister has to answer orally.

Adjournment Motion– this motion isintroduced to draw attention of the house toa definite matter of urgent publicimportance. This needs the support of50 members to be accepted in the house. It

is considered as an extraordinary device asit disrupts the normal business of the house.Since it involves an element of censureagainst the government, this motioncannot be introduced in the RajyaSabha.Educational Objective: Questionhour – 1st hour of a parliamentary sittingAdjournment Motion – It involvescensure, so it can’t be introduced in theRajya Sabha.Source: Indian Polity by Laxmikant (4thEdition) – page no: 22.13 and 22.15

20. Answer: CExplanation: Article 75 of theconstitution of India says that the council ofministers shall be collectivelyresponsible to the Lok Sabha. Thismeans that the ministers can stay in officeas long as they enjoy majority in the LokSabha.A No-Confidence Motion is introduced toascertain the confidence in the council ofministers. If it is passed in the Lok Sabhathen the council of ministers have to resignfrom their office. This motion needs thesupport of 50 members to be admittedand it can only be introduced in theLok Sabha. It is not necessary to statereasons for its adoption. There is nomention of this motion in the constitution.It is a parliamentary convention and ismentioned in Rule 198 of the Lok Sabha.Educational Objective: No-ConfidenceMotion – moved against the council ofministers and not against an individualminister. Can be introduced only in the LokSabha and needs support of 50 members.Source: Indian Polity by Laxmikant (4thEdition) – page no: 22.15

21. Answer: AExplanation: Adjournment Motion–this motion is introduced to draw attentionof the house to a definite matter ofurgent public importance. This needsthe support of 50 members to beaccepted in the house. It is considered as anextraordinary device as it disrupts the

Page 65: PolityPreviousYearQuestionBank · 1979. 8. 31.  · PolityPreviousYearQuestionBank PageNo.5 Company can give licence of trade to individuals and its own employees to trade in India:

Polity Previous Year Question Bank

www.laex.in https://elearn.laex.inPage No. 62

normal business of the house. Since itinvolves an element of censure against thegovernment, this motion cannot beintroduced in the Rajya Sabha.The following restrictions are placed on theadoption of this motion –i. raise a matter that is recent, definite,

factual, urgent and of public importance.ii. should not cover more than one matter.iii. should not raise a question of privilege.iv. should not raise a question on a matter

that has been already discussed in thesame session.

v. Should not deal with any matter that isunder adjudication by court.

Educational Objective: AdjournmentMotion – a definite matter of urgent publicimportance.Source: Indian Polity by Laxmikant (4thEdition) – page no: 22.1522. Answer: AExplanation: Article 75 of theconstitution of India says that the council ofministers shall be collectively responsible tothe Lok Sabha. This means that theministers can stay in office as long as theyenjoy majority in the Lok Sabha.A No-Confidence Motion is introduced toascertain the confidence in the council ofministers. If it is passed in the Lok Sabhathen the council of ministers have to resignfrom their office. This motion needs thesupport of 50 members to be admittedand it can only be introduced in theLok Sabha. It is not necessary to statereasons for its adoption. There is nomention of this motion in the constitution.It is a parliamentary convention and ismentioned in the Rule 198 of the LokSabha.Procedure - Once a notice is given for aNo-Confidence Motion, the Speaker reads itin the house and asks for those who are infavour of it. In case there are 50 MPs infavour, a date for discussing the motion isassigned and this date should be within 10days from the admission of the motion.Educational Objective: No-ConfidenceMotion – Can be introduced only in theLok Sabha and needs support of 50

members. Discussion on the motion shouldtake place within 10 days.Source: Indian Polity by Laxmikant (4thEdition) – page no: 22.15

23. Answer: AExplanation: The Constitution of India ofestablished a parliamentary form ofgovernment in which the Executive isresponsible to the Parliament for itspolicies and acts. The Parliament exercisescontrol over the Executive through questionhour, zero hour, calling attention motion,etc. It also supervises the work of theExecutive with the help of its committees.Some of the parliamentary committeesare – Public Accounts Committee,Estimates Committee, Committee onSubordinate Legislation, etc.Educational Objective: ParliamentaryCommittees – exercise control over theExecutive.Source: Indian Polity by Laxmikant

(4thEdition) – page no: 22.26

24. Answer: AExplanation: According to Article 112,the president shall cause to be laid anAnnual Financial Statement (Budget)before both the houses of the Parliament.The Constitution contains the followingprovisions with regard to enactment ofbudget –i. No demand for a grant shall be made

except on the recommendation of thePresident.

ii. No money shall be withdrawn from theconsolidated fund of India except underappropriation made by law.

The Budget includes the following- Vote on account – it is the interim

permission of the Parliament to thegovernment to spend money (usuallyaround 1/6thof the total expenditure).Previously Budget used to be presentedon 31st March. Since the financial yearstarts from 1st April, the governmentneeds money to run the administrationbefore the budget can be passed.Therefore, a Vote on Account is passed.

Page 66: PolityPreviousYearQuestionBank · 1979. 8. 31.  · PolityPreviousYearQuestionBank PageNo.5 Company can give licence of trade to individuals and its own employees to trade in India:

Polity Previous Year Question Bank

www.laex.in https://elearn.laex.inPage No. 63

But it is not necessary now because theintroduction of the budget is preponedto 28th February.

Appropriation Bill – this bill has tobe passed by the Parliament for theGovernment to withdraw money fromthe Consolidated Fund of India.

Finance Bill – this bill deals with howthe Government plans to raise themoney to meet its expenditure. This billhas to be passed by the Parliament forthe Government to raise money throughtaxes and other means.

Supplementary Grant –it is granted bythe Parliament when money previouslyauthorized through the Appropriation Actfor a particular service is not sufficient. TheGovernment has no authority to utilize themoney appropriated for service for anotherpurpose without parliamentary approval.Review of the Government policies,programmes and expenditure aremonitored through parliamentarycommittees not through parliamentarybudget office.Educational Objective: AnnualFinancial Statement (Budget) –parliamentary control over public finance.Source: Indian Polity by Laxmikant (4thEdition) – page no: 20.3

25. Answer: CExplanation: The Budget is prepared bythe Budget Division in the Department ofEconomic Affairs, Ministry of Finance,Government of India. The FinanceMinister is the head of the Budget makingCommittee.Types of funds –i. Consolidated Fund of

India – this is the chief account ofthe Government of India. The inflowto this fund is by way of taxes andalso non-tax revenues like loans,issue of treasury bills, etc. TheGovernment meets all itsexpenditure including loanrepayments through this fund.Withdrawal of funds from this fundrequires authorization of theParliament (Appropriation Act).

This fund is constituted under theprovision of Article 266(1) of theConstitution of India. Every statehas its own Consolidated Fund.

ii. Public Account of India –all other monies (other than thosecovered under the ConsolidatedFund of India) received by or onbehalf of the Government of Indiaare credited to the Public Account ofIndia. For example, money fromsmall saving scheme, pensionschemes, etc. are credited to thisaccount. Parliamentaryauthorization for paymentsfrom this account is notrequired. Public Account of Indiais constituted under Article 266(2)of the Constitution of India. Everystate has its own Public Account.

iii. Contingency Fund ofIndia – this fund is created underthe provision of Article 267(1) ofthe constitution of India. Thecorpus of this fund is Rs. 500 crores.Advances from this fund are madeto meet unforeseen expenditure (forexample expenditure due to naturalcalamities) by the President ofIndia. This amount is replenishedas soon as the Parliamentauthorizes additional expenditure.The Secretary, Department ofEconomic Affairs, Ministry ofFinance in the Government ofIndia holds the fund on the behalf ofthe President.

Educational Objective: Budget preparedby Department of Economic Affairs,Ministry of Finance, Government of India.Consolidated Fund of India - Withdrawal offunds from this fund requires authorizationof the Parliament.Public Account of India - Parliamentaryauthorization for payments from thisaccount is not required.Contingency Fund of India – the Secretary,Department of Economic Affairs, Ministryof Finance in the Government of Indiaholds the fund on the behalf of thePresident of India.

Page 67: PolityPreviousYearQuestionBank · 1979. 8. 31.  · PolityPreviousYearQuestionBank PageNo.5 Company can give licence of trade to individuals and its own employees to trade in India:

Polity Previous Year Question Bank

www.laex.in https://elearn.laex.inPage No. 64

Source: Indian Polity by Laxmikant (4thEdition) – page no: 22.25

26. Answer: CExplanation: Vote on Account is a grantin advance to enable the government tocarry on until the voting of demands forgrants and the passing of the AppropriationBill and Finance Bill.This enables thegovernment to fund its expenses for a shortperiod of time or until a full-budget ispassed. As a convention, a vote-on-accountis treated as a formal matter and passed byLok Sabha without discussion.An Interim Budget in all practical senseis a full budget but made by thegovernment during the last year of itsterm – i.e. just before the election(caretaker Government). But it may notcontain big policy proposals.While a Vote-on-Account only dealswith the expenditure side of thegovernment's budget, an InterimBudget is a complete set of accounts,including both expenditure andreceipts.Educational Objective: Vote-on-Account – only deals with the expenditureside of the government's budget.Interim Budget- is a complete set ofaccounts, including both expenditure andreceipts and usually without any big policyproposals.Source: Indian Polity by Laxmikant (4thEdition) – page no: 20.3

27. Answer: CExplanation: Types of funds –i. Consolidated Fund of India – this is

the chief account of the Government ofIndia. All revenues received by theUnion Government by way of taxesand also non-tax revenues likeloans, issue of treasury bills, etc.for the conduct of governmentbusiness are credited to this fund.The Government meets all itsexpenditure including loan repaymentsthrough this fund. Withdrawal of fundsfrom this fund requires authorization of

the Parliament(Appropriation Act). Thisfund is constituted under the provisionof Article 266(1) of the Constitution ofIndia. Every state has its ownConsolidated Fund.

ii. Public Account of India – all othermonies (other than those covered underthe Consolidated Fund of India)received by or on behalf of theGovernment of India are credited to thePublic Account of India. For example,money from small saving scheme,pension schemes, etc. are credited tothis account. Parliamentaryauthorization for payments from thisaccount is not required. Public Accountof India is constituted under Article266(2) of the Constitution of India.Every state has its own Public Account.

iii. Contingency Fund of India – thisfund is created under the provision ofArticle 267(1) of the constitution ofIndia. The corpus of this fund is Rs.500 crores. Advances from this fund aremade to meet unforeseen expenditure(for example expenditure due to naturalcalamities) by the President of India.This amount is replenished as soon asthe Parliament authorizes additionalexpenditure.

Educational Objective: ConsolidatedFund of India - All revenues received bythe Union Government for the conduct ofgovernment business are credited to thisfund.Source: Indian Polity by Laxmikant (4thEdition) – page no: 22.25

28. Answer: BExplanation: Consolidated Fund ofIndia – this is the chief account of theGovernment of India. All revenues receivedby the Union Government by way of taxesand also non-tax revenues like loans, issueof treasury bills, etc. for the conduct ofgovernment business are credited to thisfund. The Government meets all itsexpenditure including loan repaymentsthrough this fund. Withdrawal of fundsfrom this fund requires authorizationof the Parliament (Appropriation Act).This fund is constituted under the provision

Page 68: PolityPreviousYearQuestionBank · 1979. 8. 31.  · PolityPreviousYearQuestionBank PageNo.5 Company can give licence of trade to individuals and its own employees to trade in India:

Polity Previous Year Question Bank

www.laex.in https://elearn.laex.inPage No. 65

of Article 266(1) of the Constitution ofIndia. Every state has its own ConsolidatedFund.

Educational Objective:Consolidated Fund of India -Withdrawal of funds from this fundrequires authorization of theParliament.Source: Indian Polity by Laxmikant(4th Edition) – page no: 22.25

29. Answer: DExplanation: The Constitution of India ofestablished a parliamentary form ofgovernment in which the Executive isresponsible to the Parliament for itspolicies and acts.Article 75 of theconstitution of India says that the council ofministers shall be collectivelyresponsible to the Lok Sabha. Thismeans that the ministers can stay in officeas long as they enjoy majority in the LokSabha.The Lok Sabha can express lack ofconfidence in the council of ministers in thefollowing ways –

i. By not passing a motion ofthanks.

ii. By rejecting a money bill.iii. By passing a censure or an

adjournment motion.iv. By passing a cut motion.

Therefore, when the Union Budget ispassed by the parliament it shows the lossof confidence in the council of ministers. Asa result, the Prime Minister and his councilof ministers submit their resignation.Educational Objective: Non passage ofthe union budget leads to the resignation ofthe Prime Minister and his Council ofMinisters.Source: Indian Polity by Laxmikant (4thEdition) – page no: 22.2630. Answer: BExplanation: The Budget is prepared bythe Budget Division in the Department ofEconomic Affairs, Ministry of Finance,Government of India. The FinanceMinister is the head of the Budget making

Committee and presents the Budget in theParliament.Educational Objective: Union Budgetprepared by Department of EconomicAffairs, Ministry of Finance, Government ofIndia.Source: Indian Polity by Laxmikant (5thEdition).

31. Answer: BExplanation: Types of funds –

i. Consolidated Fund of India – this isthe chief account of the Government ofIndia. The inflow to this fund is by wayof taxes and also non-tax revenues likeloans, issue of treasury bills, etc. TheGovernment meets all its expenditureincluding loan repayments through thisfund. Withdrawal of funds from thisfund requires authorization of theParliament (Appropriation Act). Thisfund is constituted under the provisionof Article 266(1) of the Constitution ofIndia. Every state has its ownConsolidated Fund.

ii. Public Account of India – all othermonies (other than those covered underthe Consolidated Fund of India)received by or on behalf of theGovernment of India are credited to thePublic Account of India. For example,money from small saving scheme,pension schemes, etc. are credited tothis account. Parliamentaryauthorization for payments fromthis account is not required. PublicAccount of India is constituted underArticle 266(2) of the Constitution ofIndia. Every state has its ownPublic Account.

iii. Contingency Fund of India – thisfund is created under the provision ofArticle 267(1) of the constitution ofIndia. The corpus of this fund is Rs.500 crores. Advances from this fund aremade to meet unforeseen expenditure(for example expenditure due to naturalcalamities) by the President of India.This amount is replenished as soon asthe Parliament authorizes additionalexpenditure. The Secretary,Department of Economic Affairs,

Page 69: PolityPreviousYearQuestionBank · 1979. 8. 31.  · PolityPreviousYearQuestionBank PageNo.5 Company can give licence of trade to individuals and its own employees to trade in India:

Polity Previous Year Question Bank

www.laex.in https://elearn.laex.inPage No. 66

Ministry of Finance in theGovernment of India holds the fund onthe behalf of the President. Everystate has its own Contingency Fund.

The Government of India used to have twobudgets – the General Budget and theRailway Budget. The Railway Budget wasseparated from the General Budget in 1921on the recommendations of the AcworthCommittee. However, from the financialyear 2017-18 onwards both the budgetswere merged and every year only oneGeneral Budget is presented.Appropriations and disbursements underthe Railway Budget were used to besubjected to the same form ofparliamentary control as appropriationsand disbursements under the GeneralBudget.Educational Objective: Both the Unionand the States have their own ConsolidatedFund, Public Account and Contingencyfund.Source: Indian Polity by Laxmikant (4thEdition) – page no: 22.25

32. Answer: BExplanation: The Constitution of India ofestablished a parliamentary form ofgovernment in which the Executive isresponsible to the Parliament for itspolicies and acts. Itsupervises the work ofthe Executive with the help of itscommittees. Some of the parliamentarycommittees are – Public AccountsCommittee, Estimates Committee,Committee on Subordinate Legislation, etc.Committee on GovernmentAssurances – each house of theParliament has a committee onGovernment Assurances which examinesthe assurances, promises and undertakingsgiven by the ministers on the floor of thehouse and reports to the house on theextent of their implementation. The LokSabha Committee on Assurances has 15members and Rajya Sabha Committee onAssurances has 10 members.Committee on SubordinateLegislation – each house of the parliamenthas a Committee on subordinate legislationwhose main function is to examine and

report to the housewhether the powers tomake regulations, rules, sub-rules, by-lawsetc. delegated by the Parliament are beingproperly exercised by it.This committee has15 members each in both the Rajya Sabhaand the Lok Sabha.Rules Committee – each house of theParliament has a rules committee whichconsiders the issues related to procedureand conduct of the business in that houseand suggest necessary amendments oradditions to the rules of the house. The LokSabha committee has 15 membersincluding the Speaker as its ex-officiochairman. Rajya Sabha committee has 16members including the Vice-President asits ex-officio Chairman.Business Advisory Committee - eachhouse of the Parliament has a businessadvisory committee which allocates time forlegislations and other business broughtbefore the house by the Government. TheLok Sabha committee has 15 membersincluding the Speaker as its ex-officiochairman. Rajya Sabha committee has 11members including the Vice-President asits ex-officio Chairman.Educational Objective: Committee onSubordinate Legislation – deals withdelegated legislation.Source: Indian Polity by Laxmikant (4thEdition) – page no: 20.3

33. Answer: BExplanation: The Committee on PublicAccounts – this committee was first setupin 1921 under the provisions ofGovernment of India Act, 1919. Thefunction of this committee is to examine theannual audit reports submitted by theComptroller and Auditor General of India(CAG). It looks into the prudence, wisdomand proprietary of public expenditure andto bring out the cases of waste, loss,corruption, extravagance and inefficiency.The committee is assisted by CAG infulfilment of its functions. In fact, CAG actsas a guide, friend and philosopher of thecommittee. At present, the committeeconsists of 22 members (Lok Sabha – 15and Rajya Sabha – 7). The members areelected every year through proportional

Page 70: PolityPreviousYearQuestionBank · 1979. 8. 31.  · PolityPreviousYearQuestionBank PageNo.5 Company can give licence of trade to individuals and its own employees to trade in India:

Polity Previous Year Question Bank

www.laex.in https://elearn.laex.inPage No. 67

representation by means of singletransferable vote. The Chairman of thecommittee is appointed by the Speaker.Since 1967 chairman of the committee isselected from among the members of theOpposition as a convention.The Committee on Estimates – theorigin of this committee can be traced to theStanding Financial Committee set up in1921. The first Estimates Committee wassetup in 1950 on the recommendation ofJohn Matai, the then finance minister. Thefunction of this committee is to examine theestimates included in the budget andsuggest economies in public expenditure. Atpresent the committee consists of 30members (all 30 from Lok Sabhaitself).The members are elected every yearthrough proportional representation bymeans of single transferable vote. Thechairman is appointed by the speaker andhe is invariably from the ruling party.The Committee on PublicUndertakings – this committee wascreated in 1964 on the recommendation ofthe Krishna Menon Committee. Thefunction of this committee is to examine thereports of CAG on public undertakings andto examine whether the publicundertakings are working efficiently or not.At present, the committee consists of 22members (Lok Sabha – 15 and RajyaSabha – 7). The members are elected everyyear through proportional representationby means of single transferable vote. TheChairman of the committee is appointed bythe Speaker from amongst the members ofthe Lok Sabha.The Committee on Petitions – eachhouse of the Parliament has its ownCommittee on petitions. It examinespetitions on bills and on matters of generalpublic importance. The Lok SabhaCommittee consists of 15 Members,while the Rajya Sabha Committeeconsists of 10 members.Educational Objective: The term of theparliamentary committees is 1 year and themembers are elected every yearthroughproportional representation bymeans of single transferable vote.Source: Indian Polity by Laxmikant (4thEdition) – page no: 23.2-4 and 23.7

34. Answer: BExplanation: The Committee on PublicAccounts – this committee was first setupin 1921 under the provisions ofGovernment of India Act, 1919. Thefunction of this committee is to examinethe annual audit reports submitted bythe Comptroller and Auditor Generalof India (CAG). It looks into the prudence,wisdom and proprietary of publicexpenditure and to bring out the cases ofwaste, loss, corruption, extravagance andinefficiency. The committee is assisted byCAG in fulfilment of its functions. In fact,CAG acts as a guide, friend and philosopherof the committee. At present, the committeeconsists of 22 members (Lok Sabha – 15and Rajya Sabha – 7). The members areelected every year through proportionalrepresentation by means of singletransferable vote. The Chairman of thecommittee is appointed by the Speaker.Since 1967 chairman of the committee isselected from among the members of theOpposition as a convention.The committee looks into theappropriation accounts and thefinance accounts of the UnionGovernment. Appropriation accountscompare the actual expenditure with theexpenditure sanctioned by the Parliamentthrough the Appropriation Act. The financeaccounts show the annual receipts anddisbarments of the Union Government.Educational Objective: The Committeeon Public Accounts – examines CAGreports, appropriation accounts and financeaccounts of the Union Government and alsolooks into the prudence and wisdom of thepublic expenditure.Source: Indian Polity by Laxmikant (4thEdition) – page no: 23.2

35. Answer: AExplanation: The Committee on PublicAccounts – this committee was first setupin 1921 under the provisions ofGovernment of India Act, 1919. Thefunction of this committee is to examinethe annual audit reports submitted bythe Comptroller and Auditor General

Page 71: PolityPreviousYearQuestionBank · 1979. 8. 31.  · PolityPreviousYearQuestionBank PageNo.5 Company can give licence of trade to individuals and its own employees to trade in India:

Polity Previous Year Question Bank

www.laex.in https://elearn.laex.inPage No. 68

of India (CAG). It looks into the prudence,wisdom and proprietary of publicexpenditure and to bring out the cases ofwaste, loss, corruption, extravagance andinefficiency. The committee is assisted byCAG in fulfilment of its functions. In fact,CAG acts as a guide, friend and philosopherof the committee. At present, the committeeconsists of 22 members (Lok Sabha – 15and Rajya Sabha – 7). The members areelected every year through proportionalrepresentation by means of singletransferable vote. The Chairman of thecommittee is appointed by the Speaker.Since 1967 chairman of the committee isselected from among the members of theOpposition as a convention. The committeecan consult eminent persons of industryand trade but they are not allowed tobecome members of the committee.Educational Objective: The Committeeon Public Accounts has 22 members (LokSabha – 15 and Rajya Sabha – 7) and theChairman is appointed by the Speaker.Source: Indian Polity by Laxmikant (4thEdition) – page no: 23.2

36. Answer: AExplanation:

Committees onPublic Accounts

22 members (LokSabha-15 andRajya Sabha-7)

Committee onPublicUndertakings

22 members (LokSabha-15 andRajya Sabha-7)

Committee onEstimates

30 members (LokSabha-30)

Cabinet is the highest decision-makingbody in the Government. CabinetCommittees are setup to assist the Cabinet.The decisions taken by the cabinet arebinding on the individual ministers(Collective responsibility of the council ofministers to the Parliament). The CabinetCommittee on Parliamentary Affairs looksafter the progress of government businessin the Parliament. This committee isheaded by the Home Minister, Governmentof India. Therefore, we can say that theMinistry of Parliamentary Affairs works

under the overall direction of CabinetCommittee on Parliamentary Affairs.Consultative Committees – thesecommittees are attached to variousministries of the Union Government. TheMinister/ Minister of State in charge of theconcerned ministry acts as the Chairman ofthe Consultative committee of thatministry. These committees provide aforum for informal discussions between theMinisters and the MPs on policies andprogrammes of the government and theirimplementation. These committees areconstituted by the Ministry ofParliamentary Affairs.The Minister of Parliamentary Affairs doesnot nominate MPsto all the Committees,Councils, Board and Commissions etc. setup by the Government of India in thevarious ministries.Educational Objective: ConsultativeCommittees are constituted by the Ministryof Parliamentary Affairs.Source: Indian Polity by Laxmikant (4thEdition) – page no: 20.3

37. Answer: CExplanation: Article 110 of theConstitution of India deals with thedefinition of money bills. It states that abill is deemed to be a money bill if itcontains provisions dealing with thefollowing matters only –i. Imposition, abolition, remission,

alteration or regulation of any tax.ii. Regulation of the borrowing of money or

giving of any guarantee by theGovernment of India.

iii. Custody of the Consolidated Fund ofIndia or the Contingency Fund of Indiaand payment or withdrawal of moneyfrom these funds.

iv. Appropriation of money out of theConsolidated Fund of India.

v. Declaration of charged expenditure onthe Consolidated Fund of India orincreasing such expenditure.

vi. Receipt of money on the account of theConsolidated Fund of India or the

Page 72: PolityPreviousYearQuestionBank · 1979. 8. 31.  · PolityPreviousYearQuestionBank PageNo.5 Company can give licence of trade to individuals and its own employees to trade in India:

Polity Previous Year Question Bank

www.laex.in https://elearn.laex.inPage No. 69

Public Account of India and the audit ofthe accounts of the Union or of a State.

vii. Any matter incidental to any of theabove matters.

Contingency Fund of India – this fundis created under the provision of Article267(1) of the constitution of India. Thecorpus of this fund is Rs. 500 crores.Advances from this fund are made to meetunforeseen expenditure (for exampleexpenditure due to natural calamities) bythe President of India. This amount isreplenished as soon as the Parliamentauthorizes additional expenditure.Therefore, a money bill is not concernedwith appropriation of money out of theContingency Fund of India but it isconcerned with replenishment of theContingency Fund of India.Educational Objective: Article 110 ofthe Constitution of India deals with MoneyBills.Source: Indian Polity by Laxmikant (4thEdition) – page no: 22.18

38. Answer: DExplanation: Private Member’s Bill – isa bill introduced by any member of theParliament other than a Minister. Itgenerally reflects the stand of theopposition on public matters. Drafting ofthe bill is the responsibility of theconcerned member. It has a lesser chancebecome an act. Its introduction requires atleast a notice of 1 month. They can bediscussed only on Fridays.Up to 1997, private members couldintroduce up to three Bills in a week. Thisled to a piling up of Bills that wereintroduced but never discussed; ChairmanK R Narayanan, therefore, capped thenumber of private member’s Bills to threeper session.Fourteen Private Member’s Bills —five of which were introduced in RajyaSabha — have become law so far.Educational Objective: Private Member’sBill – introduced by any MP other than aMinister and 14 such bills have become lawtill now.

Source: Indian Polity by Laxmikant (4thEdition) – page no: 22.17

39. Answer: BExplanation: Article 107 of theConstitution of Indiadeals with certainsituations under which bills lapse in IndianParliament. The position of the bills ineither house of the parliament- Lok Sabha& Rajya Sabha varies with respect to thecircumstances in the parliament. Thedissolution of Lok Sabha is followed bylapsing of all business including bills,motions, resolutions, notices, petitions andso on pending before it or its committees.However, there are a few bills which do notlapse even on its dissolution.The table below explains which bills inwhat position lapse when Lok Sabhadissolves:

Sl.No

Position of theBill

Lapse ofthe Bill

1 A bill pending inthe Lok Sabha

Lapses

2 A bill passed by theLok Sabha butpending in theRajya Sabha

Lapses

3 A bill not passed bythe two Houses dueto disagreement andif the president hasnotified the holdingof a joint sittingbefore thedissolution of LokSabha

Does notLapse

4 A bill pending in theRajya Sabha butnot passed bythe Lok Sabha

Does notLapse

Page 73: PolityPreviousYearQuestionBank · 1979. 8. 31.  · PolityPreviousYearQuestionBank PageNo.5 Company can give licence of trade to individuals and its own employees to trade in India:

Polity Previous Year Question Bank

www.laex.in https://elearn.laex.inPage No. 70

5 A bill passed by bothHouses butpending assentof the president

Does notLapse

6 A bill passed by bothHouses butreturned by thepresident forreconsiderationof Houses

Does notLapse

Prorogation of the parliamentarysession by the President terminatesthat particular session of theParliament. Prorogation does notaffect any pending bills or anyother business pending before theHouse. However, all pending notices(other than those for introducing bills)lapse on prorogation.Educational Objective: Prorogationdoes not have affecton the pending billsor any other business pending beforethe House.The dissolution of Lok Sabha is followedby lapsing of all pending bills, except afew bills that do not lapse -i. A bill not passed by the two

Houses due to disagreement andif the president has notified theholding of a joint sitting beforethe dissolution of Lok Sabha.

ii. A bill pending in the RajyaSabha but not passed by the LokSabha.

iii. A bill passed by both Houses butpending assent of the president.

iv. A bill passed by both Houses butreturned by the president forreconsideration of Houses.

Source: Indian Polity by Laxmikant (4thEdition) – not available.

40. Answer: AExplanation: Money Bill is defined inArticle 110 of the Indian Constitution.Money bills are concerned with financial

matters like taxation, public expenditure,etc.A Money Bill may only be introduced inLok Sabha, on the recommendation of thePresident. It must be passed in Lok Sabhaby a simple majority of all members presentand voting. Following this, it will be sent tothe Rajya Sabha for its recommendations,which Lok Sabha may reject if it chooses to.If such recommendations are not givenwithin 14 days, it will be deemed to bepassed by Parliament.The Lok Sabha hasmore powers than the Rajya Sabha in caseof money bills only.When a money bill is presented to thepresident, he may give his assent to the billor withhold his assent to the bill but hecannot return the bill for reconsideration(money bill introduced with the priorpermission of the President).The provision of a joint sitting is notnecessary in case of a money bill since thereis no possibility for a deadlock situationbetween the Lok Sabha and the RajyaSabha.Educational Objective: Article 110 dealswith money bills.The Rajya Sabha has restricted powerswith regard to money bills.Source: Indian Polity by Laxmikant (4thEdition) – page no: 22.19

41. Answer: AExplanation: Appropriation Bill – thisbill has to be passed by the Parliament forthe Government to withdraw money fromthe Consolidated Fund of India.TheConstitution of India says that no moneyshall be withdrawn from the ConsolidatedFund of India except under theappropriation made by the AppropriationAct. Since Appropriation Bill is a money billit is not required to be passed by both theLok Sabha and the Rajya Sabha.Finance Bill – this bill deals with how theGovernment plans to raise the money tomeet its expenditure. This bill has to bepassed by the Parliament for theGovernment to raise money through taxesand other means.The proposals of thegovernment for levy of new taxes,

Page 74: PolityPreviousYearQuestionBank · 1979. 8. 31.  · PolityPreviousYearQuestionBank PageNo.5 Company can give licence of trade to individuals and its own employees to trade in India:

Polity Previous Year Question Bank

www.laex.in https://elearn.laex.inPage No. 71

modification of the existing tax structure orcontinuance of the existing tax structurebeyond the period approved by Parliamentare submitted to Parliament through theFinance bill.The Finance Bill can beintroduced only in Lok Sabha (since it is amoney bill) and the Rajya Sabha can onlyrecommend amendments in the bill. Thebill has to be passed by the Parliamentwithin 75 days of its introduction.A Money Bill is defined in Article 110 ofthe Indian Constitution. Money bills areconcerned with financial matters liketaxation, public expenditure, etc.A MoneyBill may only be introduced in Lok Sabha,on the recommendation of the President.Educational Objective: A Money Billcanonly be introduced in Lok Sabha, on therecommendation of the President.An Appropriation Bill has to be passedby the Parliament for the Government towithdraw money from the ConsolidatedFund of India.A Finance bill contains the proposals ofthe government for levy of new taxes,modification of the existing tax structure orcontinuance of the existing tax structurebeyond the period approved by Parliament.Source: Indian Polity by Laxmikant (4thEdition) – page no: 22.24

42. Answer: DExplanation: Types of Majority -i. Absolute majority - It refers to a

majority of more than 50% of the totalmembership of the house.

ii. Effective Majority – itmeans morethan 50% of the effective strength of thehouse. This implies that vacant seatsare not taken into consideration.

iii. Simple Majority - this refers to themajority of more than 50% of themembers present and voting. This isalso known as functional majority orworking majority.

iv. Special Majority – it requires amajority of 2/3rd members present andvoting supported by more than 50% ofthe total strength of the house. In caseof changes to the federal structure

assent of more than 50% of the StateLegislatures is also required.

An ordinary bill, a Money Bill and aFinance Bill require only a simple majorityof the members present and voting to bepassed by the Parliament. However, aConstitutional Amendment Bill underArticle 368 requires a special majority to bepassed by the Parliament. Changes to theConstitution other than those coveredunder Article 368, can be made by a simplemajority of the members present and voting.Educational Objective: An ordinary billand a Money Bill require only a simplemajority of the members present and voting.Constitutional Amendment Bill underArticle 368 requires special majority to bepassed by the Parliament.Source: Indian Polity by Laxmikant (4thEdition) – page no: 3.2 and 22.18

43. Answer: AExplanation: Money Bill is defined inArticle 110 of the Indian Constitution.Money bills are concerned with financialmatters like taxation, public expenditure,etc.A Money Bill may only be introduced inLok Sabha, on the recommendation of thePresident. It must be passed in Lok Sabhaby a simple majority of all members presentand voting. Following this, it will be sent tothe Rajya Sabha for its recommendations,which Lok Sabha may reject if it chooses to.If such recommendations are not givenwithin 14 days, it will be deemed to bepassed by Parliament. The Lok Sabha hasmore powers than the Rajya Sabha in caseof money bills only.When a money bill is presented to thepresident, he may give his assent to the billor withhold his assent to the bill but hecannot return the bill for reconsideration(money bill introduced with the priorpermission of the President).The decision of the Speaker of the LokSabha is final in case a question arises onwhether a bill is money bill or not. Hisdecision in this regard cannot be questionedin any court of law or in either House of theParliament or even by the President. The

Page 75: PolityPreviousYearQuestionBank · 1979. 8. 31.  · PolityPreviousYearQuestionBank PageNo.5 Company can give licence of trade to individuals and its own employees to trade in India:

Polity Previous Year Question Bank

www.laex.in https://elearn.laex.inPage No. 72

Speaker endorses a bill as a money billwhen such bill is transmitted to the RajyaSabha and when it is presented to thePresident for his assent.Educational Objective: The decision ofthe Speaker of the Lok Sabha is final incase a question arises on whether a bill ismoney bill or not.Source: Indian Polity by Laxmikant (4thEdition) – page no: 22.19

44. Answer: BExplanation: The Vice-President ofIndia is the ex-officio Chairman of theRajya Sabha. Therefore, he is not a memberof the Rajya Sabha.The Deputy Chairman of the RajyaSabha is elected by the Rajya Sabha itselffrom among its members. He presides overthe House when the Chairman is notavailable.A panel of Vice-Chairpersons of theRajya Sabha is nominated by theChairman from amongst its members. Anyone of them can preside over the House inthe absence of both the Chairman and theDeputy-Chairman.Election of the President – he/she iselected indirectly (by people’srepresentatives) through proportionalrepresentation by means of singletransferable vote system. The electoralcollege consists of –

i. The elected members of the boththe Houses of Parliament.

ii. The elected members of theLegislative Assemblies of thestates.

iii. The elected members of theLegislative Assemblies of theUnion Territories of Delhi andPuducherry.

Nominated members of the legislature bothat the Union and the States does notparticipate in the presidential election.Election of the Vice-President – he/sheis elected indirectly (by people’srepresentatives) through proportionalrepresentation by means of singletransferable vote system. The electoral

college consists of the members from theboth houses of the Parliament. Nominatedmembers of the Parliament also participatein the Vice-Presidential election.Educational Objective: The Vice-President of Indiawho is the ex-officioChairman of the Rajya Sabha is notconsidered to be a member of the RajyaSabha.Nominated members of the legislature bothat the Union and the States does notparticipate in the presidential election.Nominated members of the Parliament alsoparticipate in the Vice-Presidential election.Source: Indian Polity by Laxmikant (4thEdition) – page no: 22.9, 17.1 and 18.1

45. Answer: BExplanation: The Speaker of the LokSabha is elected from amongst its membersby a simple majority. The date of theelection is fixed by the President. He/shehas to be a member of the Lok Sabha at thetime of the election.The speaker doesn’t hold office during thepleasure of the President. The office of theSpeaker is a Constitutional office. Powersof the Speaker are derived from theConstitution of India, Rules of Procedureand Parliamentary Conventions.The Speaker has to vacate his office if –

i. He/she ceases to be a member ofthe Lok Sabha.

ii. He/she resigns by writing to theDeputy Speaker.

iii. He/she is removed by aresolution passed by Lok Sabhawith absolute majority (morethan 50% of the totalmembership of the Lok Sabha).

Whenever the Lok Sabha is dissolved, theSpeaker does not vacate his office andcontinues till the newly elected Lok Sabhameets.Educational Objective: The Speaker ofthe Lok Sabha is elected from its membersby a simple majority.The Speaker can resign by writing to theDeputy Speaker.

Page 76: PolityPreviousYearQuestionBank · 1979. 8. 31.  · PolityPreviousYearQuestionBank PageNo.5 Company can give licence of trade to individuals and its own employees to trade in India:

Polity Previous Year Question Bank

www.laex.in https://elearn.laex.inPage No. 73

Source: Indian Polity by Laxmikant (4thEdition) – page no: 22.7

46. Answer: DExplanation: The President summonseach of the Houses of the Parliament tomeet on the recommendation of the PrimeMinister. A session of the Parliament is theperiod spanning between the first sitting ofthe House and its prorogation. When thebusiness of the House is completed, theHouse is adjourned sine die by its presidingofficer. Within the next few days, thePresident issues a notification forprorogation of the session. However, thePresident can also prorogue the Housewhile it is in session. The House can meetagainafter its prorogation only after thePresident summons the House to meet.The Lok Sabha is not a continuing chamberlike the Rajya Sabha. The normal term ofthe Lok Sabha is 5 years from the date ofits first meeting after the general elections,after which it is automatically dissolved.However, the term of the Lok Sabha can beextended by 1 year at a time by a law of theParliament during the period of NationalEmergency (Article 352).Whenever the Lok Sabha is dissolved, theSpeaker does not vacate his office andcontinues till the newly elected Lok Sabhameets.Educational Objective: The Lok Sabhagets dissolved automatically at the end of 5years except during National Emergency(Article 352).Whenever the Lok Sabha is dissolved, theSpeaker does not vacate his office andcontinues till the newly elected Lok Sabhameets.Source: Indian Polity by Laxmikant (4thEdition) – page no: 22.4 and 22.7

47. Answer: DExplanation: Decorum refers to abehavior that is socially correct, calm, andpolite.Crossing the floor means passingbetween the member addressing the House

and the Chair, which is considered breachof Parliamentary etiquette.Interpellation is the process of askingquestions formally of a minister in theParliament.Yielding the floor refers to a situationwhen thespeaker of the Lok Sabha asks amember of the house to stop speaking andlet another member speak.Educational Objective:Yielding the floor refers to a situationwhen thespeaker of the Lok Sabha asks amember of the house to stop speaking andlet another member speak.Source: Indian Polity by Laxmikant (4thEdition) – not available.

48. Answer: AExplanation: Article 175 of theConstitution of India deals with legislativepower of the Governor to address eitherHouse of the state legislature or both theHouses assembled together.The Governorcan send message to a House or both theHouses of the state legislature with respectto a pending bill or otherwise and suchmessage shall be considered by the House.Article 176 of the Constitution of Indiadeals with the specialaddress by theGovernor to the state legislature at thecommencement of the first session aftereach general election and the first sessionof each year.Article 208 of the Constitution of Indiadeals with the Rules of procedure of thestate legislature. According to it -

i. A House of the State Legislaturemay make rules for regulatingits procedure and the conduct ofits business (subject to theprovisions of this Constitution).

ii. Until such rules are made, therules of procedure and standingorders in force immediatelybefore the commencement of thisConstitution with respect to theLegislature for thecorresponding Province shallhave effect in relation to theLegislature of the State subject

Page 77: PolityPreviousYearQuestionBank · 1979. 8. 31.  · PolityPreviousYearQuestionBank PageNo.5 Company can give licence of trade to individuals and its own employees to trade in India:

Polity Previous Year Question Bank

www.laex.in https://elearn.laex.inPage No. 74

to such modifications andadaptations as may be madetherein by the Speaker of theLegislative Assembly, or theChairman of the LegislativeCouncil.

iii. The Governor after consultation withthe Speaker of the Legislative Assemblyand the Chairman of the LegislativeCouncil, may make rules regarding theprocedure for communication betweenthe Houses.This implies that state legislature doesnot follow Lok Sabha rules, as no suchprovision exists.Educational Objective: Article 175and 176 of the Constitution of India –Power of the Governor to address theHouses in the state legislature.Article 208 of the Constitution ofIndia – Power a House in the statelegislature to make rules for regulatingits procedure and the conduct of itsbusiness.Source: Indian Polity by Laxmikant(4th Edition) – page no: 26.5

49. Answer: BExplanation: Articles 168-212 in partVI of the Constitution of India deal withthe State Legislature. Article 170 of theConstitution of India deals with thecomposition of the Legislative Assembly.According to it, the assembly shall consistof representatives elected by the people onthe basis of universal adult franchise. Theconstitution has fixed the maximumstrength of the assembly at 500 and theminimum strength at 60. The no. of seatsin the assembly is determined by thepopulation of the state. However, in case ofArunachal Pradesh, Sikkim and Goa theminimum number is 30 and in case ofMizoram and Nagaland the minimumnumber is 40 and 46 respectively. Somemembers of the legislative assemblies inSikkim and Nagaland are also electedindirectly.Article 173 of the Constitution of Indiadeals with the qualifications to become amember of the state legislature. The

Constitution lays down the followingqualifications –i. he/she must be a citizen of India.ii. he/she must make and subscribe to an

oath or affirmation before a personauthorized by the Election Commissionof India.

iii. he/she mustnot be less than 30years of age in the case ofLegislative Council and not lessthan 25 years of age in case of theLegislative Assembly.

iv. he/she must possess other qualificationsprescribed by the Parliament.

The Parliament has laid down the followingadditional qualifications in theRepresentation of People Act, 1951 –i. A person to be elected to the legislative

council must be a registered as anelector for an assembly constituency inthe concerned state and to be qualifiedfor the governor’s nomination he mustbe a resident in the concerned state.

ii. A person to be elected to the legislativeassembly must be a registered as anelector for an assembly constituency inthe concerned state.

iii. In order to contest for areservedseat,he/she must belong to aScheduled Caste or a Scheduled Tribe.However, a SC/ST can also contest for aseat not reserved for them.

Educational Objective: The constitutionhas fixed the maximum strength of theassembly at 500 and the minimum strengthat 60 (few exceptions like ArunachalPradesh, Sikkim, Goa, Mizoram andNagaland).In order to be qualified for the statelegislature, a person must not be less than30 years of age in the case of LegislativeCouncil and not less than 25 years of age incase of the Legislative Assembly.Source: Indian Polity by Laxmikant (4thEdition) – page no: 29.2 and 29.3

50. Answer: AExplanation: The Speaker of thelegislative assembly is elected fromamongst the members of the assembly. The

Page 78: PolityPreviousYearQuestionBank · 1979. 8. 31.  · PolityPreviousYearQuestionBank PageNo.5 Company can give licence of trade to individuals and its own employees to trade in India:

Polity Previous Year Question Bank

www.laex.in https://elearn.laex.inPage No. 75

Speaker has to vacate his office in thefollowing cases –i. If he/she ceases to be a member of the

assembly.ii. If he/she resigns by writing to the

Deputy Speaker.iii. If he/she is removed by a resolution

passed by a majority of all the thenmembers of the assembly (EffectiveMajority).

Whenever the state legislative assembly isdissolved, the Speaker does not vacate hisoffice and continues till the newly electedlegislative assembly meets.

Educational Objective: In case of adissolution, the Speaker continues to bein office till the first meeting of thenewly elected legislative assembly.Source: Indian Polity by Laxmikant(4th Edition) – page no: 29.5

51. Answer: DExplanation: Article 171 of theConstitution of India deals with thecomposition of the state Legislative Council.According to it, the maximum strength ofthe Council shall not exceed one-third ofthe total membership of the LegislativeAssembly and the minimum strength isfixed at 40. This is to ensure the dominanceof the assembly (directly elected) in thelegislative affairs of the state. Theconstitution has authorized the Parliamentto modify the composition of the council.However, no such law has been made so far.Members of the legislative council areelected indirectly through proportionalrepresentation by means of a singletransferable vote system. The manner inwhich they are elected –i. 1/3rd are elected by the members of the

local bodies in the state likemunicipalities, district boards, etc.

ii. 1/12th are elected by graduates (musthave graduated 3 years prior to theelection) residing in the state.

iii. 1/12th are elected by teachers (musthave a working experience of 3 yearsand they must be teaching at least in aSecondary School) residing in the state.

iv. 1/3rd are elected by the members of thelegislative assembly of the state.

v. the remainder i.e. 1/6th are nominatedby the Governor from amongst thepersons who have special knowledge ofliterature, science, art, cooperativemovement and social service.

The Chairman of the Council is electedfrom amongst its members, unlike the Vice-President who is the ex-officio Chairman ofthe Rajya Sabha.Educational Objective: The maximumstrength of the Legislative Council shall notexceed one-third of the total membership ofthe Legislative Assembly and the minimumstrength is fixed at 40.The Chairman of the Council is electedfrom amongst its members.Source: Indian Polity by Laxmikant (4thEdition) – page no: 29.2 and 29.6

8. Judiciary

8.1 Appointment and Impeachment ofJudges –

8.1.1 The Supreme Court

1. Consider the followingstatements:

1. The motion to impeach a Judge of theSupreme Court of India cannot berejected by the Speaker of the LokSabha as per the Judges (Inquiry)Act, 1968.

2. The Constitution of India defines andgives details of what constitutes‘incapacity and proved misbehaviourof the Judges of the Supreme Court ofIndia.

3. The details of the process ofimpeachment of the Judges of theSupreme Court of India are given inthe Judges (Inquiry) Act, 1968.

4. If the motion for the impeachment ofa Judge is taken up for voting, thelaw requires the motion to be backedby each House of the Parliament andsupported by a majority of totalmembership of that House and by not

Page 79: PolityPreviousYearQuestionBank · 1979. 8. 31.  · PolityPreviousYearQuestionBank PageNo.5 Company can give licence of trade to individuals and its own employees to trade in India:

Polity Previous Year Question Bank

www.laex.in https://elearn.laex.inPage No. 76

less than two-thirds of total membersof that House present and voting.

Which of the statements given aboveis/ are correct?

(a) 1 and 2(b) 3 only(c) 3 and 4 only(d) 1, 3 and 4

8.1.2 High Court2 Consider the following

statements:1. A person who has held office as a

permanent Judge of a High Courtcannot plead or act in any court orbefore any authority in India exceptof the Supreme Court.

2. A person is not qualified forappointment as a Judge of a HighCourt in India unless he has for atleast five years held a judicial officein the territory of India.

Which of the statement(s) given aboveis/are correct?a) 1 onlyb) 2 onlyc) Both 1 and 2d) Neither 1 nor 2

3. Consider the followingstatements:

1. The mode of removal of a Judge of aHigh Court in India is same as thatof removal of a Judge of the SupremeCourt.

2. After retirement from the office, apermanent judge of a High Courtcannot plead or act in any court orbefore any authority in India.

Which of the statements given aboveis/are correct?

a) 1 onlyb) 2 onlyc) Both 1 and 2d) Neither 1 nor 2

8.2 Independence/Autonomy of theJudiciary

8.2.1 – Supreme Court

4. What is the provision tosafeguard the autonomy of theSupreme Court of India?

1. While appointing the Supreme CourtJudges, the President of India has toconsult the Chief Justice of India.

2. The Supreme Court Judges can beremoved by the Chief Justice of Indiaonly.

3. The salaries of the Judges arecharged on the Consolidated Fund ofIndia to which the legislature doesnot have to vote.

4. All appointments of officers andstaffs of the Supreme Court of Indiaare made by the Government onlyafter consulting the Chief Justice ofIndia.

Which of the statements given aboveis/are correct?

a) 1 and 3 onlyb) 3 and 4 onlyc) 4 onlyd) 1, 2, 3 and 4

8.2.2 High Courts5. The salaries and allowances of

the Judges of the High Court arecharged on the:

a) Consolidated Fund of Indiab) Consolidated Fund of the Statec) Contingency Fund of Indiad) Contingency Fund of the State

8.3 Jurisdiction and Power ofJudiciary

8.3.1 Supreme court

6. The power of the Supreme Courtof India to decide disputes

Page 80: PolityPreviousYearQuestionBank · 1979. 8. 31.  · PolityPreviousYearQuestionBank PageNo.5 Company can give licence of trade to individuals and its own employees to trade in India:

Polity Previous Year Question Bank

www.laex.in https://elearn.laex.inPage No. 77

between the Centre and theStates falls under its (2014)

a) advisory jurisdictionb) appellate jurisdictionc) original jurisdictiond) writ jurisdiction

7. Which of the following areincluded in the originaljurisdiction of the SupremeCourt?

1. A dispute between the Government ofIndia and one or more States.

2. A dispute regarding elections toeither House of the Parliament orthat of Legislature of a State.

3. A dispute between the Government ofIndia and a Union Territory.

4. A dispute between two or more States.

Select the correct answer using thecodes given below:

a) 1 and 2 onlyb) 2 and 3 onlyc) 1 and 4 onlyd) 3 and 4 only

8. Consider the followingstatements:The Supreme Court of Indiatenders advice to the Presidentof India on matters of law or fact.

1. On its own initiative (on any matterof larger public interest).

2. If he/she seeks such an advice.3. Only if the matters relate to the

Fundamental Rights of the citizensWhich of the statements given below

is/are correct?a) 1 onlyb) 2 onlyc) 3 onlyd) 1 and 2 only

9. Consider the followingstatements

1. The Judges (Inquiry) Bill 2006contemplates to establish a JudicialCouncil which will receive complaintsagainst judges of the Supreme Courtincluding the Chief Justice of India,High Court Chief Justices andJudges.

2. Under the Protection of Women fromDomestic Violence Act, 2005, awoman can file a petition before a 1stclass judicial magistrate.

Which of the statements given aboveis/are correct?

a) 1 onlyb) 2 onlyc) Both 1 and 2d) Neither 1 nor 2

10. The power to enlarge thejurisdiction of the SupremeCourt of India with respect toany matter included in the UnionList of Legislative Powers restswith:

a) The President of Indiab) The Chief Justice of Indiac) The Parliamentd) The Union Ministry of Law, Justice

and Company Affairs

11. Consider the followingstatements:

1. The Parliament cannot enlarge thejurisdiction of the Supreme Court ofIndia as its jurisdiction is limited tothat conferred by the Constitution.

2. The officers and servants of theSupreme Court and High Courts areappointed by the concerned ChiefJustice and the administrativeexpenses are charged on theConsolidated fund of India.

Which of the statements given aboveis/are correct?

a) 1 only

Page 81: PolityPreviousYearQuestionBank · 1979. 8. 31.  · PolityPreviousYearQuestionBank PageNo.5 Company can give licence of trade to individuals and its own employees to trade in India:

Polity Previous Year Question Bank

www.laex.in https://elearn.laex.inPage No. 78

b) 2 onlyc) Both 1 and 2d) Neither 1 nor 2

8.3.2 High Court

12. Assertion (A): In India, every Statehas a High Court in its territory.Reason (R): The Constitution ofIndia provides a High Court in eachState.

Codes:a) Both A and R are individually true

and R is the correct explanation of A.b) Both A and R are individually true

but R is not the correct explanation ofA.

c) A is true but R is false.d) A is false but R is true.

13. Which one of the following HighCourts has the TerritorialJurisdiction over Andaman andNicobar Islands?

a) Andhra Pradeshb) Kolkatac) Chennaid) Orissa

14. Consider the followingstatements regarding the HighCourts in India:

1. There are eighteen High Courts inthe country.

2. Three of them have jurisdiction overmore than one state.

3. No Union Territory has a High Courtof its own.

4. Judges of the High Court hold officetill the age of 62 years

Which of these statements is/arecorrect?

a) 1, 2 and 4b) 2 and 3

c) 1 and 4d) 4 only

15. How many High Courts in Indiahave jurisdiction over more thanone State (Union Territories notincluded)?

a) 2b) 3c) 4d) 5

16. Consider the followingstatements:

1. There are 25 High Courts in India.2. Punjab, Haryana and the Union

Territory of Chandigarh have acommon HighCourt.

3. National Capital Territory of Delhihas a High Court of its own.

Which of the statements given aboveis/are correct?

a) 2 and 3b) 1 and 2c) 1, 2 and 3d) 3 only

8.4 Judicial Review , Judicial Activismand Judicial Overreach

17. In India, Judicial Review impliesa) The power of the Judiciary to

pronounce upon the constitutionalityof laws and executive orders.

b) The power of the Judiciary toquestion the wisdom of the lawsenacted by the Legislatures.

c) The power of the Judiciary to reviewall the legislative enactments beforethey are assented to by the President.

d) The power of the Judiciary to reviewits own judgements given earlier insimilar or different cases.

Page 82: PolityPreviousYearQuestionBank · 1979. 8. 31.  · PolityPreviousYearQuestionBank PageNo.5 Company can give licence of trade to individuals and its own employees to trade in India:

Polity Previous Year Question Bank

www.laex.in https://elearn.laex.inPage No. 79

18. Consider the followingstatements:

1. The 44th Amendment to theConstitution of India introduced anarticle placing the election of thePrime Minister beyond judicialreview.

2. The Supreme Court of India struckdown the 99th Amendment to theConstitution of India as beingviolative of the independence ofjudiciary.

Which of the statements given aboveis/are correct?

a) 1 onlyb) 2 onlyc) Both 1 and 2d) Neither 1 nor 2

19. With reference to theConstitution of India, considerthe following statements:

1. No High Court shall have thejurisdiction to declare any central lawto be constitutionally invalid.

2. An amendment to the Constitution ofIndia cannot be called into questionby the Supreme Court of India.

Which of the statements given above is/are correct?

a) 1 onlyb) 2 onlyc) Both 1 and 2d) Neither 1 nor 2

8.5 Subordinate Courts

20. Consider the followingstatements:

1. The highest criminal court of thedistrict is the Court of District andSession Judge.

2. The District Judge are appointed bythe Governor in consultation with theHigh Courts.

3. A person to be eligible forappointment as a District Judgeshould be an advocate or a pleader ofseven years standing or more, or anofficer in judicial service of the Unionor the State.

4. When the sessions judge awards adeath sentence, it must be confirmedby the High Court before it is carriedout.

Which of the statements given aboveare correct?

a) 1 and 2b) 2, 3 and 4c) 3 and 4d) 1, 2, 3 and 4

8.6 Tribunals for other matters

21. How is the National GreenTribunal (NGT) different fromthe Central Pollution ControlBoard (CPCB)?

1. The NGT has been established by anAct whereas the CPCB has beencreated by an executive order of theGovernment.

2. The NGT provides environmentaljustice and helps reduce the burdenof litigation in the higher courtswhereas the CPCB promotescleanliness of streams and wells, andaims to improve the quality of air inthe country.

Which of the statements given aboveis/are correct?

a) 1 onlyb) 2 onlyc) Both 1 and 2d) Neither 1 nor 2

Page 83: PolityPreviousYearQuestionBank · 1979. 8. 31.  · PolityPreviousYearQuestionBank PageNo.5 Company can give licence of trade to individuals and its own employees to trade in India:

Polity Previous Year Question Bank

www.laex.in https://elearn.laex.inPage No. 80

22. Consider the followingstatements:

1. Central Administrative Tribunal(CAT) was set up during the PrimeMinistership of Lal Bahadur Shastri.

2. The Members of CAT are drawn fromboth judicial and administrativestreams.

Which of the statements given aboveis/are correct?

a) 1 onlyb) 2 onlyc) Both 1 and 2d) Neither 1 nor 2

23. The National Green Tribunal Act,2010 was enacted in consonancewith which of the followingprovisions of the Constitution ofIndia?

1. Right to healthy environment,construed as a part of Right to lifeunder Article 21.

2. Provision of grants for raising thelevel of administration in theScheduled Areas for the welfareofScheduled Tribes under Article275(1).

3. Powers and functions of Gram Sabhaas mentioned under Article 243 (A)

Select the correct answer using thecodes given below: (2012)

a) 1 onlyb) 2 and 3 onlyc) 1 and 3 onlyd) 1, 2 and 3

8.7 Alternative Dispute RedressalMechanisms

24. With reference to Lok Adalat,consider the followingstatements:

1. An award made by a Lok Adalat isdeemed to be a decree of a civil courtand no appeal lies against there toany court.

2. Matrimonial/Family disputes are notcovered under Lok Adalat.

Which of the statements given aboveis/are correct? (2010)

a) 1 onlyb) 2 onlyc) Both 1 and 2d) Neither 1 nor 225. With reference to the ‘Gram

Nyayalaya Act’, which of thefollowing statements is/arecorrect?

1. As per the Act, Gram Nyayalayas canhear only civil cases and not criminalcases.

2. The Act allows local social activists asmediators/reconciliators.

Select the correct answer using thecode given below:

a) 1 onlyb) 2 onlyc) Both 1 and 2d) Neither 1 nor 2

26. With reference to National LegalServices Authority, consider thefollowing statements:

1. Its objective is to provide free andcompetent legal services to theweaker sections of the society on thebasis of equal opportunity.

2. It issues guidelines for the StateLegal Services Authorities toimplement the legal programmes andschemes throughout the country.

Which of the statements given aboveis/are correct? (2013)a) 1 onlyb) 2 onlyc) Both 1 and 2d) Neither 1 nor 2

8.8 Miscellaneous27. Which of the following are

regarded as the main features ofthe “Rule of Law"?

Page 84: PolityPreviousYearQuestionBank · 1979. 8. 31.  · PolityPreviousYearQuestionBank PageNo.5 Company can give licence of trade to individuals and its own employees to trade in India:

Polity Previous Year Question Bank

www.laex.in https://elearn.laex.inPage No. 81

1. Limitation of powers2. Equality before law3. People's responsibility to the

Government4. Liberty and civil rights

Select the correct answer using thecode given below:

a) 1 and 3 onlyb) 2 and 4 onlyc) 1, 2 and 4 onlyd) 1, 2, 3 and 4

Key & Explanation1. Answer: CExplanation: Article 124 of theConstitution of India deals with theestablishment and constitution of theSupreme Court. According to it there shallbe a Supreme Court of India consisting of aChief Justice of India and until theParliament by law prescribes a largernumber, of not more than 7 other judges.Every judge of the Supreme Court shall beappointed by the President.A judge of the Supreme Court can only beremoved by an order of the President. Suchan order can be issued only after anaddress by the Parliament has beenpresented to him in the same session forsuch removal. The address must besupported by a special majority i.e.majority of the total membership of thehouse and a majority of not less than two-thirds of the members present and votingin both the houses of the Parliament. Thegrounds for removal are provedmisbehaviour and incapacity. Theconstitution has not defined these groundsand it allows the Parliament to regulate theprocedure for the presentation of anaddress and for investigation and proof ofthe misbehaviour or incapacity of a judge.

The Parliament has enacted TheJudges (Inquiry) Act, 1968to regulatethe impeachment process. According toit –i. A removal motion signed by at

least 100 members in case of theLok Sabha or 50 members in

case of the Rajya Sabha is to begiven to the presiding officer.

ii. The presiding officer mayadmit the motion or refuse it.

iii. If admitted, a 3-membercommittee to investigate into thecharges is constituted by thepresiding officer and thecommittee should consist of

a. The Chief Justice of India or ajudge of the Supreme Court

b. A Chief Justice of a High Courtand

c. A distinguished juristiv. If the committee finds the judge

to be guilty of misbehaviour orincapacity then the House cantake up the consideration of themotion.

v. After the motion is passed byeach house of the parliament byspecial majority, an address ispresented to the President forthe removal of the judge.

vi. Finally, the President passes anorder to remove the judge.

The removal of judge of the Supreme courtor that of a High court is a quasi-judicialprocess. Till now no judge of the SupremeCourt or that of a High court has beenimpeached.Educational Objective: Grounds forremoval of a judge of the Supreme Court orthat of a High court are provedmisbehaviour and incapacity only.The impeachment process of the judges isregulated by The Judges (Inquiry) Act,1968.Source: Indian Polity by Laxmikant (4thEdition) – page no: 25.2

Appointment and Impeachment ofJudges – High Courts

2. Answer: DExplanation: Article 217 of theConstitution of India says that every judgeof a High court shall be appointed by thePresident by warrant under his hand andseal. A person shall not be qualified to

Page 85: PolityPreviousYearQuestionBank · 1979. 8. 31.  · PolityPreviousYearQuestionBank PageNo.5 Company can give licence of trade to individuals and its own employees to trade in India:

Polity Previous Year Question Bank

www.laex.in https://elearn.laex.inPage No. 82

become the judge of a High court unlesshe/she is a citizen of India and

i. has for at least 10 years been anadvocate of a High Court or

ii. has for at least 10 years held ajudicial office in the territory ofIndia.

According to the Third Judges Case, 1998the Supreme court has ruled thattheappointment of a judge to a High court is tobe made by a collegium consisting of theChief Justice of India and 2 senior mostjudges of the Supreme court.Independence of the judiciary is of utmostimportance in a democracy. Theconstitution has ensured the independenceof the judiciary by –i. Mode of appointment – judges are

appointed by the President inconsultation with the judiciary itself.This curtails the interference of theexecutive in the judicial appointments.

ii. Security of tenure – this means thatthey do not hold office during thepleasure of the president even thoughthey are appointed by him.

iii. Fixed service conditions –the salaries,allowances, privileges and pension ofthe judges of a High court is determinedby the Parliament and they cannot bechanged to their disadvantage exceptduring a Financial Emergency.

iv. Expenses are charged on theConsolidated Fund of the state in caseof a High court.

v. The conduct of the judges cannot bediscussed in the legislature exceptduring an impeachment motion.

vi. Ban on practice after retirement –the retired judge of a High court isprohibited from practicing law in anycourt or before any authority in Indiaexcept the Supreme court and otherHigh courts.

vii. Power to punish for its contempt isvested in a High court to maintain itsauthority, dignity and honour.

viii. The jurisdiction of the High court asgiven in the Constitution cannot becurtailed by the legislature.

ix. Article 50 of the Constitution of Indiadirects the state to separate thejudiciary from the executive.

x. The High court is given the freedom toappoint its staff without anyinterference from the executive.Educational Objective: Qualificationsto become a judge of High court –i. must be a citizen of Indiaii. has for at least 10 years been an

advocate of a High Court oriii. has for at least 10 years held a

judicial office in the territory ofIndia.

A retired judge of a High court can onlypractice law in the Supreme court and inthose High courts in which he was not ajudge of before retirement.Source: Indian Polity by Laxmikant (4thEdition) – page no: 30.2 and 30.4

3. Answer: AExplanation: Article 218 of theConstitution of states that process ofremoval of a High court judge is same asthat of a judge of the Supreme court.A judge of a High Court can only beremoved by an order of the President. Suchan order can be issued only after anaddress by the Parliament has beenpresented to him in the same session forsuch removal. The address must besupported by a special majority i.e.majority of the total membership of thehouse and a majority of not less than two-thirds of the members present and votingin both the houses of the Parliament. Thegrounds for removal are provedmisbehaviour and incapacity. Theconstitution has not defined these groundsand it allows the Parliament to regulate theprocedure for the presentation of anaddress and for investigation and proof ofthe misbehaviour or incapacity of a judge.The Parliament has enacted The Judges(Inquiry) Act, 1968 to regulate theimpeachment process. According to it –i. A removal motion signed by at least 100

members in case of the Lok Sabha or 50

Page 86: PolityPreviousYearQuestionBank · 1979. 8. 31.  · PolityPreviousYearQuestionBank PageNo.5 Company can give licence of trade to individuals and its own employees to trade in India:

Polity Previous Year Question Bank

www.laex.in https://elearn.laex.inPage No. 83

members in case of the Rajya Sabha isto be given to the presiding officer.

ii. The presiding officer may admit themotion or refuse it.

iii. If admitted, a 3-member committee toinvestigate into the charges isconstituted by the presiding officer andthe committee should consist of

a. The Chief Justice of India or a judge ofthe Supreme Court

b. A Chief Justice of a High Court andc. A distinguished jurist

iv. If the committee finds the judge to beguilty of misbehaviour or incapacitythen the House can take up theconsideration of the motion.

v. After the motion is passed by eachhouse of the parliament by specialmajority, an address is presented to thePresident for the removal of the judge.

vi. Finally, the President passes an orderto remove the judge.

The removal of judge of the Supreme courtor that of a High court is a quasi-judicialprocess.Till now no judge of the SupremeCourt or that of a High court has beenimpeached.To ensure the independence of the judiciary,the Constitution has placed a ban onpractice after retirement – the retiredjudge of a High court is prohibited frompracticing law in any court or before anyauthority in India except the Supremecourt and other High courts.Educational Objective: The process ofremoval (impeachment) of a High courtjudge is same as that of a judge of theSupreme court.Source: Indian Polity by Laxmikant (4thEdition) – page no: 30.2 and 30.4

4. Answer: AExplanation: Article 124 of theConstitution of India says that the ChiefJustice of the Supreme Court shall beappointed by the President afterconsultation with such judges of theSupreme court and of the High Courts ashe deems necessary. In case of appointment

of a judge to the Supreme Court the ChiefJustice of India shall always be consultedby the President.In the Second Judges Case, 1993 theSupreme court has said that theconsultation by the President with theChief Justice of India means concurrencewith views of the CJI. In the ThirdJudges Case,1998 the Supreme court hassaid that a judge to the Supreme court isappointed by the President in accordancewith the decision of a Collegium consistingof the Chief Justice of India and 4 seniormost judges of the Supreme court.

Independence of the judiciary is ofutmost importance in a democracy. Theconstitution has ensured theindependence of the judiciary by –

i. Mode of appointment – judges areappointed by the President inconsultation with the judiciary itself.This curtails the interference of theexecutive in the judicial appointments.

ii. Security of tenure – this means thatthey do not hold office during thepleasure of the president even thoughthey are appointed by him. They canonly be removed by the President onlyafter an address by the Parliament hasbeen presented to him in the samesession for such removal. The addressmust be supported by a specialmajority i.e. majority of the totalmembership of the house and a majorityof not less than two-thirds of themembers present and voting in both thehouses of the Parliament.

iii. Fixed service conditions – the salaries,allowances, privileges and pension ofthe judges of the Supreme court isdetermined by the Parliament and theycannot be changed to theirdisadvantage except during a FinancialEmergency.

iv. Expenses are charged on theConsolidated Fund of India.

v. The conduct of the judges cannot bediscussed in the legislature exceptduring an impeachment motion.

vi. Ban on practice after retirement – theretired judge of the Supreme court is

Page 87: PolityPreviousYearQuestionBank · 1979. 8. 31.  · PolityPreviousYearQuestionBank PageNo.5 Company can give licence of trade to individuals and its own employees to trade in India:

Polity Previous Year Question Bank

www.laex.in https://elearn.laex.inPage No. 84

prohibited from practicing law in anycourt or before any authority in India.

vii. Power to punish for its contempt isvested in a Supreme court to maintainits authority, dignity and honour.

viii. The jurisdiction of the Supreme court asgiven in the Constitution cannot becurtailed by the legislature.

ix. Article 50 of the Constitution of Indiadirects the state to separate thejudiciary from the executive.

x. The Chief Justice of India is giventhe freedom to appoint the staff ofthe Supreme court without anyinterference from the executive.

Educational Objective: In the ThirdJudges Case,1998 the Supreme court hassaid that a judge to the Supreme court isappointed by the President in accordancewith the decision of a Collegium consistingof the Chief Justice of India and 4 seniormost judges of the Supreme court.Source: Indian Polity by Laxmikant (4thEdition) – page no: 25.2-4Independence/Autonomy of the Judiciary –High Courts

5. Answer: BExplanation: In order to ensureindependence of the judiciary theConstitution has provided for the salariesand allowances of the judges of the Highcourt as well as the administrativeexpenses of the High court to be charged onthe Consolidated Fund of state. However,pensions of the High court judges arecharged on the Consolidated Fund of India.Educational Objective: Salaries andallowances of the High court judges arecharged on the Consolidated Fund of thestate whereas their pensions are chargedon the Consolidated Fund of India.Source: Indian Polity by Laxmikant (4thEdition) – page no: 30.4Jurisdiction and Power of Judiciary –Supreme Court

6. Answer: C

Explanation: Article 131 of theConstitution of India deals with theoriginal jurisdiction of the Supreme Court.Original jurisdiction means the right tohear such disputes in the first instance andnot by way of appeal. The Supreme court ofIndia has the exclusive power to adjudicatethe disputes between

i. the centre and one or morestates.

ii. the centre and any state/stateson one side and one or morestates on the other.

iii. between two or more states.This jurisdiction of the Supreme court doesnot extend to the following –a. A dispute arising out of any pre-

constitution treaty, agreement, etc.b. A dispute arising of a treaty, agreement,

etc. which specifically provides that thisjurisdiction does not extend to such adispute.

c. Inter-state water disputes.d. Matters referred to the Finance

Commission.e. Ordinary dispute of commercial nature

between the centre and the states.f. Adjustment of certain expenses and

pensions between the centre and thestates.

g. Recovery of damages by a state againstthe centre.Educational Objective: The Supremecourt of India has original jurisdictioninadjudication of the disputes betweenthe centre and the states and ofdisputes among the states.Source: Indian Polity by Laxmikant(4th Edition) – page no: 25.5

7. Answer: CExplanation: Article 131 of theConstitution of India deals with theoriginal jurisdiction of the Supreme court.Original jurisdiction means the right tohear such disputes in the first instance andnot by way of appeal. The Supreme court ofIndia has the exclusive power to adjudicatethe disputes between

Page 88: PolityPreviousYearQuestionBank · 1979. 8. 31.  · PolityPreviousYearQuestionBank PageNo.5 Company can give licence of trade to individuals and its own employees to trade in India:

Polity Previous Year Question Bank

www.laex.in https://elearn.laex.inPage No. 85

i. the centre and one or morestates.

ii. the centre and any state/stateson one side and one or morestates on the other.

iii. between two or more states.This jurisdiction of the Supreme court does

not extend to the following –a. A dispute arising out of any pre-

constitution treaty, agreement, etc.b. A dispute arising of a treaty, agreement,

etc. which specifically provides that thisjurisdiction does not extend to such adispute.

c. Inter-state water disputes.d. Matters referred to the Finance

Commission.e. Ordinary dispute of commercial nature

between the centre and the states.f. Adjustment of certain expenses and

pensions between the centre and thestates.

g. Recovery of damages by a state againstthe centre.

Since the Union Territories are governed bythe centre itself, there is no scope fordisputes to arise.

Disputes regarding elections to eitherHouse of the Parliament or that ofLegislature of a State falls under theoriginal jurisdiction of the High court.Educational Objective: The Supremecourt of India has original jurisdictioninadjudication of the disputes between thecentre and the states and of disputesamong the states.Disputes regarding elections to eitherHouse of the Parliament or that ofLegislature of a State falls under theoriginal jurisdiction of the High court.Source: Indian Polity by Laxmikant (4th

Edition) – page no: 25.5 and 30.5

8. Answer: BExplanation: Article 143 of theConstitution of India authorizes thePresident to seek the opinion of the

Supreme court in matters belonging to twocategories –

i. On any question of law or fact ofpublic importance which hasarisen or which is likely to arise.

ii. On any dispute arising out ofany pre-constitution treaty,agreement, etc.

The Supreme court gives its opinion onlywhen asked by the President. In the firstcase the Supreme court may give or refuseto give its opinion to the President. But inthe second case the Supreme court mustgive its opinion to the President.In both cases the opinion of the Supremecourt is only advisory in nature and is not ajudgement. Therefore, the advice tenderedby the Supreme court is not binding on thePresident. It only facilitates thegovernment to have an authoritative legalopinion.Educational Objective: Article 143 ofthe Constitution of India deals withtheadvisory jurisdiction of the Supremecourt.Source: Indian Polity by Laxmikant (4thEdition) – page no: 25.7

9. Answer: BExplanation: The Judges (Inquiry) Bill,2006 was intended to replace the 1968 Actand establish a National JudicialCouncil (NJC)to conduct inquiries intoallegations of incapacity or misbehaviourby the High Court and the Supreme Courtjudges. The NJC shall consist of the ChiefJustice of India, two Supreme Court judgesand two High Court Chief Justices toinvestigate High Court judges; or the ChiefJustice of India and four Supreme Courtjudges to investigate Supreme Court judges.The NJC shall investigate complaintssubmitted by any person, or upon receivinga reference from Parliament based on amotion moved by 50 Rajya Sabha or 100Lok Sabha MPs. It may also entertaincomplaints from any other source.The billproposes introduction of ‘complaintprocedure’ in addition to the earlier‘reference procedure’ contained in the1968Act.

Page 89: PolityPreviousYearQuestionBank · 1979. 8. 31.  · PolityPreviousYearQuestionBank PageNo.5 Company can give licence of trade to individuals and its own employees to trade in India:

Polity Previous Year Question Bank

www.laex.in https://elearn.laex.inPage No. 86

In a ‘complaint procedure’ a complaint canbe made by anyperson to Judicial Councilagainst Judges of the Supreme Court(except theChief Justice of India), ChiefJustices and Judges of High Courts. Inthe‘reference procedure’, if there is aMotion by Members of Parliament ineitherHouse, the Speaker/Chairman can make areference to Judicial Councilfor inquiry notonly against the above Judges but alsoagainst the ChiefJustice of India. There isprovision for preliminary scrutiny andverificationby the Judicial Council in the‘complaint procedure’ though not inthe‘reference procedure’.If the allegations are proven, the NJC mayimpose minor measures or recommend theremoval of the judge. Removal of a judgeshall be through impeachment byParliament. A judge may appeal to theSupreme Court against his removal oragainst any minor measures imposed uponhim.Protection of Women from DomesticViolence Act, 2005 - defines “DomesticViolence” for the first time. It is acomprehensive definition which includesnot only physical violence but also otherforms of violence such as emotional, verbal,sexual, and economic abuse.It is a civil lawmeant only for protection and not topunishor penalize. It provides onlytemporary and emergency relief.It recognizes the right to residence ofwoman and the right to live in aviolence-free home.If a woman has anapprehension that she will be thrown out ofher matrimonial home, she can approach amagistrate ‘s court under the provisions ofthe Domestic Violence Actand obtain anorder of injunction to restrain her husbandor his family members fromdispossessingher (throwing her out) from hermatrimonial home. If thehusband or hisfamily members violate this order, they willbe liable for punishmentunder theDomestic Violence Act.Under this act, Magistrate means theJudicial Magistrate of the first class, oras the case may be, the MetropolitanMagistrate, exercising jurisdiction in thearea where the aggrieved person resides.

Educational Objective: The Judges(Inquiry) Bill, 2006 intended to replace the1968 Act and establish a National JudicialCouncil (NJC) to investigate any chargesrelating to misbehaviour and incapacity ofa judge.Protection of Women from DomesticViolence Act, 2005 - defines “DomesticViolence” as not only physical violence butalso other forms of violence such asemotional, verbal, sexual, and economicabuse.Source: Indian Polity by Laxmikant (4thEdition) – page no: not available

10. Answer: CExplanation: Article 138 of theConstitution of India says that theParliament by law can extend thejurisdiction of the Supreme court on anymatter in the Union list of legislativepowers.The Supreme court’s jurisdictioncan also be extended to other matters by aspecial agreement between the centre andthe states. However, the Parliament has noauthority to curtail the jurisdiction of theSupreme court that is guaranteed by theConstitution of India.Educational Objective: Article 138 of theConstitution of India states that the powerto enlarge the jurisdiction of the Supremecourt lies with the Parliament.Source: Indian Polity by Laxmikant (4thEdition) – page no: 25.4 and 25.9

11. Answer: DExplanation: Article 138 of theConstitution of India says that theParliament by law can extend thejurisdiction of the Supreme court on anymatter in the Union list of legislativepowers. The Supreme court’s jurisdictioncan be enlarged to other matters by aspecial agreement between the centre andthe states. However, the Parliament has noauthority to curtail the jurisdiction of theSupreme court that are guaranteed by theConstitution of India.Article 146 of the Constitution of Indiaempowers the Chief Justice of India toappoint the staff of the Supreme court. It

Page 90: PolityPreviousYearQuestionBank · 1979. 8. 31.  · PolityPreviousYearQuestionBank PageNo.5 Company can give licence of trade to individuals and its own employees to trade in India:

Polity Previous Year Question Bank

www.laex.in https://elearn.laex.inPage No. 87

also states that the administrativeexpenses of the Supreme court, includingall salaries, allowances and pensions shallbe charged on the Consolidated Fund ofIndia. Similarly, Article 229 of theConstitution of the India empowers theChief Justice of a High court to appoint thestaff of the High court. It also states thatthe administrative expenses of the Highcourt, including all salaries, allowances andpensions (except pensions of the High courtjudges which are charged on theConsolidated Fund of India) shall becharged on the Consolidated Fund of theState. These provisions in the Constitutionensures the independence of the judiciaryfrom interference by the Executive.Educational Objective: Only theParliament has the power to extend theJurisdiction of the Supreme court.The staff of the Supreme Court and HighCourts are appointed by the concernedChief Justice.The administrative expenses are chargedon the Consolidated fund of India in case ofthe Supreme court and on the ConsolidatedFund of the State in case of a High court.Source: Indian Polity by Laxmikant (4thEdition) – page no: 25.4 and 30.4Jurisdiction and Power of Judiciary – HighCourt

12. Answer: AExplanation: Article 214 of theConstitution of India states that there shallbe a High court for each state but the 7thConstitutional Amendment Act of 1956has authorized the Parliament to establisha common High court for two or moreStates. Therefore, every state in Indiadoesn’t have a High court within itsterritory. At present there are 25 Highcourts in India out of which 3 are commonHigh courts.The High court at Guwahatideals with the matters of Assam,Arunachal Pradesh, Nagaland andMizoram. The High court at Chandigarhdeals with the matters of Punjab, Haryanaand Chandigarh.The High court at Mumbaideals with the matters of Maharashtra,Goa, Daman & Diu and Dadra Nagar &Haveli. Delhi is the only Union Territory

with a separate High court. Recently, theHigh court of Andhra Pradesh atAmaravati became the 25th High court inIndia.Educational Objective: At present thereare 25 High courts in India out of which 3are common High courts.Source: Indian Polity by Laxmikant (4thEdition) – page no: 30.1.

13. Answer: BExplanation: The Kolkata High court hasthe territorial jurisdiction over Andamanand Nicobar Islands. Delhi is the onlyUnion Territory with a separate High court.

Union Territory High Court

Andaman &Nicobar

Kolkata

Lakshadweep Kerala

Puducherry Madras

Chandigarh Punjab andHaryana

Daman & Diu Bombay

Dadra Nagar &Haveli

Bombay

Educational Objective: Delhi is the onlyUnion Territory with a separate High court.Source: Indian Polity by Laxmikant (4thEdition) – page no: 30.1

14. Answer: DExplanation: Article 214 of theConstitution of India states that there shallbe a High court for each state but the 7thConstitutional Amendment Act of 1956has authorized the Parliament to establisha common High court for two or moreStates. Therefore, every state in Indiadoesn’t have a High court within itsterritory. At present there are 25 Highcourts in India out of which 3 arecommon High courts.The High court atGuwahati deals with the matters of Assam,Arunachal Pradesh, Nagaland and

Page 91: PolityPreviousYearQuestionBank · 1979. 8. 31.  · PolityPreviousYearQuestionBank PageNo.5 Company can give licence of trade to individuals and its own employees to trade in India:

Polity Previous Year Question Bank

www.laex.in https://elearn.laex.inPage No. 88

Mizoram. The High court at Chandigarhdeals with the matters of Punjab, Haryanaand Chandigarh. The High court atMumbai deals with the matters ofMaharashtra, Goa, Daman & Diu andDadra Nagar & Haveli. Delhi is the onlyUnion Territory with a separate Highcourt. Recently, the High court of AndhraPradesh at Amaravati became the 25thHigh court in India.The Constitution of India has not fixed thetenure of a judge of a High court. However,it makes the following provisions –i. He holds office until he attains the age

of 62 years.ii. He can resign from office by writing to

the President.iii. He can be removed from his office by

the President on recommendation of theParliament.

iv. He vacates his office when appointed asa judge of the Supreme court or whentransferred to another High court.

Educational Objective: Delhi is the onlyUnion Territory with a separate High court.The judge of a High court can remain inoffice until he attains the age of 62 years.Source: Indian Polity by Laxmikant (4thEdition) – page no: 30.1-2

15. Answer: BExplanation: Article 214 of theConstitution of India states that there shallbe a High court for each state but the 7thConstitutional Amendment Act of 1956has authorized the Parliament to establisha common High court for two or moreStates. Therefore, every state in Indiadoesn’t have a High court within itsterritory. At present there are 25 Highcourts in India out of which 3 arecommon High courts. The High court atGuwahati deals with the matters of Assam,Arunachal Pradesh, Nagaland andMizoram. The High court at Chandigarhdeals with the matters of Punjab, Haryanaand Chandigarh. The High court atMumbai deals with the matters ofMaharashtra, Goa, Daman & Diu andDadra Nagar & Haveli. Delhi is the onlyUnion Territory with a separate High

court. Recently, the High court of AndhraPradesh at Amaravati became the 25thHigh court in India.Educational Objective: At present thereare 25 High courts in India out of which 3are common High courts.Source: Indian Polity by Laxmikant (4thEdition) – page no: 30.1

16. Answer: CExplanation: Article 214 of theConstitution of India states that there shallbe a High court for each state but the 7thConstitutional Amendment Act of 1956has authorized the Parliament to establisha common High court for two or moreStates. Therefore, every state in Indiadoesn’t have a High court within itsterritory. At present there are 25 Highcourts in India out of which 3 arecommon High courts.The High court atGuwahati deals with the matters of Assam,Arunachal Pradesh, Nagaland andMizoram. The High court at Chandigarhdeals with the matters of Punjab, Haryanaand Chandigarh. The High court atMumbai deals with the matters ofMaharashtra, Goa, Daman & Diu andDadra Nagar & Haveli. Delhi is the onlyUnion Territory with a separate Highcourt. Recently, the High court of AndhraPradesh at Amaravati became the 25thHigh court in India.Educational Objective: Recently, theHigh court of Andhra Pradesh atAmaravati became the 25th High court inIndia.Source: Indian Polity by Laxmikant (4thEdition) – page no: 22.29Judicial Review, Judicial Activism &Judicial Overreach

17. Answer: AExplanation: The phrase ‘Judicial Review’has nowhere been used in the Constitutionof India. However, the provisions of articleslike Article 13, Article 32, and Article226 explicitly confer the power of judicialreview on the Supreme court and the Highcourts. Judicial review is the power toexamine the constitutionality of legislative

Page 92: PolityPreviousYearQuestionBank · 1979. 8. 31.  · PolityPreviousYearQuestionBank PageNo.5 Company can give licence of trade to individuals and its own employees to trade in India:

Polity Previous Year Question Bank

www.laex.in https://elearn.laex.inPage No. 89

enactments and executive orders of thegovernment. If they are found to beviolative of the constitution they can bedeclared as unconstitutional, illegal andinvalid and they cannot be enforced by thegovernment.The constitutional validity of a legislativeenactment or an executive order can bechallenged on the following grounds –

i. it infringes on the FundamentalRights,

ii. it is outside the competence ofthe authority which has framedit, and

iii. it is repugnant to theconstitutional provisions.

Judicial review emanates from theAmerican principle of judicial supremacy.The Constitution of India of neither followsthe American principle of judicialsupremacy nor the British principle ofparliamentary supremacy completely buthas a synthesis of both. The Americanconstitution provides for “due process oflaw” whereas the Indian constitutionfollows “the procedure established by law”.The difference between the two is – the dueprocess of law gives wide scope to protectthe rights of its citizens. Any law violatingthese rights can be declared void not onlythe substantive grounds (competency of theauthority to make the law) but also onprocedural grounds of being unreasonable.However, in the procedure established bylaw, a law is examined only on thesubstantive grounds and it doesn’t go intothe question of its reasonableness,suitability or policy implications.Educational Objective: The scope ofJudicial Review in India is narrower thanthat of in USA.Source: Indian Polity by Laxmikant (4thEdition) – page no: 25.8

18. Answer: BExplanation: Article 71 of theConstitution provides that disputes arisingout ofthe election of the President or Vice-President shall be decided bythe SupremeCourt. Disputes relating the election of thePrime Minister and the Speaker are

regulated by the provisions of theRepresentation of the People Act, 1951.Under this Act the High Court hasjurisdiction to adjudicate an electionpetition filed against either of them.The 39th Constitutional AmendmentAct of 1975 removed the authority of theSupreme Court and the High court toadjudicate on petitions regarding electionsof the President, Vice President, PrimeMinister and Speaker of the Lok Sabha.Instead, a body constituted by Parliamentwould be vested with the power to resolvesuch election disputes.Article 71 wasamended and Article 329A was inserted inpart XV of the Constitution of India for thispurpose. However, these provisions wererepealed by the 44th ConstitutionalAmendment Act of 1978.The 99th Constitutional AmendmentAct of 2014 was enacted to establishNational Judicial AppointmentsCommission (NJAC), abody responsible forthe appointment and transfer of judges tothe higher judiciary in India. The NJACwould have replaced the collegium systemfor the appointment of judges to the higherjudiciary.The National JudicialAppointments Commission Act, 2014, wasalso passed by the Parliament of India toregulate the functions of the NationalJudicial Appointments Commission.However, the Constitution Bench ofSupreme Court by 4:1 majoritydeclared the99th Constitutional Amendment Act andNJAC Act as unconstitutional because itinfringes upon the independence of thejudiciary.Educational Objective: Disputesregarding the election of the PrimeMinister are within the purview of Highcourt.The 99th Constitutional Amendment Actdeals with National Judicial AppointmentsCommission (NJAC).Source: Indian Polity by Laxmikant (4thEdition) – page no: not available

19. Answer: DExplanation: The 42nd ConstitutionAmendment Act of 1976 curtailed thejudicial review power of High court. It

Page 93: PolityPreviousYearQuestionBank · 1979. 8. 31.  · PolityPreviousYearQuestionBank PageNo.5 Company can give licence of trade to individuals and its own employees to trade in India:

Polity Previous Year Question Bank

www.laex.in https://elearn.laex.inPage No. 90

debarred the High courts from consideringthe constitutional validity of any centrallaw. However, the 43rd ConstitutionAmendment Act of 1977 restored thejudicial review power of High courts withrespect to the union laws.The 42nd Constitution Amendment Actof 1976amended Article 368 of theConstitution of India and declared thatthere is no limitation on the constituentpower of the Parliament and noamendment to the constitution can bequestioned in any court on any groundincluding that of the contravention of any ofthe Fundamental Rights. However, theSupreme court in the Minerva Mills case,1980 invalidated this provision as itexcluded judicial review which is a ‘basicfeature’ of the constitution. The Power ofjudicial review with the Supreme courteven includes checking the constitutionalvalidity of a Constitutional Amendment Act.Educational Objective: A High court hasthe power to declare any central lawviolating the constitution as invalid.A Constitutional Amendment Act alsocomes under the purview of the power ofjudicial review available with the Supremecourt and High courts.Source: Indian Polity by Laxmikant (4thEdition) – page no: 30.7and 11.1Judiciary – Subordinate Courts

20. Answer: DExplanation: The District judge is thehighest judicial authority in the district. Hehas both original and appellate jurisdictionin civil as well as criminal matters. TheDistrict judge when dealing with criminalmatters is called as the Sessions judge. TheDistrict judge also has administrative andsupervisory powers over all the subordinatecourts present in the district. Appealsagainst his orders and judgements lie withthe High court. The Sessions judge has thepower to impose any sentence including lifeimprisonment and death sentence.However, a death sentence awarded by himis subject to confirmation of High court,whether there is appeal or not.

The appointment, posting and promotion ofDistrict judge is made by the Governor ofthe state in consultation with the Highcourt. The qualifications required to beappointed as the District judge are –

i. He should not already be in theservice of the Union or the Stategovernment.

ii. He should have been practicinglaw for at least 7 years.

iii. He should be recommended bythe High court for appointment.

Educational Objective: Source: IndianPolity by Laxmikant (4th Edition) – page no:31.1-2

21. Answer: BExplanation: Article 48A of theConstitution of India talks about protectionand improvement of environment andsafeguarding of forests and wildlife. TheSupreme court of India has declared thatthe Right to healthy environment is a partof Right to life and personal liberty coveredunder Article 21 of the Constitution ofIndia. The Parliament in pursuit of aboveprovisions has enacted National GreenTribunal Act, 2010. This Act provides forthe establishment of a National GreenTribunal (NGT) for the effective andexpeditious disposal of cases relating toenvironmental protection and conservationof forests and other natural resources.NGT has the power to enforce any legalright relating to environment and to giverelief and compensation. The Tribunal'sdedicated jurisdiction in environmentalmatters will provide for speedyenvironmental justice andreduces theburden of litigation in the higher courts.The Tribunal is not bound by the procedurelaid down under the Code of CivilProcedure of 1908, instead it follows theprinciples of natural justice. The tribunal ismandated to dispose of a case within 6months of its filing.

The Central Pollution Control Board(CPCB) of India is a statutory organisationunder the Ministry of Environment, Forestand Climate Change. It was established in

Page 94: PolityPreviousYearQuestionBank · 1979. 8. 31.  · PolityPreviousYearQuestionBank PageNo.5 Company can give licence of trade to individuals and its own employees to trade in India:

Polity Previous Year Question Bank

www.laex.in https://elearn.laex.inPage No. 91

1974 under the Water (Prevention andControl of pollution) Act, 1974. TheCPCB is also entrusted with the powersand functions under the Air (Preventionand Control of Pollution) Act, 1981. Itprovides technical services to the Ministryof Environment and Forests under theprovisions of the Environment(Protection) Act, 1986.It Co-ordinates the activities of the StatePollution Control Boards by providingtechnical assistance and guidance and italso resolves disputes among them. It is theapex organisation in country in the field ofpollution control. CPCB, under the Water(Prevention and Control of Pollution) Act,1974, and the Air (Prevention and Controlof Pollution) Act, 1981, aims to promotecleanliness of streams and wellsbyprevention, control and abatement of waterpollution, and to improve the quality of airand to prevent, control or abate airpollution in the country.

Educational Objective: The NationalGreen Tribunal is a statutory body havingjurisdiction in environmental matters.The Central Pollution Control Board is astatutory body responsible forimplementation of legislation relating toprevention and control of environmentalpollution.Source: Indian Polity by Laxmikant (4thEdition) – page no: not available

22. Answer: BExplanation: The 42nd ConstitutionalAmendment Act of 1976 added a newpart XIV-A to the constitution. This partdeals with tribunals and consists of twoarticles – Article 323A dealing withadministrative tribunals and Article 323Bdealing with tribunals for other matters.Article 323A empowers the Parliament toprovide for the establishment ofadministrative tribunals for adjudication ofdisputes relating to recruitment andconditions of service of persons appointed topublic services of the Centre, the States,local bodies, public corporations and otherpublic authorities. Administrative

Tribunals Act of 1985 was enacted by theParliament to establish the Centraladministrative tribunal (CAT) and theState administrative tribunals. This actenvisages to provide speedy andinexpensive justice to the aggrieved publicservants.The CAT is a multi-member body consistingof a Chairman and 65 members. Themembers are given the status of a Highcourt judge. The members are appointed bythe President and are drawn from bothjudicial and administrative streams. Theyhold office for a term of 5 years or untilthey attain an age of 65 years in case of theChairman and 62 years in case of themembers. The CAT is not bound by theprocedure laid down in the Civil ProcedureCode of 1908, instead it follows theprinciples of natural justice.Lal Bahadur Shastri was the 2nd PrimeMinister of India and he was in office from9th June, 1964 to 11th January, 1966.Educational Objective: Only theParliament has the power to establishadministrative tribunals of the Centre, theStates, local bodies, public corporations andother public authorities.Source: Indian Polity by Laxmikant (4thEdition) – page no: 60.1

23. Answer: AExplanation: Article 48A of theConstitution of India talks about protectionand improvement of environment andsafeguarding of forests and wildlife. TheSupreme court of India has declared thatthe Right to healthy environment is apart of Right to life and personal libertycovered under Article 21 of theConstitution of India. The Parliament inpursuit of above provisions has enactedNational Green Tribunal Act, 2010.This Act provides for the establishment of aNational Green Tribunal (NGT) for theeffective and expeditious disposal of casesrelating to environmental protection andconservation of forests and other naturalresources.NGT has the power to enforce any legalright relating to environment and to giverelief and compensation. The Tribunal's

Page 95: PolityPreviousYearQuestionBank · 1979. 8. 31.  · PolityPreviousYearQuestionBank PageNo.5 Company can give licence of trade to individuals and its own employees to trade in India:

Polity Previous Year Question Bank

www.laex.in https://elearn.laex.inPage No. 92

dedicated jurisdiction in environmentalmatters will provide for speedyenvironmental justice andreduces theburden of litigation in the higher courts.The Tribunal is not bound by the procedurelaid down under the Code of CivilProcedure of 1908, instead it follows theprinciples of natural justice. The tribunal ismandated to dispose of a case within 6months of its filing.Educational Objective: Right to healthyenvironment is part of Right to life andpersonal liberty guaranteed by theConstitution under Article 21.Source: Indian Polity by Laxmikant (4thEdition) – page no: 7.11

24. Answer: AExplanation: Lok Adalat is one of thealternative dispute redressalmechanisms.It is a forum where casespending in the court of law or at pre-litigation stage are settledamicably. LokAdalat is given statutory status under theLegal Services Authorities Act, 1987.An award given by a Lok Adalat is deemedto be a decree of a civil court and is finaland binding on all parties and appealagainst such an award doesn’t lie with anycourt. If the parties are not satisfied withthe award of the Lok Adalat they are free toexercise their right to litigate. It iseconomical to approach Lok Adalat as thereis no court fee and any court fee alreadypaid will be refunded.Nature of Cases that can be referred to LokAdalat -

i. a case pending before any court.ii. a dispute which has not been

brought before any court and islikely to be filed before the court.

Provided that any matter relating to anoffence not compoundable under the lawshall not be settled in Lok Adalat.Compoundable offences are those offenceswhere, the complainant enters into acompromise, and agrees to have thecharges dropped against the accused.Oncean offence has been compounded it shallhave the same effect as an acquittal.

In a non-compoundable offense, acompromise is not allowedbecause thenature of offence is so grave and criminal.Even the court does not have the authorityand power to compound such offense.Most of the matrimonial or family disputescome under compoundable offences andhence are covered under Lok Adalat.Educational Objective: An award givenby the Lok Adalat is deemed to be a decreeof a civil court and is final and binding onall parties and appeal against such anaward doesn’t lie with any court.Source: Indian Polity by Laxmikant (4thEdition)

25. Answer: BExplanation: India's judicial system ischaracterized by systemic problems,including corruption, delays, pendency,increasing costs, limited legal aid, and alack of appropriately trained lawyers andjudges.To overcome these problems theParliament has enactedGram NyayalaysAct of 2008.Gram Nyayalayas are mobile village courtsestablished for speedy and easy access tojustice system in the rural areas ofIndia.They are aimed at providinginexpensive justice to people in rural areasat their doorsteps.The Act also makes thejudicial process participatory anddecentralised because it allowsappointment of local social activists andlawyers as mediators or reconciliators.The Gram Nyayalaya shall be court ofJudicial Magistrate of the first class and isestablished at every Panchayat at anintermediate level. The Nyayadhikaris whowill preside over these GramNyayalayasare appointed by the StateGovernment in consultation with the HighCourt.The Gram Nyayalaya can exercisethe powers of both Criminal and CivilCourts and shall try to settle the disputesas far as possible.Educational Objective: The GramNyayalaya can exercise the powers of bothCriminal and Civil Courts and will try tosettle the disputes as far as possible.

Page 96: PolityPreviousYearQuestionBank · 1979. 8. 31.  · PolityPreviousYearQuestionBank PageNo.5 Company can give licence of trade to individuals and its own employees to trade in India:

Polity Previous Year Question Bank

www.laex.in https://elearn.laex.inPage No. 93

Source: Indian Polity by Laxmikant (4thEdition)

26. Answer: CExplanation: Article 39A of theConstitution of India provides that theState shall strive to secure a legal systemwhich promotes justice on a basis of equalopportunity and provide free legal aid toensure that opportunities for securingjustice are not denied to any citizen byreason of economic or other disability.The National Legal Services Authority(NALSA) has been constituted under theLegal Services Authorities Act, 1987 toprovide free legal services to the weakersections of the society and to organize LokAdalats for settlement of disputes. Thefunctions of the NALSA also includespreading legal literacy and awareness.The Chief Justice of India is the Patron-in-Chief and the Senior most Judge of theSupreme Court of India is the ExecutiveChairman of the Authority.In every State, a State Legal ServicesAuthority has been constituted to giveeffect to the policies and directions of theNALSA and to give free legal services tothe people and conduct Lok Adalats in theState. The State Legal Services Authorityis headed by the Chief Justice of therespective High Court who is the Patron-in-Chief of the State Legal Services Authority.Educational Objective: Article 39A ofthe Constitution of India - justice on a basisof equal opportunity and provide free legalaid.The National Legal Services Authority(NALSA) has been constituted under theLegal Services Authorities Act, 1987 toprovide free Legal Services to the weakersections of the society and to organize LokAdalats.Source: Indian Polity by Laxmikant (4thEdition) – page no: 8.7

27. Answer: CExplanation: Generally, ‘rule of law’ is theprinciple that no one is above the law andis treated equally among citizens. The

concept of ‘Rule of Law’ was propoundedby A.V. Dicey and it has three elements –i. Absence of arbitrary power – no

man can be punished except for abreach of law (limitation of powers).

ii. Equality before the law – equalsubjection of all citizens (rich/poor,high/low, official/non-official) to theordinary law of the land administeredby ordinary courts.

iii. Primacy of the rights of theindividual – the constitution is theresult of the rights of the individual(liberty and civil rights) as defined andenforced by the courts of law ratherthan the constitution being the sourceof the individual rights.

Only the first and the second elements areapplicable to the Indian system. The thirdelement is not applicable because in Indiansystem, the constitution is the source of theindividual rights.Educational Objective: Rule of law byA.V. Dicey -

i. Absence of arbitrary power,ii. Equality before the law andiii. Primacy of the rights of the

individualSource: Indian Polity by Laxmikant(4th Edition) – page no: 7.5

9.Constitutional and Non-Constitutional Bodies

9.1 Constitutional Bodies9.1 .1 Comptroller and Auditor General

of India

1. In India, other than ensuringthat public funds are usedefficiently and for intendedpurpose, what is the importanceof the office of the Comptrollerand Auditor General (CAG)?

1. CAG exercises exchequer control onbehalf of the Parliament when thePresident of India declares nationalemergency/financial emergency.

Page 97: PolityPreviousYearQuestionBank · 1979. 8. 31.  · PolityPreviousYearQuestionBank PageNo.5 Company can give licence of trade to individuals and its own employees to trade in India:

Polity Previous Year Question Bank

www.laex.in https://elearn.laex.inPage No. 94

2. CAG reports on the execution ofprojects or programmes by theministries are discussed by the PublicAccounts Committee.

3. Information from CAG reports can beused by investigating agencies toframe charges against those whohave violated the law whilemanaging public finances.

4. While dealing with the audit andaccounting of government companies,CAG has certain judicial powers forprosecuting those who violate the law.

Which of the statements given aboveis/are correct?a) 1, 3 and 4 onlyb) 2 onlyc) 2 and 3 onlyd) 1, 2, 3 and 4

2. Which one of the following dutiesis not performed by Comptrollerand Auditor general of India?(2001).

a) To audit and report on allexpenditure from the ConsolidatedFund of India.

b) To audit and report on allexpenditure from the ContingencyFunds and Public Accounts.

c) To audit and report on all trading,manufacturing, profit and lossaccounts.

d) To control the receipt and issue ofpublic money, and to ensure that thepublic revenue is lodged in theexchequer.

9.1.2 Election Commission of India

3. Consider the followingstatements: (2017)

1. The Election Commission of India is afive-member body.

2. Union Ministry of Home Affairsdecides the election schedule for theconduct of both general elections andbye-elections.

3. Election Commission resolves thedisputes relating to splits/mergers ofrecognised political parties.

Which of the statements given aboveis/are correct?a) 1 and 2 onlyb) 2 and 3 onlyc) 2 onlyd) 3 only

4. Consider the following tasks:1. Superintendence, direction and

conduct of free and fair elections2. Preparation of electoral rolls for all

elections to the Parliament, StateLegislatures and the Office of thePresident and the Vice-President

3. Giving recognition to political partiesand allotting election symbols topolitical parties and individualscontesting the election

4. Proclamation of final verdict in thecase of election disputes

Which of the above are the functions ofthe Election Commission of India?(a) 1,2 and 3(b) 2,3 and 4(c) 1 and 3(d) 1,2 and 4

5. Which one among the followingCommissions was set up inpursuance of a definite provisionunder an Article of theConstitution of India? (2006)

a) University Grants Commissionb) National Human Rights Commissionc) Election Commissiond) Central Vigilance Commission

6. Consider the followingstatements with reference toIndia: (2002)

Page 98: PolityPreviousYearQuestionBank · 1979. 8. 31.  · PolityPreviousYearQuestionBank PageNo.5 Company can give licence of trade to individuals and its own employees to trade in India:

Polity Previous Year Question Bank

www.laex.in https://elearn.laex.inPage No. 95

1. The Chief Election Commissioner andother Election Commissioners enjoyequal powers but receive unequalsalaries

2. The Chief Election Commissioner isentitled to the same salary as isprovided to a judge of the SupremeCourt

3. The Chief Election Commissionershall not be removed from his officeexcept in like manner and on likegrounds as a judge of the SupremeCourt

4. The term of office of the ElectionCommissioner is five years from thedate he assumes his office or till theday he attains the age of 62 years,whichever is earlier

Which of these statements are correct?a) 1 and 2c) 1 and 4b) 2 and 3d) 2 and 47. Consider the following

statements regarding thepolitical parties in India:

1. The Representation of the People Act,1951 provides for the registration ofpolitical parties.

2. Registration of political parties iscarried out by the ElectionCommission.

3. A national level political party is onewhich is recognised in four or morestates.

4. During the 1999 general elections,there were six national and 48 statelevel parties recognized by theElection Commission

Which of these statements are correct?(2001)

a) 1,2 and 4b) l and 3c) 2 and 4d) 1,2, 3 and 4

9.1.3 Finance Commission8. With reference to the Finance

Commission of India, which ofthe following statements iscorrect? (2011)

a) It encourages the inflow of foreigncapital for infrastructuredevelopment

b) It facilitates the proper distribution offinances among the Public SectorUndertakings

c) It ensures transparency in financialadministration

d) None of the statements (a), (b) and (c)given above is correct in this context.

9. Consider the followingstatements:The function(s) of the Financecommission is/are:

1. To allow the withdrawal of the moneyout of the Consolidated Fund of India

2. To allocate between the States theshares of proceeds of taxes

3. To consider applications for grants-in-aid from States

4. To supervise and report on whetherthe Union and State governments arelevying taxes in accordance with thebudgetary provisions.

Which of these statements is/arecorrect? (2003a) Only 1b) 2 and 3c) 3 and 4d) 1, 2 and 4

10. Which one of the followingauthorities recommends theprinciples governing the grants-in-aid of the revenues to thestates out of the ConsolidatedFund of India? (2002)

a) Finance Commissionb) Inter-State Councilc) Union Ministry of Financed) Public Accounts Committee

Page 99: PolityPreviousYearQuestionBank · 1979. 8. 31.  · PolityPreviousYearQuestionBank PageNo.5 Company can give licence of trade to individuals and its own employees to trade in India:

Polity Previous Year Question Bank

www.laex.in https://elearn.laex.inPage No. 96

Page 100: PolityPreviousYearQuestionBank · 1979. 8. 31.  · PolityPreviousYearQuestionBank PageNo.5 Company can give licence of trade to individuals and its own employees to trade in India:

Polity Previous Year Question Bank

www.laex.in https://elearn.laex.inPage No. 97

11. The primary function of theFinance Commission in India isto: (2000)

a) Distribution of revenue between theCentre and the States

b) Prepare the Annual Budgetc) Advise the President on financial

mattersd) Allocate funds to various ministries

of the Union and State Governments.

12. Who of the following shall causeevery recommendation made bythe Finance Commission to belaid before each House ofParliament? (2010).

a) The President of Indiab) The Speaker of Lok Sabhac) The Prime Minister of Indiad) The Union Finance Minister

9.2 Non-Constitutional Bodies9.2.1 Planning commission

13. Consider the followingstatements: (2004)

1. The highest deciding body forplanning in India is the PlanningCommission of India.

2. The Secretary of the PlanningCommission of India is also theSecretary of National DevelopmentCouncil.

3. The Constitution includes economicand social planning in theConcurrent List in the SeventhSchedule of the Constitution of India.

Which of the statements given aboveis/are correct?

a) 1 and 2 onlyb) 2 and 3 onlyc) 2 onlyd) 3 only

9.2.2 National Human RightsCommission

14. According to the National HumanRights Commission Act, 1993,who amongst the following canbe its Chairman? (2004)

a) Any serving Judge of the SupremeCourt.

b) Any serving Judge of the High Court.c) A retired Chief Justice of India.d) A retired Chief Justice of a High

Court.

15. Consider the followingstatements regarding theNational Human RightsCommission of India:

1. Its Chairman must be a retired ChiefJustice of India.

2. Its powers are only recommendatoryin nature.

3. It exists in each state as StateHuman Rights Commission.

4. It is mandatory to appoint a womanas a member of the commission.

Which of the above statements arecorrect? (1999)

a) 1, 2, 3 and 4b) 2 and 4 onlyc) 2 and 3 onlyd) 1 and 3 only

9.2.3 National Development Council

16. Which of the following bodiesdoes not/do not find mention inthe Constitution? (2013)

1. National Development Council2. Planning Commission3. Zonal CouncilsSelect the correct answer using thecodes given below.a) 1 and 2 onlyc) 1 and 3 only

Page 101: PolityPreviousYearQuestionBank · 1979. 8. 31.  · PolityPreviousYearQuestionBank PageNo.5 Company can give licence of trade to individuals and its own employees to trade in India:

Polity Previous Year Question Bank

www.laex.in https://elearn.laex.inPage No. 98

b) 2 onlyd) 1, 2 and 3

17. Who among the followingconstitute the National DevelopmentCouncil?1. The Prime Minister2. The Chairman, Finance Commission3. Ministers of the Union Cabinet4. Chief Ministers of the StatesSelect the correct answer using thecodes given below. (2013).a) 1, 2 and 3 onlyb) 1, 3 and 4 onlyc) 2 and 4 onlyd) 1, 2, 3 and 4

9.2.4 National Green Tribunal

18. How is the National GreenTribunal (NGT) different fromthe Central Pollution ControlBoard (CPCB).

1. The NGT has been established by anAct whereas the CPCB has beencreated by an executive order of theGovernment.

2. The NGT provides environmentaljustice and helps reduce the burdenof litigation in the higher courtswhereas the CPCB promotescleanliness of streams and wells, andaims to improve the quality of air inthe country.

Which of the statements given aboveis/are correct?

a) 1 onlyb) 2 onlyc) Both 1 and 2d) Neither 1 nor 2

Key & Explanation1. Answer: CExplanation: Article 148 of theConstitution of India provides for a

Comptroller and Auditor General of India(CAG) to be appointed by the President.CAG is the head of the Indian Audit andAccounts Department. He is the guardianof the public purse and his duty is to upholdthe Constitution and laws in the field offinancial administration. CAG is one of thebulwarks of the democratic system in India.CAG has no control over the issue of moneyfrom the Consolidated Fund. He is onlyconcerned with the audit after expenditurehas already taken place. In reality, CAG inIndia performs the role of an Auditor-General only even though he wasenvisioned to be both the Comptroller andAuditor-General. He doesn’t exercisecontrol over the exchequer on behalf of theParliament when the President of Indiadeclares national emergency/financialemergency.CAG does not have any judicial powers toprosecute those who violate the law whilehandling public money. However, theinvestigative agencies can start prosecutionbased on the observations of the CAG.Charges were filed in the 2G-Spectrum caseand Coal Allocation Case afterinvestigations that were launched based onthe observations of the CAG. Therefore,Information from CAG reports can be usedby investigating agencies to frame chargesagainst those who have violated the lawwhile managing public finances.The function of the Public AccountsCommittee (PAC) is to examine the annualaudit reports submitted by the CAG. PACexamines the audit reports onappropriation accounts, finance accountsand of public undertakings. CAG acts as aguide, friend and philosopher to PAC.Therefore, CAG reports on the execution ofprojects or programmes by the ministriesare discussed by the PAC.Educational Objective: Article 148 of theConstitution of India – CAG to beappointed by the President.CAG has no control over the issue of moneyfrom the Consolidated Fund. He is onlyconcerned with the audit after expenditurehas already taken place.CAG acts as a guide, friend and philosopherto Public Accounts Committee.

Page 102: PolityPreviousYearQuestionBank · 1979. 8. 31.  · PolityPreviousYearQuestionBank PageNo.5 Company can give licence of trade to individuals and its own employees to trade in India:

Polity Previous Year Question Bank

www.laex.in https://elearn.laex.inPage No. 99

Source: Indian Polity by Laxmikant (4thEdition) – Page No: 22.30, 45.1-2

2. Answer: DExplanation: Article 148 of theConstitution of India provides for aComptroller and Auditor General of India(CAG) to be appointed by the President.CAG is the head of the Indian Audit andAccounts Department. He is the guardianof the public purse and his duty is to upholdthe Constitution and laws in the field offinancial administration. CAG is one of thebulwarks of the democratic system in India.Duties and Powers of CAG:i. CAG audits the accounts related to all

expenditure from the ConsolidatedFund of India, Consolidated Fund ofeach state and UT’s having a legislativeassembly.

ii. He audits all expenditure from theContingency Fund of India and thePublic Account of India as well as theContingency Fund and Public Accountof each state.

iii. He audits all trading, manufacturing,profit and loss accounts, balance sheetsand other subsidiary accounts kept byany department of the CentralGovernment and the state governments.

iv. He audits the receipts and expenditureof all bodies and authoritiessubstantially financed from the Centralor State revenues; governmentcompanies; other corporations andbodies, when so required by relatedlaws.

v. He audits the accounts of any otherauthority when requested by thePresident or Governor e.g. Local bodies.

vi. He advises the President with regard toprescription of the form in which theaccounts of the Centre and States shallbe kept.

vii. He submits his audit reports relating tothe accounts of the Centre to thePresident, who shall, in turn, placethem before both the houses ofParliament.

viii. He submits his audit reports relating tothe accounts of a State to the Governor,who shall, in turn, place them beforethe state legislature.

ix. CAG also acts as a guide, friend andphilosopher of the Public AccountsCommittee of the Parliament.

The CAG has no control over the issue ofmoney from the Consolidated Fund. He isonly concerned with the audit afterexpenditure has already taken place. Inreality, CAG in India performs the role ofan Auditor-General only even though hewas envisioned to be both the Comptrollerand Auditor-General. Therefore, he doesn’tcontrol receipt and issue of public money,and to ensure that the public revenue islodged in the exchequer.Educational Objective: Article 148 of theConstitution of India – CAG to beappointed by the President.CAG audits the accounts of both the Centreand the States.CAG has no control over the issue of moneyfrom the Consolidated Fund. He is onlyconcerned with the audit after expenditurehas already taken place.CAG acts as a guide, friend and philosopherto Public Accounts Committee.Source: Indian Polity by Laxmikant (4thEdition) – Page No: 45.1-3.

3. Answer: DExplanation: Election Commissionconsists of a Chief Election Commissionerand some other members (at present, thereare two other members). They areappointed by the President of India.Election Commission of India announcesthe election schedule for the conduct of bothgeneral elections and bye-elections.Splits, mergers and alliances havefrequently disrupted the compositions ofpolitical parties. This has led to a numberof disputes The Election Commission has toresolve these disputes, although itsdecisions can be challenged in the courts.Educational Objectives: Article 324 ofthe Constitution states that the ElectionCommission shall consist of the chief

Page 103: PolityPreviousYearQuestionBank · 1979. 8. 31.  · PolityPreviousYearQuestionBank PageNo.5 Company can give licence of trade to individuals and its own employees to trade in India:

Polity Previous Year Question Bank

www.laex.in https://elearn.laex.inPage No. 100

election commissioner and such number ofother election commissioners, if any, as thepresident may from time to time fix.Source: Indian Polity (Laxmikanth) PageNo: 42.1 and 42.3

4. Answer: CExplanation: Important function of theElection Commission is to allot symbols tothe political parties and the candidates,and also to accord recognition to thepolitical parties. The Commission hasspecified certain symbols as reserved andothers as free. The reserved symbols areonly available for candidates sponsored bythe political pin ties and the free symbolsare equally available to other candidates.Educational Objectives: A new part(Part IV A) has been added to theRepresentation of the People (Amendment)Act, 1951 on registration of political parties.Section 29 A now inserted provides forregistration with the Commission, ofassociations and bodies of individualcitizens of India as political parties forpurpose of this Act.Source: Indian Polity (Laxmikanth) PageNo: 42.3

5. Answer: CExplanation: Article 324 of theConstitution provides that the power ofsuperintendence, direction and control ofelections to parliament, state legislatures,the office of president of India and the officeof Vice President of India shall be vested inthe election commission.Educational Objectives: Article 324provides for the constitution of a bodyknown as the Election Commission of India.University Grants Commission, the CentralVigilance Commission and the NationalHuman Rights Commission have beencreated under different laws of theParliament.Source: Indian Polity (Laxmikanth) PageNo: 42.16. Answer: BExplanation: The Chief ElectionCommissioner and the two other election

commissioners have equal powers andreceive equal salary, allowances and otherperquisites, which are similar to those of ajudge of the Supreme Court. In case ofdifference of opinion amongst the Chiefelection commissioner and/or two otherelection commissioners, the matter isdecided by the Commission by majority.They hold office for a term of six years oruntil they attain the age of 65 years,whichever is earlier. They can resign at anytime or can also be removed before theexpiry of their term.Educational Objectives: The chiefelection commissioner and the two otherelection commissioners have equal powersand receive equal salary, allowances andother perquisites, which are similar tothose of a judge of the Supreme Court.Source: Indian Polity (Laxmikanth) PageNo: 42.2

7. Answer: DExplanation: Only an association or bodyof individual citizens of India calling itself apolitical party and intending to avail itselfof the provisions of Part-IV-A of theRepresentation of the People Act 1951,(relating to registration of political parties)is required to get itself registered with theElection Commission of India.If a political party is treated as a recognisedpolitical party in four or more States, itshall be known as a National Partythroughout the whole of India, but only solong as that political party continues tofulfill thereafter the conditions forrecognition in four or more States on theresults of any subsequent general electioneither to the House of the People or to theLegislative Assembly of any State.Educational Objectives: Article 324 ofthe Constitution grants the powers andfunctions of Election Commission to grantrecognition to political parties and allotelection symbols to them.Source: Indian Polity (Laxmikanth) PageNo: 42.48. Answer: DExplanation: The main function of theFinance Commission is to recommend how

Page 104: PolityPreviousYearQuestionBank · 1979. 8. 31.  · PolityPreviousYearQuestionBank PageNo.5 Company can give licence of trade to individuals and its own employees to trade in India:

Polity Previous Year Question Bank

www.laex.in https://elearn.laex.inPage No. 101

the Union government should share taxeslevied by it with the states. Theserecommendations cover a period of fiveyears. The commission also lays down rulesby which the centre should provide grants-in-aid to states out of the ConsolidatedFund of India. It is also required to suggestmeasures to augment the resources ofstates and ways to supplement theresources of panchayats and municipalities.Educational Objectives: TheConstitution does not make therecommendations of the FinanceCommission binding on the government ofthe day. However, there is a strongprecedent that governments generally go bythe suggestions as far as sharing ofrevenues is concerned. Theserecommendations relating to distribution ofUnion taxes and duties and grants-in-aidare usually implemented by a presidentialorder.Source: Indian Polity (Laxmikanth) Page

No: 45.2

9. Answer: BExplanation: The functions of the FinanceCommission are:(i) Distribution between the Union and

the States of the net proceeds of taxes,which are to be, or may be, dividedbetween them and the allocationamong the states of the respectiveshares of such proceeds.

(ii) To determine the quantum of grants-in-aid to be given by the Centre tostates [Article 275 (1)] and to evolvethe principles to govern the eligibilityof the state for such grant-in-aid;

(iii) Any other matter referred to theCommission by the President of Indiain the interest of sound finance.Several issues like debt relief,financing of calamity relief of states,additional excise duties, etc. havebeen referred to the Commissioninvoking this clause.

Educational Objectives: Article 280 (3)speaks about the functions of the FinanceCommission. The Article states that it shall

be the duty of the Commission to make therecommendations to the President.Source: Indian Polity (Laxmikanth) PageNo: 45.2

10. Answer: AExplanation: Finance Commission is aconstitutional body and it is constituted bythe President. It is set up every five yearsor before. It lays down rules by whichCentre shouldprovide grants in aid to statesout of consolidated fund of India andsuggest measures to increase the resourcesof State.Educational Objectives: According to theArticle 280 of the Constitution, the FinanceCommission of India is required to makerecommendations to the President of Indiaon the principles that should govern thegrants-in-aid to the states by the Centre(i.e., out of the consolidated fund of India).

Source: Indian Polity (Laxmikanth)Page No: 45.2

11. Answer: AExplanation: The primary role of FinanceCommission in India is to act as aninstrument to divide proceeds of divisibletaxes between the states and the Uniongovernment or in cases of taxes that arecollected by the centre but the proceeds ofwhich are allocated between the states, todetermine the principles of such allocation.Educational Objectives: Article 280 (3) ofthe Constitution speaks about the functionsof the Finance Commission. It states that itshall be the duty of the Commission tomake the recommendations to thePresident as to the distribution betweenthe Union and the States of the netproceeds of taxes, which are to be, or maybe, divided between them and theallocation among the states of therespective shares of such proceeds.Source: Indian Polity (Laxmikanth) PageNo: 45.212. Answer: AExplanation: The Finance Commissionsubmits its report to the president. He lays

Page 105: PolityPreviousYearQuestionBank · 1979. 8. 31.  · PolityPreviousYearQuestionBank PageNo.5 Company can give licence of trade to individuals and its own employees to trade in India:

Polity Previous Year Question Bank

www.laex.in https://elearn.laex.inPage No. 102

it before both the Houses of Parliamentalong with an explanatory memorandum asto the action taken on its recommendations.Educational Objectives: The Presidentshall place the Report of the Commissiontogether with an ExplanatoryMemorandum before each house ofParliament.In practice, the recommendations ofFinance Commission are accepted by theGovernment of India for the distribution ofshared tax revenue, as well as for grant-in-aid.Source: Indian Polity (Laxmikanth) PageNo: 45.2.

13. Answer: BExplanation: The PlanningCommission was the organ of planningfor social and economic development.The Prime Minister was the Chairman ofthe Planning Commission, which worksunder the overall guidance of the NationalDevelopment Council. The key function thePlanning Commission was to prepare the 5-year Plan for the most balanced andeffective usage of the country’s resources. Itwas established in 1950 by an executiveresolution of the Government of India.The National Development Council(NDC) presided over by the Prime Minister,is the highest body below theParliament responsible for policymatters with regard to planning forsocial and economic development. TheNational Development Council is one of thekey organizations of the planning system inIndia. It was established in 1952 by anexecutive resolution of the Government ofIndia. The Council comprises the PrimeMinister, all the Union Cabinet Ministers,the Chief Ministers of all States,representatives of the Union Territoriesand the members of the PlanningCommission. Its functions are to considerand approve the national plan prepared bythe Planning Commission and to review theworking of the plan. However, it is only anadvisory body to the Planning Commission.The Secretary of the Planning Commissionalso acts as the Secretary to the NDC.

Entry 20 in the Concurrent List of 7thSchedule deals with economic and socialplanning.Educational Objective: NationalDevelopment Council (NDC) - presided overby the Prime Minister, considers andapproves the national plan prepared by thePlanning Commission and to reviews theworking of the plan.Planning Commission - with the Primeminister as its Chairman, used to preparethe 5-year Plan for the most balanced andeffective usage of the country’s resources.Source: Indian Polity by Laxmikant (4thEdition) – Page No: 49.1 and Appendix-II

14. Answer: CExplanation: The National HumanRights Commission (NHRC) is astatutory body established under theProtection of Human Rights Act of 1993.The NHRC is the watchdog of humanrights in the country. NHRC wasestablished to –i. To strengthen the institutional

arrangements through which humanrights issues could be addressed in theirentirety in a more focussed manner;

ii. To look into allegations of excesses,independently of the government, in amanner that would underline thegovernment’s commitment to protecthuman rights; and

iii. To complement and strengthen theefforts that have already been made toprotect human rights.

According to the recent Protection ofHuman Rights (Amendment) Act of2019,NHRC is a multi-member bodyconsisting of a Chairman and 5 members.The Chairman should be a retired ChiefJustice of India or a retired Judge of theSupreme court,a member should be servingor retired judge of the Supreme court, aserving or retired Chief Justice of a Highcourt and 3 persons having knowledge orpractical experience with respect to humanrights. Of these 3 persons at least oneperson shall be a woman. In addition tothese full-time members, the Commissionalso has 7 ex-officio members – the

Page 106: PolityPreviousYearQuestionBank · 1979. 8. 31.  · PolityPreviousYearQuestionBank PageNo.5 Company can give licence of trade to individuals and its own employees to trade in India:

Polity Previous Year Question Bank

www.laex.in https://elearn.laex.inPage No. 103

Chairmen of National Commission for SCs,National Commission for STs, NationalCommission for Backward Classes,National Commission for the Protection ofChild Rights, National Commission forMinorities, National Commission forwomen and the Chief Commissioner forPersons with Disabilities.Educational Objective: The NHRCis astatutory body established under theProtection of Human Rights Act of 1993.According to the recent Protection ofHuman Rights (Amendment) Act of2019,the Chairman can be a retired ChiefJustice of India or a retired Judge of theSupreme court.Source: Indian Polity by Laxmikant (4thEdition) – page no: 50.1

15. Answer: BExplanation: The National HumanRights Commission (NHRC) is astatutory body established under theProtection of Human Rights Act of 1993.The NHRC is the watchdog of humanrights in the country. NHRC wasestablished to –i. To strengthen the institutional

arrangements through which humanrights issues could be addressed in theirentirety in a more focussed manner;

ii. To look into allegations of excesses,independently of the government, in amanner that would underline thegovernment’s commitment to protecthuman rights; and

iii. To complement and strengthen theefforts that have already been made toprotect human rights.

According to the recent Protection ofHuman Rights (Amendment) Act of2019, NHRC is a multi-member bodyconsisting of a Chairman and 5 members.The Chairman should be a retired ChiefJustice of India or a retired Judge of theSupreme court and a member should beserving or retired judge of the Supremecourt, a serving or retired Chief Justice of aHigh court and 3 persons having knowledgeor practical experience with respect tohuman rights. Of these 3 persons at least

one person shall be a woman. Inaddition to these full-time members, theCommission also has 7 ex-officio members –the Chairmen of National Commission forSCs, National Commission for STs,National Commission for Backward Classes,National Commission for the Protection ofChild Rights, National Commission forMinorities, National Commission forwomen and the Chief Commissioner forPersons with Disabilities.The functions of the Commission aremainly recommendatory in nature. It hasneither the power to punish the violators ofhuman rights nor to award any relief to thevictim. Its recommendations are notbinding on the concerned government orauthority. But, the NHRC should beinformed about the action the action on itsrecommendations within 1 month. TheNHRC submits its annual reports to theCentral and the concerned StateGovernments. These reports are laid beforethe legislature along with a memorandumof action taken on the recommendations ofNHRC and reasons for non-acceptance ofany of such recommendations.The Protection of Human Rights Act of1993 provides for the creation of not onlythe NHRC but also a State Human RightsCommission (SHRC) at the State level.Accordingly, 24 states have establishedSHRCs.Educational Objective: The NHRC is astatutory body established under theProtection of Human Rights Act of 1993.According to the recent Protection ofHuman Rights (Amendment) Act of2019, it is mandatory to appoint a womanas a member of the commission.Source: Indian Polity by Laxmikant (4thEdition) – page no: 50.1-3 and 51.1.

16. Answer: DExplanation: The National DevelopmentCouncil was established in August 1952 byan executive resolution of the Governmentof India on the recommendation of the FirstFive Year Plan. It is neither aconstitutional body nor a statutory body.The Planning Commission was establishedin March 1950 by an executive resolution of

Page 107: PolityPreviousYearQuestionBank · 1979. 8. 31.  · PolityPreviousYearQuestionBank PageNo.5 Company can give licence of trade to individuals and its own employees to trade in India:

Polity Previous Year Question Bank

www.laex.in https://elearn.laex.inPage No. 104

the Government of India. It was neither aconstitutional body nor a statutory body. InIndia, it was the supreme organ of planningfor social and economic development.The Zonal Councils are the statutory (andnot the constitutional) bodies. They areestablished by an Act of the Parliament,that is, States Reorganisation Act of 1956.Educational Objectives: Like theerstwhile Planning Commission, theNational Development Council is neither aconstitutional or statutory body.Zonal councils are statutory (and not theconstitutional) bodies.Source: Indian Polity (Laxmikanth) PageNo: 52.10, 52.8 and 15.4

17. Answer: BExplanation: The National DevelopmentCouncil is composed of the followingmembers: (i) the Prime Minister of India(as its chairman/head). (ii) all UnionCabinet Ministers (since 1967). (iii) theChief Ministers of all the states. (iv) theChief Ministers/administrators of all unionterritories. (v) members of the PlanningCommission (now NITI Aayog).Educational Objectives:National Development Council comprisingPrime Minister, Union cabinet ministers,state chief ministers and Lt Governors ofUTs, was set up in 1952 as the advisorybody which provided advices andrecommendations on the proposals ofPlanning commission.Source: Indian Polity (Laxmikanth) PageNo: 52.10

18. Answer: BExplanation: The National GreenTribunal is established under NationalGreen Tribunal Act in 2010, its mainobjective is to protect the environment,conserve the forest and natural resources.It is guided by the principles of naturaljustice and not laid down the CivilProcedure Code 1908.Central Pollution Control Board isestablished under the Water (Preventionand control of pollution) Act 1974 and

generates their power from Air (Preventionand control of pollution) Act 1981.Educational Objectives: Both theNational Green Tribunal (NGT) andCentral Pollution Control Board (CPCB)are established by the Parliament Acts.Source:https://greentribunal.in/https://cpcb.nic.in/Introduction/

10. Emergency Provisions

10.1 National Emergency

1. Which of the following are notnecessarily the consequences of theproclamation of the President'srule in a State? (2017)

1. Dissolution of the State LegislativeAssembly

2. Removal of the Council of Ministers inthe State

3. Dissolution of the local bodiesSelect the correct answer using the

code below:a) 1 and 2 onlyc) 2 and 3 onlyb) 1 and 3 onlyd) 1,2 and 3

2. If the President of India exerciseshis power as provided underArticle 356 of the Constitution inrespect of a particular State, then(2018)

a) The Assembly of the State isautomatically dissolved.

b) The powers of the Legislature of thatState shall be exercisable by or underthe authority of the Parliament.

c) Article 19 is suspended in that State.d) The President can make laws relating

to that State.

3. Consider the following statements:(2000)

Page 108: PolityPreviousYearQuestionBank · 1979. 8. 31.  · PolityPreviousYearQuestionBank PageNo.5 Company can give licence of trade to individuals and its own employees to trade in India:

Polity Previous Year Question Bank

www.laex.in https://elearn.laex.inPage No. 105

1. The Rajya Sabha alone has the power todeclare that it would be in nationalinterest for the Parliament to legislatewith respect to a matter in the StateList.

2. Resolutions approving the Proclamationof Emergency are passed only by theLok Sabha.

Which of the statements given aboveis/are correct?

a) 1 onlyc) Both 1 and 2b) 2 onlyd) Neither 1 nor 2

10.2 Financial Emergency

4. Consider the following statementsin respect of financial emergencyunder Article 360 of the Constitution(2007)1. A Proclamation of financial emergency

issued shall cease to operate at theexpiration of two months, unless beforethe expiration of that period it has beenof India approved by the resolutions ofboth Houses of Parliament.

2. If any Proclamation of financialemergency is in operation, it iscompetent for the President of India toissue directions for the reduction ofsalaries and allowances of all or anyclass of persons serving in connectionwith the affairs of the Union butexcluding the Judges of Supreme Courtand the High Courts.

Which of the statements given aboveis/are correct?

a) 1 onlyc) Both 1 and 2b) 2 onlyd) Neither 1 nor 2

Key & Explanation1. Answer: BExplanation: It is not required for thePresident to dissolve the State LegislativeAssembly in the event of proclamation of

President’ Rule, he may either suspend ordissolve it.There is no mention about local bodies incase of proclamation of local bodies.So these two are not necessarily theconsequence of the proclamation ofPresident’s Rule.Educational Objective: Article 356 of theConstitution is popularly known as theimposition of President’s Rule in a state.Under this, the President either suspendsor dissolves the state legislative assembly.Source: Indian Polity (Laxmikanth) Pageno: 16.12

2. Answer: BExplanation: If the President uses hispowers provided by the Article 356 of theIndian constitution, then the statelegislature becomes temporarilysuspended. In this case the parliamentuses its authority on issues which were tobe decided by the state legislature. In otherwords parliament can take decisions onsubjects which were under authority ofstate government.Educational Objectives: Under Article356, the President may issue aproclamation to impose emergency in astate if he or she is satisfied on receipt of areport from the Governor of the concernedState, or otherwise, that a situation hasarisen under which the administration ofthe State cannot be carried on according tothe provisions of the constitution.Source: Indian Polity (Laxmikanth) Pageno: 16.12

3. Answer: AExplanation: The Constitution gives thatRajya Sabha may pass a resolution, by alarger part of at the very least two-thirds ofthe Members present and casting a ballotsuch that it is fundamental or convenient inthe national interest that Parliamentshould make law as for any issue countedin the State List. At that point, Parliamentis engaged to make a law regarding thematter indicated in the goals for the entireor any piece of the domain of India.

Page 109: PolityPreviousYearQuestionBank · 1979. 8. 31.  · PolityPreviousYearQuestionBank PageNo.5 Company can give licence of trade to individuals and its own employees to trade in India:

Polity Previous Year Question Bank

www.laex.in https://elearn.laex.inPage No. 106

The proclamation of emergency must beapproved by both Houses of the Parliament,this is a constitutional requirement.Educational Objectives: The 44thAmendment Act of 1978 says that everyresolution approving the proclamation ofemergency or its continuance must bepassed by either House of Parliament by aspecial majority.Source: Indian Polity (Laxmikanth) PageNo: 16.3.

4. Answer: AExplanation: Article 360 (1) laying downthe provisions of Financial Emergency statethat the proclamation of FinancialEmergency shall cease to operate at theexpiration of two months, unless before theexpiration of that period it has beenapproved by resolutions of both Houses ofParliament.The President is empowered to reduce thesalaries of all government officials,including judges of the supreme court andhigh courts, in cases of a financialemergency.Educational Objectives: Article 360 ofthe Constitution provides that if thePresident is satisfied that the financialstability or credit of India or any of its partis in danger, he may declare a state ofFinancial Emergency. Financial Emergencycan operate as long as the situationdemands and may be revoked by asubsequent proclamation.Source: Indian Polity (Laxmikanth) PageNo: 16.18

11.Parliamentary Provisions

1. The Indian parliamentary system isdifferent from the Britishparliamentary system in that Indiahas (1998).

a) Both a real and a nominal executiveb) A system of collective responsibilityc) Bicameral legislatured) The system of judicial review

2. The main advantage of theparliamentary form of governmentis that (2017)

a) The executive and legislature workindependently.

b) It provides continuity of policy and ismore efficient.

c) The executive remains responsible tothe legislature.

d) The head of the government cannot bechanged without election.

3. There is a Parliamentary System ofGovernment in India because the(2015)

a) Lok Sabha is elected directly by thepeople

b) Parliament can amend the Constitutionc) Rajya Sabha cannot be dissolvedd) Council of Ministers is responsible to

the Lok Sabha

4. In the context of India, which ofthe following prínciples is/areimplied institutionally in theparliamentary government (2013).

1. Members of the Cabinet are Members ofthe Parliament.

2. Ministers hold the office till they enjoyconfidence in the Parliament.

3. Cabinet is headed by the Head of theState.

Select the correct answer using thecodes given below.

a) 1 and 2 onlyb) 2 and 3 onlyc) 3 onlyd) 1,2 and 3

Key & Explanation1. Answer: DExplanation: The parliamentary system ofgovernment in India is largely based ontheBritish parliamentary system. However,it never became a replica of theBritishsystem and differs in the following respects:

Page 110: PolityPreviousYearQuestionBank · 1979. 8. 31.  · PolityPreviousYearQuestionBank PageNo.5 Company can give licence of trade to individuals and its own employees to trade in India:

Polity Previous Year Question Bank

www.laex.in https://elearn.laex.inPage No. 107

India has a republican system in placeof British monarchical system. Inotherwords, the Head of the State in India(that is, President) is elected,while theHead of the State in Britain (that is,King or Queen) enjoys ahereditaryposition.

The British system is based on thedoctrine of the sovereignty ofParliament, while the Parliament isnot supreme in India and enjoyslimitedand restricted powers due to a writtenConstitution, federalsystem, judicialreview and fundamental rights.

In Britain, the prime minister should bea member of the Lower House(House ofCommons) of the Parliament. In India,the prime minister maybe a member ofany of the two Houses of Parliament.

Usually, the members of Parliamentalone are appointed as ministersinBritain. In India, a person who is nota member of Parliament can alsobeappointed as minister, but for amaximum period of six months.

Britain has the system of legalresponsibility of the minister whileIndiahas no such system. Unlike inBritain, the ministers in India arenotrequired to countersign the officialacts of the Head of the State.

‘Shadow cabinet’ is an uniqueinstitution of the British cabinet system.Itis formed by the opposition party tobalance the ruling cabinet andtoprepare its members for futureministerial office. There is nosuchinstitution in India.

Educational Objective: The Britishparliamentary system is based on thedoctrine of the sovereignty of Parliament,while the Parliament is not supreme inIndia and enjoys limited and restrictedpowers due to a written Constitution,federal system, judicial review andfundamental rights.Source: Indian Polity by Laxmikant (5thEdition) – Page No: 12.72. Answer: CExplanation: Responsible Government-by its very nature, the parliamentary

system establishes a responsiblegovernment. The ministers are responsibleto the Parliament for all their acts ofomission and commission. The Parliamentexercises control over the ministers throughvarious devices like question hour,discussions, adjournment motion, noconfidence motion, etc.Educational Objective: Theparliamentary system establishes aresponsible government by its very nature.Source: Indian Polity by Laxmikant (5thEdition) – Page No: 12.5

3. Answer: DExplanation: Collective Responsibility- this is the bedrock principle ofparliamentary government. The ministersare collectively responsible to theParliament in general and to the LokSabha in particular (Article 75). They actas a team, and swim and sink together. Theprinciple of collective responsibility impliesthat the Lok Sabha can remove the Councilof Ministers headed by the Prime Ministerfrom office by passing a vote of noconfidence.Educational Objective: The ministersare collectively responsible to theParliament in general and to the LokSabha in particular (Article 75).Source: Indian Polity by Laxmikant (5thEdition) – Page No: 12.4

4. Answer: AExplanation: In the context of India, thefollowing prínciples is/are impliedinstitutionally in the parliamentarygovernment – Collective Responsibility - this is the

bedrock principle of parliamentarygovernment. The ministers arecollectively responsible to theParliament in general and to the LokSabha in particular (Article 75). Theyact as a team, and swim and sinktogether. The principle of collectiveresponsibility implies that the LokSabha can remove the Council ofMinisters headed by the Prime Minister

Page 111: PolityPreviousYearQuestionBank · 1979. 8. 31.  · PolityPreviousYearQuestionBank PageNo.5 Company can give licence of trade to individuals and its own employees to trade in India:

Polity Previous Year Question Bank

www.laex.in https://elearn.laex.inPage No. 108

from office by passing av ote of noconfidence.

Nominal and Real Executives ThePresident is the nominal executive (dejure executive or titular executive)while the Prime Minister is the realexecutive (de facto executive). Thus, thePresident is head of the State, while thePrime Minister is head of thegovernment. Article 74provides for acouncil of ministers headed by thePrime Minister to aid and advise thePresident in the exercise of hisfunctions. The advice so tendered isbinding on the President.Educational Objective: Article 74provides for a council of ministersheaded by the Prime Minister to aidand advise the President in the exerciseof his functions. The advice so tenderedis binding on the President.Source: Indian Polity by Laxmikant(5th Edition) – Page No: 12.3

12. Federalism

12.1Federal and Unitary features ofthe constitution

1. Which one of the following is not afeature of Indian federalism? (2017)a) There is an independent judiciary in

India.b) Powers have been clearly divided

between the Centre and the States.c) The federating units have been given

unequal representation in the RajyaSabha.

d) It is the result of an agreement amongthe federating units

12.2 Centre – State Relations2. Which one of the following Articles

of the Constitution of India saysthat the executive power of everyState shall be so exercised as not toimpede or prejudice the exercise ofthe executive power of the Union?(2004)

a) Article 257b) Article 258c) Article 355d) Article 358

3. The Parliament can make any lawfor the whole or any part India forimplementing internationaltreaties: (2000).

a) With the consent of all the Stateb) With the consent of the majority of

Statesc) With the consent of the States

concernedd) Without the consent of any State

12.3 Division of Powers

4. Which one of the following subjectsis under the Union List in theSeventh Schedule of theConstitution of India? (2006)

a) Regulation of labour and safety inmines and oilfields

b) Agriculturec) Fisheriesd) Public Health

5. With reference to the Constitutionof India, which one of the followingpairs is not correctly matched?(2004).

a) Forests :Concurrent Listb) Stock Exchange:Concurrent Listc) Post Office Savings Bank: Union Listd) Public Health:State List.

12.4 Financial Relations

6. With reference to the FinanceCommission of India, which of thefollowing statements is correct?(2011).

Page 112: PolityPreviousYearQuestionBank · 1979. 8. 31.  · PolityPreviousYearQuestionBank PageNo.5 Company can give licence of trade to individuals and its own employees to trade in India:

Polity Previous Year Question Bank

www.laex.in https://elearn.laex.inPage No. 109

a) It encourages the inflow of foreigncapital for infrastructure development

b) It facilitates the proper distribution offinances among the Public SectorUndertakings

c) It ensures transparency in financialadministration

d) None of the statements

7. Consider the following statements:In India, stamp duties on financialtransactions are:

1. Levied and collected by the StateGovernment

2. Appropriated by the Union Government.Which of these statements is/arecorrect? (2003).a) Only 1b) Only 2c) Both 1 and 2d) Neither 1 nor 2

8. Consider the following statements:The function(s) of the Financecommission is/are:

1. To allow the withdrawal of the moneyout of the Consolidated Fund of India

2. To allocate between the States theshares of proceeds of taxes

3. To consider applications for grants-in-aid from States

4. To supervise and report on whether theUnion and State governments arelevying taxes in accordance with thebudgetary provisions.

Which of these statements is/arecorrect?

a) Only 1b) 2 and 3c) 3 and 4d) 1, 2 and 4

9. Which one of the followingauthorities recommends theprinciples governing the grants-in-

aid of the revenues to the states outof the Consolidated Fund of India?(2002).

a) Finance Commissionb) Inter-State Councilc) Union Ministry of Financed) Public Accounts Committee

10. The primary function of theFinance Commission in India is to:(2000)

a) Distribution of revenue between theCentre and the States

b) Prepare the Annual Budgetc) Advise the President on financial

mattersd) Allocate funds to various ministries of

the Union and State Governments

12.5 Committees/Commissions andtheir Recommendations

11. Which one of the followingsuggested that the Governor shouldbe an eminent person from outsidethe State and should be a detachedfigure without intense politicallinks or should not have taken partin politics in the recent past? (2000).

a) First Administrative ReformsCommission (1966)

b) Rajamannar Committee (1969)c) Sarkaria Commission (1983)d) National Commission to Review the

Working of the Constitution

12.6 Miscellaneous

12. Which of the following bodies doesnot/do not find mention in theConstitution? (2013)

1. National Development Council2. Planning Commission3. Zonal Councils

Page 113: PolityPreviousYearQuestionBank · 1979. 8. 31.  · PolityPreviousYearQuestionBank PageNo.5 Company can give licence of trade to individuals and its own employees to trade in India:

Polity Previous Year Question Bank

www.laex.in https://elearn.laex.inPage No. 110

Select the correct answer using thecodes given below.

a) 1 and 2 onlyb) 2 onlyc) 1 and 3 onlyd) 1, 2 and 3

13. Consider the following statements:1. National Development Council is an

organ of the Planning Commission.2. The Economic and Social Planning is

kept in the Concurrent List in theConstitution of India.

3. The Constitution of India prescribesthat Panchayats should be assigned thetask of preparation of plans foreconomic development and social justice.

Which of the statements given aboveis/are correct? (2013)

a) 1 onlyb) 2 and 3 onlyc) 1 and 3 onlyd) 1, 2 and 3

Key & Explanations1. Answer: D

The Constitution of India establishes afederal system of government.

1. Federal features of the Constitution:2. Dual polity i.e Union at the centre and

states at the periphery3. Written constitution4. Division of powers between Centre and

State5. Supremacy of the constitution6. Rigid constitution7. Independent judiciary8. BicameralismBut, the term ‘Federation’ has nowherebeen used in the Constitution.Article 1, on the other hand, describes Indiaas a ‘Union of States’ which implies twothings: one, Indian Federation is not theresult of an agreement by the states; andtwo, no state has the right to secede fromthe federation.

Source: Laxmikant (Chapter 13)

2. Answer: AArticle 257 of the Constitution of India saysthat the executive power of every Stateshall be exercised as not to impede orprejudice the exercise of the executivepower of the Union. It also states that theexecutive power of the Union shall alsoextend to the giving of directions to a Stateas to the construction and maintenance ofmeans of communication declared in thedirection to be of national or militaryimportance.Article 258 of Constitution of India dealswith Power of the Union to confer powers,etc., on States in certain cases.Article 355 entrusts the duty upon Union toprotect the states against “externalaggression” and “internal disturbance” toensure that the government of every Stateis carried on in accordance with theprovisions of Constitution.Article 358 suspends all the fundamentalrights under Article 19 of the IndianConstitution as soon as emergency isdeclared. It is operated only in the case ofexternal emergency and suspends all thefundamental rights under Article 19 of theIndian Constitution for the entire durationof emergencySource: Indian Constitution

2. Answer: DArticle 215 empowers the High Courts ofthe states to be courts of record andempowers High Courts to punish people fortheir respective contemptArticle 275 makes provisions for statutorygrants to needy states and not every state.These are charged on the ConsolidatedFund of India. The basis of these grants isrecommendations of finance commission.According to Article 325, there shall be onegeneral electoral roll for every territorialconstituency for election to either House ofParliament or to the House or either Houseof the Legislature of a State and no personshall be ineligible for inclusion in any suchroll or claim to be included in any special

Page 114: PolityPreviousYearQuestionBank · 1979. 8. 31.  · PolityPreviousYearQuestionBank PageNo.5 Company can give licence of trade to individuals and its own employees to trade in India:

Polity Previous Year Question Bank

www.laex.in https://elearn.laex.inPage No. 111

electoral roll for any such constituency ongrounds only of religion, race, caste, sex orany of themAccording to Article 355, it is the duty ofthe Union to protect States againstexternal aggression and internaldisturbance.Source: Indian Constitution

3. Answer: DArticle 253-“Legislation for giving effect tointernational agreements Notwithstandinganything in the foregoing provisions of thisChapter, Parliament has power to makeany law for the whole or any part of theterritory of India for implementing anytreaty, agreement or convention with anyother country or countries or any decisionmade at any international conference,association or other body”

Source: Indian Constitution.

4. Answer: ALegislative section of the Constitution ofIndia is divided into three lists: Union List,State List and Concurrent List. These lists,List -I(the Union list), List -II(the State list)and List -III(the Concurrent list) are givenin Seventh Schedule.Union List has presently 100 subjects andoriginally 97 subjects.State List has presently 61 subjects andoriginally 66 subjects.Concurrent List has presently 52 subjectsand originally 47 subjects.42 Constitutional Amendment Act, 1976shifted 5 subjects from the State List to theConcurrent List i.e. Education forests,protection of wild animals and birds,weights and measure and administration ofjustice, constitution and organisation of allcourts except the Supreme Court and theHigh court.Source: Laxmikant appendix 4 (Lists),Centre-State Relations (Chapter 14)

5. Answer: B

Explanation: The Constitution inSchedule 7 has Union, State andconcurrently which divides legislaturepowers between center and states. The 42ndAmendment act of 1976 slightly modifiedthis division of legislative powers bytransferring five subjects from concurrentlist to the state list. They are Education,Forest, weight and Measure, Protection ofWild Life and Administration of Justice.The student is expected to know theimportant items under each of these lists.Source: Laxmikanth Chapter: 14

6. Answer: DThe Finance Commission is a constitutionalbody which is constituted by the Presidentof India to divide Central Government'staxes into all states.So far 14 Finance Commission has beenconstituted and the first FinanceCommission was constituted in 1951.The recommendations of the FinanceCommission are advisory to the CentralGovernment and not binding in nature.Department of Industrial Policy andPromotion (DIPP), Ministry of Commerce &Industry, Government of India makespolicy pronouncements on FDI (inflow offoreign capital).Finance ministry undertakes properdistribution of finances among the PublicSector Undertakings.Comptroller and Auditor General of India(CAG) shall ensure transparency andlegality of its operations and activelypromotes ethical behaviour throughout theorganisation.Source: Current affairs.

7. Answer: DArticle 268: Duties levied by the Union butcollected and appropriated by the StatesArticle 269: Taxes Levied and Collected bythe Centre but Assigned to the States.Stamp duty is nothing but adirect tax levied by the government andpayable under section 3 of the IndianStamp Act, 1899 on all

Page 115: PolityPreviousYearQuestionBank · 1979. 8. 31.  · PolityPreviousYearQuestionBank PageNo.5 Company can give licence of trade to individuals and its own employees to trade in India:

Polity Previous Year Question Bank

www.laex.in https://elearn.laex.inPage No. 112

documented financialtransactions including bills of exchange,letters of credit, promissory notes, letters ofcredit as well as property transactionsParliament has the powers toprescribe stamp duty rates on instrumentssuch as bills of exchange, cheques,promissory notes, bills of lading, letters ofcredit, and insurance policies, transfer ofshares, debentures and proxies.In the case of other instruments, the powerto prescribe rates rests with the stateslegislatureSource: Current affairs and LaxmikanthCentre-State relations (Chapter 14)

8. Answer: BAccording to Article 114, No money canbe withdrawn from the Consolidated Fundof India except under appropriation madeby law (Statement 1 is wrong)Consolidated Fund of India is createdunder Article 266 of the IndianConstitution.The Finance Commission is a constitutionalbody which is constituted by the Presidentof India to divide Central Government'staxes into all states.Commission shall make recommendationsas to the following matters:1. the distribution between the Union and

the States of the net proceeds of taxeswhich are to be, or may be, dividedbetween them and the allocationbetween the States of the respectiveshares of such proceeds (Statement 2 isright)

2. principles which should govern thegrants-in-aid of the revenues of theStates out of the Consolidated Fund ofIndia and the sums to be paid to theStates which are in need of assistanceby way of grants-in-aid of theirrevenues under article 275 of theConstitution (Statement 3 is right)

3. measures needed to augment theConsolidated Fund of a State tosupplement the resources of thePanchayats and Municipalities in theState on the basis of the

recommendations made by the FinanceCommission of the State

The recommendations of the FinanceCommission are advisory to the CentralGovernment and not binding in nature(Statement 4 is wrong).Source: Laxmikanth Parliament (Chapter22), Finance commission (Chapter 45)

9. Answer: AExplanation: besides sharing of taxesbetween center and states the constitutionprovides grants in aid to the states from thecentral resources. The Finance Commissionunder article 275 recommends thestatutory grants.Source: Laxmikanth Chapter -14).

10. Answer: AFinance Commission is created to addressissues of vertical and horizontal imbalancesof federal finances in India. Theconstitutional mandate of the FinanceCommission is:To decide on the proportion of tax revenueto be shared with the States principleswhich should govern the grants-in-aid toStatesIn addition to the core mandate, theFinance Commission is also entrusted withthe responsibility to makerecommendations on various policy issuesand tender advice to the President on anyother matter referred to it in the interest ofsound public finance.Budget is prepared by the Budget Division,Department of Economic Affairs, Ministryof Finance.Source: Laxmikanth Finance commission(Chapter 45)

11. Answer: CThe Sarkaria Commission on Inter-stateRelations has suggested additional criteriafor the selecting a person for the office ofGovernor:1. A politician belonging to the ruling

party at the Centre should not be

Page 116: PolityPreviousYearQuestionBank · 1979. 8. 31.  · PolityPreviousYearQuestionBank PageNo.5 Company can give licence of trade to individuals and its own employees to trade in India:

Polity Previous Year Question Bank

www.laex.in https://elearn.laex.inPage No. 113

appointed as Governor of a State Beingrun by some other party or by acombination of other parties.

2. He must be appointed after consultationwith the Chief Minister of the Stateconcerned.

3. He should be an eminent person insome walk of life,

4. He should be from outside the State anda detached figure not too intimatelyconnected with the local politics of theState,

5. He should be a person who has nottaken active part in politics, generallyand particularly, in the recent past.

6. Persons belonging to the minoritygroups should continue to be given achance as hitherto.

7. His tenure of office must be guaranteedand should not be disturbed except forextremely compelling reasons and if anyaction is to be taken against thegrounds on which he is sought to beremoved. In case of such termination orresignation by the governor, thegovernment should lay before both theHouses of Parliament a statementexplaining the circumstances leading tosuch removal or resignation, as the casemay be;

8. After demitting his office, the personappointed as Governor should not beeligible for any other appointment oroffice of profit under the Union or aState Government except for a secondterm as Governor or election as Vice-President or President of India, as thecase may be; and

9. At the end of his tenure, reasonablepost-retirement benefits should beprovided.

Source: Sarkaria commissionrecommendations.12. Answer : DNational Development council andPlanning commission are neither aconstitutional or statutory body.Zonal councils are statutory and not theconstitutional bodies. They were set up vide

Part-III of 'States Re-organisation Act,1956'Source: Laxmikanth NITI Aayog (Chapter52), Inter-state relations (Chapter 15)

13. Answer: BNational Development Council is anadvisory body to Planning commission thatapproves the Five year plans. The plansmade by Planning commission are placedbefore NDC for its acceptance.243 G states that the legislature of a Statemay, by law endow the panchayats withsuch powers and authorityto prepare plansfor economic development and social justice.It is a voluntary provision.Concurrent list: Economic and SocialPlanningSource: Laxmikanth Appendix 4(Concurrent list), Panchayati Raj (Chapter38)

13. Elections

1. Consider the following statements:(2017)

1. The Election Commission of India is afive-member body.

2. Union Ministry of Home Affairs decidesthe election schedule for the conduct ofboth general elections and bye-elections.

3. Election Commission resolves thedisputes relating to splits/mergers ofrecognised political parties.

Which of the statements given aboveis/are correct?

a) 1 and 2 onlyb) 2 and 3 onlyc) 2 onlyd) 3 only

2. With reference to the DelimitationCommission, consider the followingstatements: (2012).

1. The orders of the DelimitationCommission cannot be challenged in aCourt of Law.

Page 117: PolityPreviousYearQuestionBank · 1979. 8. 31.  · PolityPreviousYearQuestionBank PageNo.5 Company can give licence of trade to individuals and its own employees to trade in India:

Polity Previous Year Question Bank

www.laex.in https://elearn.laex.inPage No. 114

2. When the orders of the DelimitationCommission are laid before the LokSabha or State Legislative Assembly,they cannot effect any modifications inthe orders.

Which of the statements given aboveis/are correct?

a) 1 onlyb) 2 onlyc) Both 1 and 2d) Neither 1 nor 2

3. Consider the following tasks:1. Superintendence, direction and conduct

of free and fair elections.2. Preparation of electoral rolls for all

elections to the Parliament, StateLegislatures and the Office of theresident and the Vice-President.

3. Giving recognition to political partiesand allotting election symbols topolitical parties and individualscontesting the election.

4. Proclamation of final verdict in the caseof election disputes.

Which of the above are the functions ofthe Election Commission of India?(2004)

a) 1,2 and 3 onlyb) 1 and 3onlyc) 2,3 and 4 onlyd) 1,2 and 4 only

4. Consider the following statementsabout the recent amendments tothe Election Law by theRepresentation of the People(Amendment) Act, 1996:

1. Any conviction for the offence ofinsulting the Indian national flag or theConstitution of India shall entaildisqualification for contesting electionsto Parliament and State legislatures forsix years from the date of conviction.

2. There is an increase in the securitydeposit which a candidate has to maketo contest the election to the Lok Sabha.

3. A candidate cannot now stand forelection from more than oneParliamentary constituency.

4. No election will now be countermandedon the death of a contesting candidate.

Which of the above statements arecorrect? (1999)

a) 2 and 3 onlyb) 1,2 and 4 onlyc) 1 and 3 onlyd) 1,2, 3 and 4

5. The state which has the largestnumber of seats reserved for theScheduled Tribes in Lok Sabha is

a) Biharb) Gujratc) Uttar Pradeshd) Madhya Pradesh

Key and Explanations1. Answer: DExplanation: Article 324 of theConstitution of India states that theElection Commission shall consist of thechief election commissioner and suchnumber of other election commissioners, ifany, as the president may from time to timefix. At present, the Election Commissionconsists of a Chief Election Commissionersand two other Election Commissioners.The Election Commission of India decidesthe election schedule for the conduct of bothgeneral elections and bye-elections. It alsoresolves the disputes relating tosplits/mergers of recognised politicalparties.Educational Objective: Article 324 of theConstitution of India states that theElection Commission shall consist of thechief election commissioner and suchnumber of other election commissioners, ifany, as the president may from time to timefix.Source: Indian Polity by Laxmikant (4thEdition) – Page No: 42.3-5.

Page 118: PolityPreviousYearQuestionBank · 1979. 8. 31.  · PolityPreviousYearQuestionBank PageNo.5 Company can give licence of trade to individuals and its own employees to trade in India:

Polity Previous Year Question Bank

www.laex.in https://elearn.laex.inPage No. 115

2. Answer: CExplanation: The DelimitationCommission is established by theGovernment of India under the provisionsof the Delimitation Commission Act. Themain task of the Commission is to redrawboundaries of both Assembly and LokSabha constituencies. The representationfrom each State is not changed however,the number of SC and ST seats in a Stateare changed in accordance with the census.The orders of the Commission cannot bechallenged in a court of law. The Lok Sabhaand the State Legislatures does not havethe power to modify these orders. TheDelimitation Commission has been setuponly 4 times in the past (1952,1963, 1973and 2002).Eucational Objective: DelimitationCommission of India is a statutory bodythat redraws boundaries of both Assemblyand Lok Sabha constituencies.Source: Indian Polity by Laxmikant (5thEdition) – Page No: 22.33. Answer: AExplanation: The powers and functions ofthe Election Commission with regard toelections to the Parliament, statelegislatures and offices of President andVice-President are –i. To determine the territorial areas of the

electoral constituencies throughout thecountry on the basis of the DelimitationCommission Act of Parliament.

ii. To prepare and periodically reviseelectoral rolls and to register all eligiblevoters.

iii. To notify the dates and schedules ofelections and to scrutinize nominationpapers.

iv. To grant recognition to political partiesand allot election symbols to them.

v. To act as a court for settling disputesrelated to granting of recognition topolitical parties and allotment ofelection symbols to them.

vi. To appoint officers for inquiring in todisputes relating to electoralarrangements.

vii. To determine the code of conduct to beobserved by the parties and thecandidates at the time of elections.

viii. To prepare a roster for publicity of thepolicies of the political parties on radioand TV in times of elections.

ix. To advise the president on mattersrelating to the disqualifications of themembers of Parliament.

x. To advise the governor on mattersrelating to the disqualifications of themembers of state legislature.

xi. To cancel polls in the event of rigging,booth capturing, violence and otherirregularities.

xii. To request the president or the governorfor requisitioning the staff necessary forconducting elections.

xiii. To supervise the machinery of electionsthroughout the country to ensure freeand fair elections.

xiv. To advise the president whetherelections can be held in a state underpresident’s rule in order to extend theperiod of emergency after one year.

xv. To register political parties for thepurpose of elections and grant them thestatus of national or state parties on thebasis of their poll performance.

The disputes regarding the election areadjudicated by the respective State Highcourts and the Supreme court in case ofelection to the President of India.Educational Objective: The disputesregarding the election are adjudicated bythe respective State High courts and theSupreme court in case of elcetion to thePresident of India.Source: Indian Polity by Laxmikant (5thEdition) – Page No: 42.4

4. Answer: BExplanation: The Representation of thePeople (Amendment) Act of 1996 restrictedthe number of seats a candidate can contestfrom to 2 seats.Educational Objective: TheRepresentation of the People (Amendment)

Page 119: PolityPreviousYearQuestionBank · 1979. 8. 31.  · PolityPreviousYearQuestionBank PageNo.5 Company can give licence of trade to individuals and its own employees to trade in India:

Polity Previous Year Question Bank

www.laex.in https://elearn.laex.inPage No. 116

Act of 1996 restricted the number of seats acandidate can contest from to 2 seats.Source: Indian Polity by Laxmikant (5thEdition) – Page No: not available

5. Answer : DExplanation: The State which has thelargest number of seats reserved for theScheduled Tribes in Lok Sabha is MadhyaPradesh with 6 seats.Educational Objective: Madhya pradeshhas the maximum number of seats reservedfor STs in the Lok Sabha.Source: Indian Polity by Laxmikant (5thEdition) – Page No: not availableAmendment (Questions are basicallyfactual)

14. Amendment of Constitution

1. Consider the following statements:1. The 44th Amendment to the

Constitution of India introduced anArticle placing the election of the PrimeMinister beyond judicial review.

2. The Supreme Court of India struckdown the 99th Amendment to theConstitution of India as being violativeof the independence of judiciary.

Which of the statements given aboveis/are correct? (2019)

a) 1 onlyb) 2 onlyc) Both 1 and 2d) Neither 1 nor 2

2. The Ninth Schedule was introducedin the Constitution of India duringthe prime Ministership of? (2019).

a) Jawaharlal Nehrub) Indira Gandhic) Lal Bahadur Shastrid) Morarji Desai

3. Which of the followingConstitutional Amendments are

related to raising the number ofMembers of Lok Sabha be electedfrom the States? (2003).

a) 6th and 22nd

b) 7th and 31st

c) 13th and 38th

d) 11th and 42nd

4. Which one of the followingamendments to the IndianConstitution empowers thePresident to send back any matterfor reconsideration by the Councilof Ministers? (2002).

a) 39th

b) 40th

c) 42nd

d) 44th

5. The 86th ConstitutionalAmendment deals with the (2002).

a) Continuation of reservation forbackward classes in governmentemployment.

b) Free and compulsory education for allchildren between the age of 6 and 14years.

c) Reservation of 30 percent posts forwomen in government recruitments.

d) Allocation of more number ofparliamentary seats for recently createdStates.

6 . Match List-I with List-II and selectthe correct answer using the codesgiven below the Lists: (2001).

List I(Amendments tothe Constitution)

List II(Contents)

A. The Constitution(Sixty-ninthAmendment)Act, 1991

1. Establishmentof state levelRentTribunals.

B. The Constitution(Seventy fifth

2. Noreservations

Page 120: PolityPreviousYearQuestionBank · 1979. 8. 31.  · PolityPreviousYearQuestionBank PageNo.5 Company can give licence of trade to individuals and its own employees to trade in India:

Polity Previous Year Question Bank

www.laex.in https://elearn.laex.inPage No. 117

Amendment)Act, 1994.

for ScheduledCastes inPanchayats inArunachalPradesh.

C. The Constitution(EightiethAmendment)Act, 2000.

3. Constitution oflevelsPanchayats invillages or atother local.

D. The Constitution(Eighty-thirdAmendment)Act, 2000

4. Accepting therecommendations of the TenthFinanceCommission.

Accorded thestatus of Delhi.

Choose the correct answer using thecodes given below:

a) A-5; B-1; C-4; D-2b) A-1; B-5; C-3; D-4c) A-5; B-1; C-3; D-4d) A-1; B-5; C-4; D-27. The 73rd Constitution Amendment

Act 1992 refersa) Generation of gainful employment for

the unemployed and the under-employment men and women in ruralarea.

b) Generation of employment for the ablebodied adults who are in need anddesirous of work during the leanagricultural season.

c) Laying the foundation for strong andvibrant Panchayati Raj Institutions inthe country.

d) Guarantee of right to life, liberty andsecurity of person, equality before lawand equal protection withoutdiscrimination

8. Consider the following statements:An amendment to the Constitutionof India can be initiated by the(1999)

1. Lok Sabha2. Rajya Sabha

3. President4. State Legislatures

Which of the above statements is/arecorrect?

a) 1 onlyb) 2,3 and 4 onlyc) 1,2 and 3 onlyd) 1 and 2 only

9. Consider the following statements:(2013).

1. An amendment to the Constitution ofIndia can be initiated by anintroduction of a bill in the Lok Sabhaonly.

2. If such an amendment seeks to makechanges in the federal character of theconstitution, the amendment alsorequires to be ratified by the legislatureof all the states of India.

Which of the statements given aboveis/are correct?

a) 1 onlyb) 2 onlyc) Both 1 and 2d) Neither 1 nor 2

10. The Constitution (Seventy-ThirdAmendment) Act, 1992, which aimsat promoting the Panchayati RajInstitutions in the country,provides for which of thefollowing?(2011).

1. Constitution of District PlanningCommittees.

2. State Election Commissions to conductall panchayat elections.

3. Establishment of State FinanceCommissions.

Select the correct answer using thecodes

a) 1 only

Page 121: PolityPreviousYearQuestionBank · 1979. 8. 31.  · PolityPreviousYearQuestionBank PageNo.5 Company can give licence of trade to individuals and its own employees to trade in India:

Polity Previous Year Question Bank

www.laex.in https://elearn.laex.inPage No. 118

b) 1 and 2 onlyc) 2 and 3 onlyd) 1, 2 and 3

11. Under which one of the followingConstitution Amendment Acts, fourlanguages were added to thelanguages under the EighthSchedule of the Constitution ofIndia, thereby raising their numberto 22?

a) Constitution (Ninetieth Amendment)Act

b) Constitution (Ninety-first Amendment)Act

c) Constitution (Ninety-secondAmendment) Act

d) Constitution (Ninety-third Amendment)Act

12. Which of the following ConstitutionAmendment seeks that the size ofthe Councils of Ministers at theCentre and in a State must notexceed15 per cent of the totalnumber of members in the LokSabha and the total number ofmembers of the LegislativeAssembly of that State, respectively?

a) 91st

b) 93rd

c) 95th

d) 97th

13. What does the 104th ConstitutionAmendment Bill relate to? (2006)

a) Abolition of Legislative Councils incertain States.

b) Introduction of dual citizenship forpersons of Indian origin living outsideIndia.

c) Providing quota to socially andeducationally backward classes inprivate Educational institutions.

d) Providing quota for religious minoritiesin services under the centralgovernment.

14. The Constitution (99th Amendment)Act is related to:

a) Empowering the Centre to levy andappropriate service tax.

b) The constitution of the NationalJudicial Commission.

c) Readjustment of electoral constituencieson the basis of the Population Census2001.

d) The demarcation of new boundariesbetween States.

15. The Ninth Schedule to the IndianConstitution was added by (2003)

a) First Amendmentb) Ninth Amendmentc) Eighth Amendmentd) Forty second Amendment

Key and Explanations1. Answer: BExplanation: The 42nd ConstitutionalAmendment Act of 1976 has placed theelection to the President, Vice-Presidentand the Prime Minister beyond judicialreview. However, this provision wasrepealed by the 44th ConstitutionAmendment Act of 1978.99th Constitution Amendment Act of2014 has replaced the collegium system ofappointing judges to the Supreme Courtand High Courts with a new body called theNational Judicial AppointmentsCommission (NJAC). However, in 2015, theSupreme Court has declared thisamendment act as unconstitutional andvoid as it was violative of the independenceof judiciary. Consequently, the earliercollegium system became operative again.Educational Objective: The 42ndConstitutional Amendment Act of 1976 hasplaced the election to the President, Vice-President and the Prime Minister beyondjudicial review.In 2015, the Supreme Court has declared99th Constitution Amendment Act of 2014as unconstitutional and void as it wasviolative of the independence of judiciary.

Page 122: PolityPreviousYearQuestionBank · 1979. 8. 31.  · PolityPreviousYearQuestionBank PageNo.5 Company can give licence of trade to individuals and its own employees to trade in India:

Polity Previous Year Question Bank

www.laex.in https://elearn.laex.inPage No. 119

Source: Indian Polity by Laxmikant (5thEdition) – Appendix VI Page No: 4 and 9

2. Answer: AExplanation: 1st ConstitutionAmendment Act of 1951 enactedduring the prime ministership ofJawaharlal Nehru, has made thefollowing changes to the Constitution ofIndia - Empowered the state to make special

provisions for the advancement ofsocially and economicallybackwardclasses (Article 15).

Provided for the saving of lawsproviding for acquisition of estates, etc(Article 31A).

Added Ninth Schedule to protect theland reforms and other laws included init from the judicial review.

Added three more grounds ofrestrictions on freedom of speech andexpression, viz., public order, friendlyrelations with foreign states andincitement to an offence. Also, made therestrictions ‘reasonable’ and thus,justiciable in nature (Article 19).

Provided that state trading andnationalisation of any trade or businessby the state is not to be invalid on theground of violation of the right to tradeor business (Article 19).Educational Objective: The IXthSchedule to the Constitution of Indiawas added by the 1st ConstitutionAmendment Act of 1951.Source: Indian Polity by Laxmikant(5th Edition) – Appendix VI Page No: 1

3. Answer: BExplanation: The 7th ConstitutionalAmendment Act of 1956 capped themaximum number of elected seats at 520—500 from India’s states and 20 from its UTs.The 31st Constitution Amendement Actof 1972 increased the number of Lok Sabhaseats from 525 to 545.At present the maximum strenght of theLok Sabha has been fixed at 552.

Educational Objective: The 7thConstitutional Amendment Act of 1956capped the maximum number of electedseats at 520. The 31st ConstitutionAmendement Act of 1972 increased thenumber of Lok Sabha seats from 525 to 545.Source: Indian Polity by Laxmikant (5thEdition) – Appendix VI Page No: 1 and 3

4. Answer: DExplanation: The 42nd ConstitutionalAmendment Act of 1976 (enacted by theIndira Gandhi Government) made thePresident bound by the advice of thecouncil of ministers headed by the primeminister16. The 44th ConstitutionalAmendment Act of 1978 (enacted by theJanata Party Government headed byMorarji Desai) authorised the President torequire the council of ministers toreconsider such advice either generally orotherwise. However, he ‘shall’ actinaccordance with the advice tendered aftersuch reconsideration. In other words, thePresident may return a matter once forreconsideration of his ministers, but thereconsidered advice shall be binding.Educational Objective: The 44thConstitutional Amendment Act of 1978authorised the President to require thecouncil of ministers to reconsider the advicetendered.Source: Indian Polity by Laxmikant (5thEdition) – Page No: 17.14

5. Answer: BExplanation: The 86th ConstitutionalAmendment Act of 2002 has thefollowing provisions – Made elementary education a

fundamental right. The newly-addedArticle 21A declares that “the Stateshall provide free and compulsoryeducation to all children of the age of 6to 14 years in such manner as the Statemay determine”.

Changed the subject matter of Article45 in Directive Principles. It nowreads—“The State shall endeavour toprovide early childhood care and

Page 123: PolityPreviousYearQuestionBank · 1979. 8. 31.  · PolityPreviousYearQuestionBank PageNo.5 Company can give licence of trade to individuals and its own employees to trade in India:

Polity Previous Year Question Bank

www.laex.in https://elearn.laex.inPage No. 120

education for all children until theycomplete the age of six years”.

Added a new fundamental duty underArticle 51Awhich reads—“It shall bethe duty of every citizen of India who isa parent or guardian to provideopportunities for education to his childor ward between the age of 6 and 14years”.

Educational Objective: The 86thConstitutional Amendment Act of 2002 hasmade elementary education a fundamentalright. The newly-added Article 21Adeclares that “the State shall provide freeand compulsory education to all children ofthe age of 6 to 14 years in such manner asthe State may determine”.Source: Indian Polity by Laxmikant (5thEdition) – Appendix VI Page No: 8

6. Answer: AExplanation: The 69th ConstitutionAmendment Act of 1991 accorded aspecial status to the Union Territory ofDelhi by designing it as the NationalCapital Territoryof Delhi. The amendmentalso provided for the creation of a 70-member legislative assembly and a 7-member council of ministers for Delhi.The 75th Constitution Amendment Actof 1994 provided for the establishment ofrent tribunals for the adjudication ofdisputes with respect to rent, its regulationand control and tenancy issues includingthe rights, title and interest of landlordsand tenants.The 80th Constitution Amendment Actof 2000 provided for an ‘alternative schemeof devolution’ of revenue between theCentre and states. This was enacted on thebasis of the recommendations of the TenthFinance Commission which hasrecommended that out of the total incomeobtained from Central taxes and duties,29% should be distributed among the states.The 83rd Constitution Amendment Actof 2000 provided that no reservation inpanchayats need be made for SCs inArunachal Pradesh. The total population ofthe state is tribal and there are no SCs.

Educational Objective: The 69thConstitution Amendment Act of 1991 -special status to the Union Territory ofDelhi.The 75th Constitution Amendment Actof 1994 - the establishment of renttribunals for the adjudication of disputeswith respect to rent.The 80th Constitution Amendment Actof 2000 – was enacted on the basis of therecommendations of the Tenth FinanceCommission which has recommended thatout of the total income obtained fromCentral taxes and duties, 29% should bedistributed among the states.The 83rd Constitution Amendment Actof 2000 - no reservation in panchayatsneed be made for SCs in ArunachalPradesh.Source: Indian Polity by Laxmikant(5th Edition) – Appendix VI Page No: 6-7

7. Answer:Explanation: The 73rd ConstitutionAmendment Act of 1992 grantedconstitutional status and protection to thepanchayati raj institutions. For thispurpose, the amendment has addeda newPart-IX (Article 243 to Article 243O)entitled as ‘the panchayats’ and a new XISchedule containing 29functional items ofthe panchayats.Educational Objective: The 73rdConstitution Amendment Act of 1992granted constitutional status andprotection to the panchayati raj institutions.Source: Indian Polity by Laxmikant (5thEdition) – Page No: 38.7

8. Answer: DExplanation: Article 368 of theConstitution of India states that anamendment of the Constitution can beinitiated only by the introduction of a billfor the purpose in either House ofParliament and not in the state legislatures.Educational Objective: Article 368of theConstitution of India states that anamendment of the Constitution can beinitiated only by the introduction of a bill

Page 124: PolityPreviousYearQuestionBank · 1979. 8. 31.  · PolityPreviousYearQuestionBank PageNo.5 Company can give licence of trade to individuals and its own employees to trade in India:

Polity Previous Year Question Bank

www.laex.in https://elearn.laex.inPage No. 121

for the purpose in either House ofParliament and not in the state legislatures.Source: Indian Polity by Laxmikant (5thEdition) – Page No: 10.1

9. Answer: DExplanation: Article 368 of theConstitution of India provides for two typesof amendments, that is, by a specialmajority of Parliament and also throughthe ratification of half of the states by asimple majority. But, some other articlesprovide for the amendment of certainprovisions of the Constitution by a simplemajority of Parliament, that is, a majorityof the members of each House present andvoting (similar to the ordinary legislativeprocess). Notably, these amendments arenot deemed tobe amendments of theConstitution for the purposes of Article 368.Article 368 states that, if the bill seeks toamend the federal provisions of theConstitution, it must also be ratified bythe legislatures of half of the states bya simple majority, that is, a majority ofthe members of the House present andvoting.Article 368 states that an amendment ofthe Constitution can be initiated only bythe introduction of a bill for the purpose ineither House of Parliament and not inthe state legislatures.Educational Objective: Article 368 statesthat an amendment of the Constitution canbe initiated only by the introduction of abill for the purpose in either House ofParliament and not in the state legislaturesand if the bill seeks to amend the federalprovisions of the Constitution, it must alsobe ratified by the legislatures of half of thestates by a simple majority.Source: Indian Polity by Laxmikant (5thEdition) – Page No: 10.1-2.

10. Answer: CExplanation: The 73rd ConstitutionAmendment Act of 1992 grantedconstitutional status and protection to thepanchayati raj institutions. For thispurpose, the amendment has added a newPart-IX (Article 243 to Article 243O)

entitled as ‘the panchayats’ and a new XISchedule containing 29functional items ofthe panchayats.Article 243K of the Constitution of Indiastates that the superintendence, directionand control of the preparation of electoralrolls and the conduct of all elections to thepanchayats shall be vested in the stateelection commission. It consists of aqwstate election commissioner to be appointedby the governor. The state legislature maymake provision with respect to all mattersrelating to elections to the panchayats.Article 243I of the Constitution of Indiastates that the governor of a state shall,after every five years, constitute a financecommission to review the financial positionof the panachayats. It shall make thefollowing recommendations to the Governor:

i. The principles that should govern:a) The distribution between the state and

the panchayats of the net proceeds ofthe taxes, duties, tolls and fees leviedby the state.

b) The determination of taxes, duties, tollsand fees that may beassigned to thepanchayats.

c) The grants-in-aid to the panchayatsfrom the consolidated fund of the state.

I. The measures needed to improve thefinancial position of the panchayats.

II. Any other matter referred to it by thegovernor in the interests of soundfinance of the panchayats.

The state legislature may provide for thecomposition of the commission, the requiredqualifications of its members and themanner of their selection. The governorshall place the recommendations of thecommission alongwith the action takenreport before the state legislature.TheCentral Finance Commission shall alsosuggest the measures needed to augmentthe consolidated fund of a state tosupplement the resources of the panchayatsin the states (on the basis of therecommendations made by the financecommission of the state).No provision has been made by the 73rdConstitutional Amendment Act toconstitute District planning committees.

Page 125: PolityPreviousYearQuestionBank · 1979. 8. 31.  · PolityPreviousYearQuestionBank PageNo.5 Company can give licence of trade to individuals and its own employees to trade in India:

Polity Previous Year Question Bank

www.laex.in https://elearn.laex.inPage No. 122

Educational Objective: The 73rdConstitution Amendment Act of 1992granted constitutional status andprotection to the panchayati raj institutions.Article 243K - superintendence, directionand control of the preparation of electoralrolls and the conduct of all elections to thepanchayats shall be vested in the stateelection commission.Article 243I - the governor of a state shall,after every five years, constitute a financecommission to review the financial positionof the panachayats.Source: Indian Polity by Laxmikant (5thEdition) – page no: 38.9

11. Answer: CExplanation: The 92nd ConstitutionAmendment Act of 2003 included four morelanguages in the VIII Schedule to theConstitution of India. They are Bodo, Dogri(Dongri), Mathilli (Maithili)and Santhali.With this, the total number ofconstitutionally recognised languagesincreased to 22.Educational Objective: The 92ndConstitution Amendment Act of 2003included Bodo, Dogri (Dongri), Mathilli(Maithili)and Santhali in the VIII Scheduleto the Constitution of India.Source: Indian Polity by Laxmikant (5thEdition) – Appendix VI page no: 8

12. Answer: AExplanation: The 91st ConstitutionAmendment Act of 2003 made thefollowing provisions to limit the size ofCouncil of Ministers, to debar defectorsfrom holding public offices, and tostrengthen the anti-defection law: The total number of ministers,

including the Prime Minister, in theCentral Council of Ministers shall notexceed 15% of the total strength of theLok Sabha(Article 75(1A)).

A member of either house of Parliamentbelonging toany political party who isdisqualified on the ground of defectionshall also be disqualified to beappointed as a minister (Article 75(1B)).

The total number of ministers,including the Chief Minister, in theCouncil of Ministers in a state shall notexceed 15% of the total strength of thelegislative Assembly of that state. But,the number of ministers, including theChief Minister, in a state shall not beless than 12 (Article 164(1A)).

A member of either House of a statelegislature belonging to any politicalparty who is disqualified on the groundof defection shall also be disqualified tobe appointed as a minister (Article164(1B)).

A member of either House ofParliament or either House of a StateLegislature belonging to any politicalparty who is disqualified on the groundof defection shall also be disqualified tohold any remunerative political post.The expression “remunerative politicalpost” means –i. Any office under the central

government or a state governmentwhere the salary or remunerationfor such office is paid out of thepublic revenue of the concernedgovernment; or

ii. Any office under a body, whetherincorporated or not, which is whollyor partially owned by the centralgovernment or a state governmentand the salary or remuneration forsuch office is paid by such body,except where such salary orremuneration paid is compensatoryin nature (Article361B).

The provision of the Tenth Schedule(anti-defection law) pertaining toexemption from disqualification incaseof split by one-third members oflegislature party has been deleted. Itmeans that the defectors doesn’t haveprotection on grounds of splits anymore.Educational Objective: The 91stConstitution Amendment Act of 2003states that

the total number of ministers, includingthe Prime Minister, in the CentralCouncil of Ministers shall not exceed15% of the total strength of the LokSabha(Article 75(1A))

Page 126: PolityPreviousYearQuestionBank · 1979. 8. 31.  · PolityPreviousYearQuestionBank PageNo.5 Company can give licence of trade to individuals and its own employees to trade in India:

Polity Previous Year Question Bank

www.laex.in https://elearn.laex.inPage No. 123

The total number of ministers,including the Chief Minister, in theCouncil of Ministers in a state shall notexceed 15% of the total strength of thelegislative Assembly of that state. But,the number of ministers, including theChief Minister, in a state shall not beless than 12 (Article 164(1A)).Source: Indian Polity by Laxmikant(5th Edition) – Appendix VI Page No: 8.

13. Answer : CExplanation: The 104th ConstitutionAmendment Bill empowered the state tomake special provisions for the socially andeducationally backward classes or theScheduled Castes or the Scheduled Tribesin educational institutions includingprivate educational institutions (whetheraided or unaided by the state),except theminority educational institutions (Article15(5)). This Amendment was enacted tonullify the Supreme Court judgement inthe Inamdar case(2005) where the apexcourt ruled that the state cannot impose itsreservation policy on minority andnonminority unaided private colleges,including professional colleges. The courtdeclared that reservation in private,unaided educational institutions wasunconstitutional. This became the 93rdConstitution Amendment Act of 2005.Educational Objective: 93rd ConstitutionAmendment Act of 2005 empowers thestate to make special provisions for thesocially and educationally backward classesor the Scheduled Castes or the ScheduledTribes in educational institutions includingprivate educational institutions (whetheraided or unaided by the state),except theminority educational institutions (Article15(5)).Source: Indian Polity by Laxmikant(5th Edition) – Appendix VI Page No: 9

14. Answer: BExplanation: 99th ConstitutionAmendment Act of 2014 has replaced thecollegium system of appointing judges tothe Supreme Court and High Courts with anew body called the National Judicial

Appointments Commission (NJAC).However, in 2015, the Supreme Court hasdeclared this amendment act asunconstitutional and void as it is violativeof independence of judiciary.Educational Objective: 99thConstitutionAmendment Act of 2014 has replaced thecollegium system of appointing judges tothe Supreme Court and High Courts with anew body called the National JudicialAppointments Commission(NJAC).Source: Indian Polity by Laxmikant (5thEdition) – Appendix VI Page No: 9

15 .Answer: AExplanation: 1st ConstitutionAmendment Act of 1951 has made thefollowing changes to the Constitution ofIndia - Empowered the state to make special

provisions for the advancement ofsocially and economically backwardclasses (Article 15).

Provided for the saving of lawsproviding for acquisition of estates, etc(Article 31A).

Added Ninth Schedule to protect theland reforms and other laws included init from the judicial review.

Added three more grounds ofrestrictions on freedom of speech andexpression, viz., public order, friendlyrelations with foreign states andincitement to an offence. Also, made therestrictions ‘reasonable’ and thus,justiciable in nature (Article 19).

Provided that state trading andnationalisation of any trade or businessby the state is not to be invalid on theground of violation of the right to tradeor business (Article 19).Educational Objective: The IXthSchedule to the Constitution of Indiawas added by the 1st ConstitutionAmendment Act of 1951.Source: Indian Polity by Laxmikant(5th Edition) – Appendix VI Page No:

Page 127: PolityPreviousYearQuestionBank · 1979. 8. 31.  · PolityPreviousYearQuestionBank PageNo.5 Company can give licence of trade to individuals and its own employees to trade in India:

Polity Previous Year Question Bank

www.laex.in https://elearn.laex.inPage No. 124

15. Local Government

1. Local self –government can be bestexplained as an exercise in

a) Federalismb) Democratic decentralizationc) Administrative delegationd) Direct Democracy

2. With reference to the ‘GramNyayalaya Act’ which of thefollowing statements is/are correct?

1. As per the Act, Gram Nyayalayas canhear only civil cases and not criminal cases.2. The Act allows local social activists asmediators/reconciliation.Select the correct answer using the codegiven below.a) 1 onlyb) 2 onlyc) Both 1d) Neither 1 nor 2

3. Consider the followingstatements:1. The minimum age prescribed for anyperson to be a member of Panchayat is 25years.2. A Panchayat reconstituted afterpremature dissolution continues only forthe remainder period.Which of the statements given aboveis/are correct?a) 1 onlyb) 2 onlyc) Both 1d) Neither 1 nor 2

4. The fundamental object ofPanchayati Raj system is to ensurewhich among the following?1. People’s participation indevelopment2. Political accountability3. Democratic decentralization4. Financial mobilizationSelect the correct answer using the codegiven below.

a) 1, 2 and 3 onlyb) 2 and 4 onlyc) 1 and 3 onlyd) 1, 2, 3 and 4

5. The Government enacted thepanchayat Extension to ScheduledAreas (PESA) Act in 1996. Which of theone following is not identified as itsobjective?a) To Provide self-governmentb) To recognize traditional rightsc) To Create autonomous recognize in

tribal areasd) To free tribal people from

exploitation

6. In the areas covered under thepanchayat (Extension to theScheduled Areas) Act, 1996, What isthe role/ power of Gram Sabha?1. Gram Sabha has the power toprevent alienation of land in the ScheduledAreas.2. Gram Sabha has the ownership ofminor forest produce.3. Recommendation of Gram Sabha idrequired for granting prospecting license ormining lease for any mineral in theScheduled Areas.Which of the statement given belowis/are correct?a) 1 onlyb) 1 and 2 onlyc) 2 and 3 onlyd) 1,2 and 3

7. Which one of the followingauthorities makes recommended to theGovernor of a State as to theprincipals for determining the taxesand duties, Which may beappropriated by the Panchayat in thatparticular state?a) District Planning Committeesb) State Finance Commissionc) Finance Ministry of the State

Page 128: PolityPreviousYearQuestionBank · 1979. 8. 31.  · PolityPreviousYearQuestionBank PageNo.5 Company can give licence of trade to individuals and its own employees to trade in India:

Polity Previous Year Question Bank

www.laex.in https://elearn.laex.inPage No. 125

d) Panchayati Raj Ministry of thatState.8. If a panchayat is dissolved, electionsare to be held withina) 1 monthb) 3 monthc) 6 monthd) 1 year

9. In India, the first MunicipalCorporation was set up in which onamong the following?a) Calcuttab) Madrasc) Bombayd) Delhi

10. Consider the following Statement:1. Part IX of the Constitution of Indiacontains provisions for Panchayats and wasinserted by the Constitution (73rdAmendment ) Act, 1992.2. Part IX A of the Constitution ofIndia contains provisions for Municipalities- a Municipal Cancel and a municipalCorporation of every State.Which of the statements given belowis/are correct?a) 1 onlyb) 2 onlyc) Both 1 and 2d) Neither 1 nor 2

10. Match List I with List II and selectthe correct answer from the codesgiven below the lists:

List – I (LocalBodies)

List – II (States in1999)

A. Zilla Parishadat the SubDivisional level

B. Mandal PrajaParishad

C. Tribal CouncilsD. Absence of

VillagePanchayats

1. Andhra Pradesh2. Assam3. Mizoram4. Meghalaya

Codes:

A B C Da) 2 1 4 3b) 1 2 4 3c) 3 2 1 4d) 2 1 3 4

12. “………..instill into the vast millionsof workers, men and women, whoactually do the job, a sense ofpartnership and of cooperativeperformance ……..”. The above passagerelated toa) Planned Developmentb) Community Developmentc) Panchayat Raj Systemd) Integrated Development Programme

13. A college student desires to getelected to the Municipal Council to theMunicipal council of his city . Thevalidity of his nomination woulddepend on the important condition ,among others ,thata) He obtains permission from the

principal of his collegeb) He is a member of a political partyc) His name figures in the Voter’s listd) He files a declaration owing

allegiance of the constitutions

14. In the new Panchayati Raj billenacted in 1993 there areseveral fresh provisionsdeviating from the past .Whichone of the following is not suchprovision ?

a) A number of added responsibilities inthe area of agriculture ,ruraldevelopment, primary education andsocial forestry among others

b) Elections being made mandatory for allposts at the time they are due

c) A statutory representation for women inthe panchayats , upto a third of thestrength

d) Regular remuneration to the panchayatmembers, so as to ensure theirpunctuality and accountability

Page 129: PolityPreviousYearQuestionBank · 1979. 8. 31.  · PolityPreviousYearQuestionBank PageNo.5 Company can give licence of trade to individuals and its own employees to trade in India:

Polity Previous Year Question Bank

www.laex.in https://elearn.laex.inPage No. 126

15 Panchayati Raj was firstintroduced in India in October in1959 in which state?

a) Rajasthanb) Tamil Naduc) Keralad) Karnataka

Key & Explanation

1. Answer: B

Explanation: Balwant Rai MehtaCommittee submitted its report in 1957and recommended the establishment of thescheme of “democratic decentralisation”,which ultimately came to be known asPanchayati Raj.

Democratic Decentralization: It isthe process of devolving the functionsand resources of the state from the centreto the elected representatives at the lowerlevels so as to facilitate greater directparticipation of citizens in governance.

Educational Objective: Panchayati RajInstitutions were constitutionalisedthrough the 73rd ConstitutionalAmendment Act 1992. These areestablished in all the states of India by Actsof the state legislatures to build democracyat the grass root level.

Source: Indian Polity by Laxmikant (6thEdition) – page no: 38.1

2. Answer: B

Explanation: Gram Nyayalaya Act, 2008came into force in 2009.

Important Provisions:1) A Grama Nyayalaya is a mobile courtand exercises the powers of both Criminaland Civil Courts. Gram Nyayalaya trycriminal cases, civil suits, claims ordisputes which are specified in the FirstSchedule and the Second Schedule to theAct.

2) Gram Nyayalaya are supposed to try tosettle the disputes as far as possible bybringing about conciliation between theparties and for this purpose, it can makeuse of the appointed conciliators (Section26)

3) Section 27: Appointment ofConciliators— (1) For purposes of section26, the District Court shall, in consultationwith District Magistrate, prepare a panelconsisting of the names of social workers atthe village level having integrity forappointment as Conciliators who possesssuch qualifications and experience as maybe prescribed by the High Court.

Educational Objective: Objective of theAct is to provide inexpensive justice topeople in rural areas at their doorsteps.Each Gram Nyayalaya is a court of JudicialMagistrate of the first class and itspresiding officer is appointed by StateGovernment A Gram Nyayalaya isestablished for every Panchayat atintermediate level or a group of contiguousPanchayats at intermediate level in adistrict.

Source: http://prsindia.org/billtrack/the-gram-nyayalayas-bill-2008-432

3. Answer: B

Explanation: Art.243E: Duration of Panchayats, etc.-(1) Every Panchayat, unlesssooner dissolved under any law for thetime being in force, shall continue for fiveyears from the date appointed for its firstmeeting and no longer.

(3) An election to constitute a Panchayatshall be completed-

(a) before the expiry of its durationspecified in clause (1);

(b) before the expiration of a period of sixmonths from the date of its dissolution:

Provided that where the remainder ofthe period for which thedissolved Panchayat would have continued

Page 130: PolityPreviousYearQuestionBank · 1979. 8. 31.  · PolityPreviousYearQuestionBank PageNo.5 Company can give licence of trade to individuals and its own employees to trade in India:

Polity Previous Year Question Bank

www.laex.in https://elearn.laex.inPage No. 127

is less than six months, itshall not be necessary to hold any electionunder this clause for constituting thePanchayat for such period.

(4) A Panchayat constituted upon thedissolution of a Panchayat before theexpiration of its duration shall continueonly for the remainder of the period forwhich the dissolved Panchayat wouldhave continued under clause (1) had it notbeen so dissolved.

Minimum agefor election asa member of aPanchayat

21 Years 243F(1)(a)

Educational Objective: Panchayati RajInstitutions were constitutionalisedthrough the 73rd ConstitutionalAmendment Act 1992. These areestablished in all the states of India by Actsof the state legislatures to build democracyat the grass root level.

Source: Indian Polity by Laxmikant (6thEdition) – page no: 38.8 – 38.9

4. Answer: C

Explanation: 1) Balwant Rai MehtaCommittee submitted its report in 1957and recommended the establishment of thescheme of “democratic decentralisation”,which ultimately came to be known asPanchayati Raj 2) 72nd and 73rdConstitutional Amendments provides forparticipation of people by creating 3rd tier ofgovernance. Financial mobilization is notthe fundamental objective of PRls. Eventhough Panchayats have the powers tocollect revenuePolitical accountabilityrefers to the responsibility or obligation ofgovernment officials (both politicians andcivil servants) to act in the best interests ofsociety or face consequences.

Educational Objective: Panchayati RajInstitutions were constitutionalisedthrough the 73rd ConstitutionalAmendment Act 1992. These areestablished in all the states of India by Acts

of the state legislatures to build democracyat the grass root level.

Source: Indian Polity by Laxmikant (6thEdition) – page no: 38.1

5. Answer: C

Explanation: One of the objectives ofthe Act is: To provide self-rule for the bulkof the tribal population

Important Features of the Act:

1) A state legislation on the Panchayats inthe Scheduled Areas shall be in consonancewith the customary law, social andreligious practices and traditionalmanagement practices of communityresources. ---------- (To recognisetraditional rights)

2) To endow Panchayats with such powersto enable them to: a) Prevent alienation ofland in the Scheduled Areas and to takeappropriate action to restore anyunlawfully alienated land of ScheduledTribe b) Power to exercise control overmoney lending to the Scheduled Tribe --------------- (To free tribal from exploitation)

Create autonomous regions in tribalareas is not an objective under the Act.Sixth Schedule of the Constitution ofIndia allows for the formation ofAutonomous District Councils (regions)

Educational Objective: Provisions ofPart IX relating to the Panchayats are notapplicable to the 5th Schedule areas. Hence,Parliament has enacted the PESA Act,1996 to extend local self governance inthese areas with necessary modifications.

Source: Indian Polity by Laxmikant (6thEdition) – page no: 38.12 – 38.146. Answer: B

Explanation: Important Features ofPESA Act, 1996:

1) Recommendations of the Gram Sabha orthe Panchayats at the appropriate levelshall be mandatory for grant of prospecting

Page 131: PolityPreviousYearQuestionBank · 1979. 8. 31.  · PolityPreviousYearQuestionBank PageNo.5 Company can give licence of trade to individuals and its own employees to trade in India:

Polity Previous Year Question Bank

www.laex.in https://elearn.laex.inPage No. 128

licence or mining lease for minorminerals in the Scheduled Areas -------------- (3rd statement is not true – only minormineral)

2) State Legislature shall ensure that thePanchayats at the appropriate level and theGram Sabha are endowed specifically with:a) ownership of minor forest produce (2ndstatement is correct) b) Power to preventalienation of land in Scheduled Areas andto take appropriate action to restore anyunlawfully alienated land of the ScheduledTribe (1st statement is correct)

Educational Objective: Provisions ofPart IX relating to the Panchayats are notapplicable to the 5th Schedule areas. Hence,Parliament has enacted the PESA Act,1996 to extend local self governance inthese areas with necessary modifications.

Source: Indian Polity by Laxmikant (6thEdition) – page no: 38.14

7. Answer: B

Explanation:

Art. 243-Y: Governor of a state shall, afterevery five years, constitute a financecommission to review the financial positionof municipalities and makerecommendations to the Governor as to:

1) The principles that should govern:

a) The distribution between the state andthe municipalities of the net proceeds of thetaxes, duties, tolls and fees levied by thestate and allocation of shares amongst themunicipalities at all levels.

b) The determination of the taxes, duties,tolls and fees that may be assigned to themunicipalities

c) The grants-in-aid to the municipalitiesfrom the consolidation fund of the state.

Educational Objective: The system ofurban government was constitutionalisedthrough 74th constitutional Amendment Actof 1992 At the central level, the subject of

‘urban local government’ is dealt with bythe following three ministries:

a) Ministry of Housing and Urban Affairsb) Ministry of Defence (Cantonment

boards)c) Ministry of Home Affairs (in case of

UT’s)

Source: Indian Polity by Laxmikant (6thEdition) – page no: 39.4

8.73rd CONSTITUTIONALAMENDMENT ACT 1992It gaveconstitutional status to Panchayat RajInstitutions. The act provides for a five-year term of office to the panchayat atevery level. Fresh elections to constitute apanchayat shall be completed

(a) before the expiry of its duration offive years; or

(b) in case of dissolution, before theexpiry of a period of six months fromthe date of its dissolution.

Educational objective – to know aboutSalient Features and provisions of thePanchayat Act Source – Indian Polityby Laxmikant ( 5th edition )

9. The first Municipal Corporation of Indiawas set up in Madras in 1687 with theformation of Madras Municipal Corporation.Later it was established in the cities ofCalcutta and Bombay in 1726.

According to the 1991 Census of India,urban local bodies are classified into thefour major categories1. Municipal corporation2. Municipality (municipal council,municipal board, municipalcommittee)3. Town area committee4. Notified area committeePresently they are established in the statesby the acts of the concerned statelegislatures, and in the unionterritories by the acts of the

Page 132: PolityPreviousYearQuestionBank · 1979. 8. 31.  · PolityPreviousYearQuestionBank PageNo.5 Company can give licence of trade to individuals and its own employees to trade in India:

Polity Previous Year Question Bank

www.laex.in https://elearn.laex.inPage No. 129

Parliament of India. About MunicipalCorporation A municipal corporation has three

authorities, namely, the council, thestanding committees and thecommissioner.

The Council is the deliberative andlegislative wing of the corporation.

It consists of the Councillors directlyelected by the people, as well as a fewnominated persons

The Council is headed by a Mayor. He isassisted by a Deputy Mayor. He iselected in a majority of the states for aone-year renewable term.

The standing committees are created tofacilitate the working of the council,which is too large in size.

The municipal commissioner is the chiefexecutive authority of the corporation.

Educational objective –to know aboutthe evolution of Urban Local Bodies

Source – INDIAN POLITY BYLAXMIKANTH pg no.39.6

10.73rdConstitutionalAmendmentAct

Added new Part-IXentitled as ‘ThePanchayats’

Articles 243 to 243O.

EleventhSchedule wasadded to theConstitution whichcontains 29functional itemsof the panchayats.

74thConstitutionalAmendmentAct

Added a new PartIX-A entitled as‘TheMunicipalities’

Articles 243P to243-ZG.

Twelfth Schedulewas added whichcontains eighteenfunctional items ofmunicipalities.

Article 243Q deals with Constitution ofMunicipalitiesIt deals with setting up of

(a) a Nagar Panchayat, an area intransition from a rural area to an urbanarea(b) a Municipal Council for a smallerurban area; and(c) a Municipal Corporation for a largerurban area

Educational objective –to know aboutthe constitutional provisions andArticles related to Panchayath raj

Source – INDIAN CONSTITUTION

11. Refer table 39.4Source – Indian Polity by Laxmikant( 5th edition )

12. The United Nations definescommunity development as "a processwhere community members come togetherto take collective action and generatesolutions to common problems".

Educational objective –to know aboutthe definitions of importantterminologies

Source UN WEBSITE

13. Qualification for contesting MunicipalCorporation elections She/he must be a citizen of India She/he must have attained the age of

21 years His/her name is registered in the

Electoral Roll of a ward She/he is not earlier disqualified for

contesting Municipal Corporationelections.

She/he must not be an employee of anyMunicipal Corporation in India

Hence, he/she should satisfy the conditionof OPTION C i.e His name figures in thevoters list.For which he must have attained

Page 133: PolityPreviousYearQuestionBank · 1979. 8. 31.  · PolityPreviousYearQuestionBank PageNo.5 Company can give licence of trade to individuals and its own employees to trade in India:

Polity Previous Year Question Bank

www.laex.in https://elearn.laex.inPage No. 130

18 years. If he/she doesn’t fulfil this criteriathen the criteria of 21 years is not satisfied.Educational objective –to know aboutthe eligibility criteria for conductingvarious elections

Source – ECI Website

14. The Panchayat Raj Act of 1973 do nothave any provisions related torenumeration of panchayat membersEducational objective –to have acomparative study between the oldand new provisions about panchayatsystem and its evolutionSource – Indian Polity by Laxmikant( 5th edition )

15. Balwant Rai Mehta CommitteeIn January 1957, the Government of Indiaappointed a committee to examine theworking of the Community DevelopmentProgramme (1952) and the NationalExtension Service (1953) and to suggestmeasures for their better working.

Recommendations Establishment of a three-tier

panchayati raj system The village panchayat should be

constituted with directly electedrepresentatives, whereas the panchayatsamiti and zila parishad should beconstituted with indirectly electedmembers.

The panchayat samiti should be theexecutive body while the zila parishadshould be the advisory, coordinatingand supervisory body

These recommendations of the committeewere accepted by the National DevelopmentCouncil in January 1958. Rajasthan wasthe first state to establish Panchayati Raj,which was followed by Andhra Pradesh.Educational objective – to know aboutthe evolution of Panchayat system andvarious committees related topanchayatsSource – Indian Polity byLaxmikant ( 5th edition )